Vous êtes sur la page 1sur 78

1.

A 22-year-old previously healthy patient presents to your ‫ ﺣﻀﺮﺕ ﺇﻟﻰ ﺍﻟﻌﻴﺎﺩﺓ‬،‫ ﻭﻫﻲ ﺑﺼﺤﺔ ﺟﻴﺪﺓ‬،‫ ﻋﺎﻣﺎ‬22 ‫ﻣﺮﻳﻀﺔ ﻋﻤﺮﻫﺎ‬
office complaining of abnormal vaginal bleeding. Your ‫ ﺃﻱ ﻣﻦ ﺍﻟﺘﺎﻟﻲ ﻳﻌﺘﺒﺮ ﻣﻦ‬.‫ﻭﻫﻲ ﺗﺸﻜﻮ ﻣﻦ ﻧﺰﻳﻒ ﻣﻬﺒﻠﻲ ﻏﻴﺮ ﻁﺒﻴﻌﻲ‬
first consideration in the differential diagnosis is which :‫ﺃﻭﻝ ﺍﻹﻫﺘﻤﺎﻣﺎﺕ ﻓﻲ ﺍﻟﺘﺸﺨﻴﺺ ﺍﻟﺘﻔﺮﻳﻘﻲ؟‬
one of the following ? .‫ ﺍﻹﻋﺘﻼﻝ ﺍﻟﺨﺜﺎﺭﻱ‬-A
A- Coagulopathy .‫ ﺟﺴﻢ ﻏﺮﻳﺐ‬-B
B- Foreign body .‫ ﺍﻻﻧﺘﺎﻥ‬-C
C- Infection .‫ ﺍﻟﺤﻤﻞ‬-D
D- Pregnancy .‫ ﺍﻟﻜﺪﻣﺔ‬-E
E- Trauma
D
Une patiente de 22 ans, en bonne santé, se présente à
votre cabinet se plaignant d'un saignement vaginal
anormal. Votre première considération dans le diagnostic
différentiel est laquelle des suivantes?
A- Coagulopathie
B- Corps étranger
C- Infection
D- Grossesse
E- Traumatisme
2. What is the most appropriate treatment for a patient with ‫ﻣﺎ ﻫﻮ ﺍﻟﻌﻼﺝ ﺍﻷﻛﺜﺮ ﻣﻼءﻣﺔ ﻟﻤﺮﻳﺾ ﻳﻌﺎﻧﻲ ﻣﻦ ﺗﺤﺼﻲ ﻗﻨﺎﺓ‬
choledocholithiasis 2 years after cholecystectomy? :‫ﺍﻟﺼﻔﺮﺍء ﺑﻌﺪ ﻋﺎﻣﻴﻦ ﻋﻠﻰ ﺇﺟﺮﺍﺋﻪ ﻟﻌﻤﻠﻴﺔ ﺍﺳﺘﺌﺼﺎﻝ ﺍﻟﻤﺮﺍﺭﺓ؟‬
a- chenodesoxycolic acid .‫ ﺣﻤﺾ ﺷﻴﻨﻮ ﺩﻳﺰﻭﻛﺴﻲ ﻛﻮﻟﻴﻚ‬-A
b- endoscopic sphincterotomy and stone .‫ ﺑﻀﻊ ﺍﻟﻤﺼﺮﺓ ﺑﺎﻟﻤﻨﻈﺎﺭ ﻭﺍﺳﺘﺨﺮﺍﺝ ﺍﻟﺤﺼﺎﺓ‬-B
extraction .T ‫ ﺑﻀﻊ ﻗﻨﺎﺓ ﺍﻟﺼﻔﺮﺍء ﺑﺄﻧﺒﻮﺏ‬-C
c- choledocotomy with T tube .‫ ﺗﻔﺘﻴﺖ ﺍﻟﺤﺼﺎﺓ‬-D
d- lithotripsy ‫ ﻛﻞ ﻣﺎ ﺳﺒﻖ‬-E
e- All of the above
B
Quel est le traitement le plus approprié pour un patient
atteint de cholédocholithiase, 2 ans après une
cholécystectomie?
a- acide chénodésoxycolique
b- sphinctérotomie endoscopique et extraction
du calcul
c- Cholédocotomie avec tube en T (Kehr)
d- Lithotritie
e- Tous les précédents
3. A 65 years old man, diabetic, on sublingual nitrates to ‫ ﻳﺴﺘﺨﺪﻡ ﻧﻴﺘﺮﺍﺕ‬،‫ ﻳﻌﺎﻧﻲ ﻣﻦ ﺩﺍء ﺍﻟﺴﻜﺮﻱ‬،‫ ﻋﺎﻣﺎ‬65 ‫ﺭﺟﻞ ﻋﻤﺮﻩ‬
unstable angina, presented for erectile dysfunction, ‫ ﻭﻫﻮ ﻳﺸﻜﻮ ﻣﻦ‬،‫ﺗﺤﺖ ﺍﻟﻠﺴﺎﻥ ﺑﺴﺒﺐ ﺍﻟﺨﻨﺎﻕ ﺍﻟﺼﺪﺭﻱ ﻏﻴﺮ ﺍﻟﻤﺴﺘﻘﺮ‬
universal and progressive since 5 years. Which of the ‫ ﺃﻱ ﻣﻦ ﻁﺮﻕ‬.‫ ﺳﻨﻴﻦ‬5 ‫ﺧﻠﻞ ﻋﺎﻡ ﻓﻲ ﻭﻅﻴﻔﺔ ﺍﻻﻧﺘﺼﺎﺏ ﻳﺘﻔﺎﻗﻢ ﻣﻨﺬ‬
following treatment is an absolute contraindication with :‫ﺍﻟﻤﻌﺎﻟﺠﺔ ﺍﻟﺘﺎﻟﻴﺔ ﻫﻲ ﻣﻀﺎﺩ ﺍﺳﺘﻄﺒﺎﺏ ﻣﻄﻠﻖ ﻣﻊ ﺍﻟﻨﻴﺘﺮﺍﺕ؟‬
nitrates? .‫ ﺳﻠﺪﻳﻨﺎﻓﻴﻞ‬-A
a. Sildenafil .‫ ﺟﻬﺎﺯ ﺗﺨﻠﻴﺔ‬-B
b. Vacuum device .‫ ﻣﺘﻤﻤﺎﺕ ﺍﻟﺠﻨﺴﻨﻎ‬-C
c. Ginsec supplements .‫ ﺩﺍﺧﻞ ﺍﻹﺣﻠﻴﻞ‬E ‫ ﺑﺮﻭﺳﺘﺎﻏﻼﻧﺪﻳﻦ‬-D
d. Intraurethral Prostaglandin E ‫ ﻛﻞ ﻣﺎ ﺳﺒﻖ‬-E
e. All of the above
A
Un homme de 65 ans, diabétique, sous nitrates sub lingual
pour angine de poitrine, se présente pour dysfonction
erectile, généralisée et progressive depuis 5 ans. Quel
traitement est une contreindication absolue avec les
nitrates ?
a. Sildénafil
b. Vacuum
c. Ginsec
d. Prostaglandine E intraurétrale
e. Tous les précédents

1A
4. A 25 years old man presented to the emergency for a ‫ ﻋﺎﻣﺎ ﺗﻌﺮﺽ ﻟﺤﺎﺩﺙ ﺳﻴﺎﺭﺓ ﻧﻘﻞ ﻋﻠﻰ ﺃﺛﺮﻩ‬25 ‫ﺭﺟﻞ ﻳﺒﻠﻎ ﻣﻦ ﺍﻟﻌﻤﺮ‬
motor vehicle accident. He was hemodynamically stable. ‫ ﺃﻅﻬﺮ ﺍﻟﻤﺴﺢ‬.‫ ﺍﻟﺪﻳﻨﺎﻣﻴﻜﻴﺔ ﺍﻟﺪﻣﻮﻳﺔ ﻣﺴﺘﻘﺮﺓ‬.‫ﺇﻟﻰ ﻗﺴﻢ ﺍﻟﻄﻮﺍﺭﺉ‬
A CT scan of the abdomen and pelvis with IV contrast ،‫ ﺑﺎﻟﻤﺸﺎﺭﻛﺔ ﻣﻊ ﻣﺎﺩﺓ ﻅﻠﻴﻠﺔ ﻭﺭﻳﺪﻳﺎ‬،‫ﺍﻟﻤﻘﻄﻌﻲ ﻟﻠﺒﻄﻦ ﻭﺍﻟﺤﻮﺽ‬
showed a 10 cm retroperitoneal hematoma resulting from ‫ ﺳﻢ ﻧﺎﺗﺞ ﻋﻦ ﺇﺻﺎﺑﺔ‬10 ‫ﻭﺟﻮﺩ ﻭﺭﻡ ﺩﻣﻮﻱ ﺧﻠﻒ ﺍﻟﺼﻔﺎﻕ ﺑﺤﺠﻢ‬
a grade 4 right renal injury with extravasation of IV ‫ ﻓﻲ ﺍﻟﻜﻠﻴﺔ ﺍﻟﻴﻤﻨﻰ ﻣﻊ ﺗﺴﺮﺏ ﺍﻟﻤﺎﺩﺓ ﺍﻟﻈﻠﻴﻠﺔ ﺍﻟﻮﺭﻳﺪﻳﺔ‬4 ‫ﺫﺍﺕ ﺩﺭﺟﺔ‬
contrast from the lower calyx, with a bladder rupture with ‫ ﻭﺗﻤﺰﻕ ﺍﻟﻤﺜﺎﻧﺔ ﻣﻊ ﺗﺴﺮﺏ ﺍﻟﻤﺎﺩﺓ ﺍﻟﻈﻠﻴﻠﺔ ﺩﺍﺧﻞ‬،‫ﻣﻦ ﺍﻟﻜﺄﺱ ﺍﻟﺴﻔﻠﻲ‬
extravasation of the contrast into the peritoneum. No ‫ ﺃﻱ ﻣﻦ‬.‫ ﻟﻢ ﻳﻈﻬﺮ ﺃﻱ ﻣﺸﺎﻛﻞ ﺃﺧﺮﻯ ﻓﻲ ﺍﻟﺒﻄﻦ ﺃﻭ ﺍﻟﺤﻮﺽ‬.‫ﺍﻟﺼﻔﺎﻕ‬
other abdominal or pelvic problem was detected. Which :‫ﺍﻟﺘﺎﻟﻲ ﻫﻮ ﻣﺆﺷﺮ ﻹﺟﺮﺍء ﻋﻤﻞ ﺟﺮﺍﺣﻲ؟‬
of the following is an indication for surgical management? .‫ ﺳﻢ ﻭﺭﻡ ﺩﻣﻮﻱ ﺧﻠﻒ ﺍﻟﺼﻔﺎﻕ‬10 -A
a. 10 cm retroperitoneal hematoma .4 ‫ ﺇﺻﺎﺑﺔ ﺍﻟﻜﻠﻴﺔ ﺍﻟﻴﻤﻨﻰ ﻣﻦ ﺍﻟﺪﺭﺟﺔ‬-B
b. Grade 4 right renal injury ‫ ﺗﺴﺮﺏ ﺍﻟﻤﺎﺩﺓ ﺍﻟﻈﻠﻴﻠﺔ ﺍﻟﻮﺭﻳﺪﻳﺔ ﻣﻦ ﺍﻟﻜﺄﺱ ﺍﻟﻜﻠﻮﻱ‬-C
c. Extravasation of IV contrast from a lower renal .‫ﺍﻟﺴﻔﻠﻲ‬
calyx .‫ ﺗﻤﺰﻕ ﺍﻟﻤﺜﺎﻧﺔ ﺩﺍﺧﻞ ﺍﻟﺼﻔﺎﻕ‬-D
d. Intraperitoneal bladder rupture ‫ ﻛﻞ ﺍﻟﻜﻞ ﺍﻹﺟﺎﺑﺎﺕ ﺍﻟﺴﺎﺑﻘﺔ ﺧﻄﺄ‬-E
e. None of the above

Un homme de 25 ans se présente aux urgences pour un D


accident de la voie publique. Ses signes vitaux sont
stables. Le CT scan abdomino-pelvien avec injection de
produit de contraste a montré un hématome
rétropéritonéal de 10 cm provenant d’un trauma rénal de
grade 4 avec extravasation du produit de contraste au
niveau du calice inférieur, avec une rupture vésicale et
extravasation du produit de contraste en intapéritonéal.
Pas d’autres pathologies abdominales ou pelviennes.
Quelle réponse parmi les suivantes est considérée une
indication opératoire ?
a. L’hématome rétropéritonéal de 10 cm
b. Lésion rénale droite grade 4
c. Extravasation du produit de contraste du calice
inférieur
d. Rupture vésicale intrapéritonéale
e. Acucun des précédents
5. Surgical treatment of congenital cataract :‫ﺇﻥ ﺍﻟﻌﻼﺝ ﺍﻟﺠﺮﺍﺣﻲ ﻟﻠﺴﺎﺩ ﺍﻟﺨﻠﻘﻲ‬
a. Is mainly cosmetic .‫ ﻫﻮ ﺗﺠﻤﻴﻠﻲ ﺑﺎﻷﺳﺎﺱ‬-A
b. Indicated only if there is a strabismus. .‫ ﻳﺴﺘﺪﻋﻲ ﺇﺟﺮﺍﺅﻩ ﻓﻲ ﺣﺎﻝ ﻭﺟﻮﺩ ﺣﻮﻝ‬-B
c. To prevent amblyopia. .‫ ﻫﻮ ﻟﻤﻨﻊ ﺣﺪﻭﺙ ﺍﻟﻐﻤﺶ‬-C
d. Shouldn’t be done before the age of 6 month. .‫ ﻳﺠﺐ ﻋﺪﻡ ﺇﺟﺮﺍﺅﻩ ﻗﺒﻞ ﻋﻤﺮ ﺍﻟﺴﺘﺔ ﺃﺷﻬﺮ‬-D
e. All of the above ‫ ﻛﻞ ﺍﻹﺟﺎﺑﺎﺕ ﺍﻟﺴﺎﺑﻘﺔ‬-E
C
Le traitement chirurgical de la cataracte congénitale
a- Est principalement cosmétique
b- Est indiqué seulement s'il y a un strabisme.
c- Est indiqué pour prévenir l'amblyopie.
d- Ne devrait pas être fait avant l'âge de 6 mois.
e- Tous les précédents

2A
6. Exophthalmos of the inflammatory orbital pseudotumor ‫ ﻟﺪﻳﻪ‬،‫ﺍﻟﺠﺤﻮﻅ ﺍﻟﻨﺎﺗﺞ ﻋﻦ ﻣﺘﻼﺯﻣﺔ ﻭﺭﻡ ﻛﺎﺫﺏ ﻣﺤﻮﺭﻱ ﺍﻟﺘﻬﺎﺑﻲ‬
syndrome has one essential characteristic: :‫ﺻﻔﺔ ﻭﺍﺣﺪﺓ ﺃﺳﺎﺳﻴﺔ‬
a. Benign lymphoid proliferation of the intraorbital .‫ ﺗﻜﺎﺛﺮ ﻟﻤﻔﺎﻭﻱ ﺣﻤﻴﺪ ﻓﻲ ﺍﻷﻧﺴﺠﺔ ﺍﻟﻤﺤﻮﺭﻳﺔ ﺍﻟﺪﺍﺧﻠﻴﺔ‬-A
tissues. .‫ ﺣﺴﺎﺱ ﻟﻠﻌﻼﺝ ﺑﺎﻟﻜﻮﺭﺗﻴﺰﻭﻥ‬-B
b. Sensitive to corticotherapy. .‫ ﺛﻨﺎﺋﻲ ﺍﻟﺠﺎﻧﺐ‬-C
c. Bilateral. .‫ ﻧﺎﺑﺾ‬-D
d. Pulsatile. ‫ ﻛﻞ ﺍﻹﺟﺎﺑﺎﺕ ﺍﻟﺴﺎﺑﻘﺔ ﺧﻄﺎ‬-E
e. None of the above
B
L'exophtalmie du syndrome pseudotumorale orbitaire
inflammatoire a une caractéristique essentielle:
a- Prolifération lymphoïde bénigne des tissus
intraorbitaires.
b- Sensible à la corticothérapie.
c- Bilatérale.
d- Pulsatile.
e- Aucun des précédents
7. Central retinal artery occlusion gives:(one wrong answer) :(‫)ﺍﺧﺘﺮ ﺍﻹﺟﺎﺑﺔ ﺍﻟﺨﻄﺄ‬:‫ﺍﻧﺴﺪﺍﺩ ﺷﺮﻳﺎﻥ ﺍﻟﺸﺒﻜﻴﺔ ﺍﻟﻤﺮﻛﺰﻱ ﻳﺆﺩﻱ ﺇﻟﻰ‬
a- Is a medical emergency. .‫ ﻋﻤﻞ ﺟﺮﺍﺣﻲ ﻋﺎﺟﻞ‬-A
b- Retinal edema. .‫ ﻭﺫﻣﺔ ﺷﺒﻜﻴﺔ‬-B
c- Homonymous hemianopsia. .‫ ﻋﻤﻰ ﺷﻘﻲ ﻣﻤﺎﺛﻞ ﺍﻟﺠﺎﻧﺐ‬-C
d- Cherry red spot at the macula. .‫ ﺑﻘﻌﺔ ﻛﺮﺯﻳﺔ ﺣﻤﺮﺍء ﻓﻲ ﺍﻟﺒﻘﻌﺔ‬-D
e- All of the above are correct ‫ ﻛﻞ ﺍﻹﺟﺎﺑﺎﺕ ﻫﻲ ﺻﺤﻴﺤﺔ‬-E
C
L'occlusion de l'artère centrale de la rétine donne: (choisir
la mauvaise réponse)
a- Est une urgence médicale
b- Œdème rétinien.
c- Hémianopsie homonyme..
d- Tache rouge cerise au niveau de la macula
e- Toutes les réponses sont justes
8. Acute anterior uveitis is associated with (choose the :(‫ﺍﻟﺘﻬﺎﺏ ﺍﻟﻌﻨﺒﻴﺔ ﺍﻷﻣﺎﻣﻲ ﺍﻟﺤﺎﺩ ﻳﺘﺮﺍﻓﻖ ﻣﻊ )ﺍﺧﺘﺮ ﺍﻹﺟﺎﺑﺔ ﺍﻟﺨﻄﺄ‬
wrong answer) .‫ ﺍﻟﺨﻼﻳﺎ ﺍﻻﻟﺘﻬﺎﺑﻴﺔ ﻓﻲ ﺍﻟﻐﺮﻓﺔ ﺍﻷﻣﺎﻣﻴﺔ‬-A
a. inflammatory cells in the anterior chamber ‫ ﺍﻟﻮﺫﻣﺔ ﺍﻟﺒﻘﻌﻴﺔ ﺍﻟﻜﻴﺴﻴﺔ ﺍﻟﺘﻲ ﻫﻲ ﻣﻦ ﺍﻟﻤﻀﺎﻋﻔﺎﺕ‬-B
b. cystoid macular edema is a common .‫ﺍﻟﺸﺎﺋﻌﺔ‬
complication .‫ ﻭﺟﻮﺏ ﺍﻟﻤﻌﺎﻟﺠﺔ ﺑﺴﺘﻴﺮﻭﺋﻴﺪ ﺟﻬﺎﺯﻱ‬-C
c. should be treated by systemic steroids .‫ ﺇﻣﻜﺎﻧﻴﺔ ﺃﻥ ﻳﻜﻮﻥ ﻣﺠﻬﻮﻝ ﺍﻟﺴﺒﺐ‬-D
d. can be idiopathic
C
L'uvéite antérieure aiguë est associée à (choisir la
mauvaise réponse)
a- cellules inflammatoires dans la chambre
antérieure
b- L'œdème maculaire cystoïde est une complication
fréquente
c- devrait être traité par des stéroïdes systémiques
d- peut être idiopathique

3A
9. The preferred view to visualize the impaction of the ‫ﺍﻟﻤﺸﻬﺪ ﺍﻟﺸﻌﺎﻋﻲ ﺍﻟﻤﻔﻀﻞ ﻟﺮﺅﻳﺔ ﺍﻧﺤﺸﺎﺭ ﺭﺃﺱ ﺍﻟﻌﻀﺪ ﻓﻲ ﺧﻠﻊ‬
humeral head in posterior shoulder dislocation is: :‫ﺍﻟﻜﺘﻒ ﺍﻟﺨﻠﻔﻲ ﻫﻮ‬
A. AP (anteroposterior) view .AP ‫ ﻣﺸﻬﺪ‬-A
B. AP view with 30° upward inclination of the .‫ﻣﻴﻞ ﻋﻠﻮﻱ ﻷﻧﺒﻮﺏ ﺍﻷﺷﻌﺔ ﺍﻟﺴﻴﻨﻴﺔ‬°30 ‫ ﻣﻊ‬AP ‫ ﻣﺸﻬﺪ‬-B
radiographic tube .‫ ﻣﺸﻬﺪ ﺇﺑﻄﻲ‬-C
C. Axillary view .‫ ﻣﺸﻬﺪ ﻋﺒﺮ ﺍﻟﺼﺪﺭ‬-D
D. Trans-thoracic view .‫ ﻛﻞ ﺍﻷﺟﻮﺑﺔ ﺃﻋﻼﻩ ﻏﻴﺮ ﺻﺤﻴﺤﺔ‬-E
E. None of the above
C
L’incidence radiologique préférée pour visualiser
l'impaction de la tête humérale dans la luxation de
l'épaule postérieure est:
A. Vue- incidence AP (antérieure postérieure)
B. Vue- incidence AP avec inclinaison de 30°
vers le haut du tube radiographique
C. Vue- incidence axillaire
D. Vue- incidence trans-thoracique
E. Aucune de ces réponses
10. 30-year-old female presents complaining of several ‫ ﻋﺎﻣﺎ ﺗﺸﻜﻮ ﻣﻦ ﻧﻮﺑﺎﺕ ﺩﻭﺍﺭ‬30 ‫ﺃﻧﺜﻰ ﺗﺒﻠﻎ ﻣﻦ ﺍﻟﻌﻤﺮ‬
episodes of vertigo. Her vertigo is induced by head ،‫ﺇﻥ ﺍﻟﺪﻭﺍﺭ ﻧﺎﺟﻢ ﻋﻦ ﺩﻭﺭﺍﻥ ﺍﻟﺮﺃﺱ ﻧﺤﻮ ﺍﻟﻴﺴﺎﺭ‬.‫ﻋﺪﻳﺪﺓ‬
rotation to the left side; it persists for few seconds and is ‫ ﻻ‬.‫ﺣﻴﺚ ﻳﺪﻭﻡ ﻟﺒﻀﻊ ﺛﻮﺍﻧﻲ ﻭﻳﺘﺮﺍﻓﻖ ﻣﻊ ﻏﺜﻴﺎﻥ ﻭﺇﻗﻴﺎء‬
associated with nausea and vomiting. No ear tinnitus or ‫ ﻋﻨﺪ‬.‫ﻳﻮﺟﺪ ﻁﻨﻴﻦ ﺃﻭ ﺇﻣﺘﻼء ﻓﻲ ﺍﻷﺫﻧﻴﻦ ﺧﻼﻝ ﺍﻟﻨﻮﺑﺎﺕ‬
fullness occurs during attacks. Upon physical exam, all ‫ ﺗﺒﻴﻦ ﺃﻥ ﺟﻤﻴﻊ ﺍﻷﻋﺼﺎﺏ ﺍﻟﻘﺤﻔﻴﺔ‬،‫ﺍﻟﻔﺤﺺ ﺍﻟﺴﺮﻳﺮﻱ‬
cranial nerves were found intact, and her Romberg test ‫ﻟﻘﺪ ﺣﺼﻠﺖ ﺭﺃﺭﺃﺓ‬.‫ ﻭﺍﺧﺘﺒﺎﺭ ﺭﻭﻣﺒﻴﺮﻍ ﺳﻠﺒﻲ‬،‫ﺳﻠﻴﻤﺔ‬
was negative. A left beating nystagmus was induced ‫ﻣﺘﺠﻬﺔ ﻧﺤﻮ ﺍﻟﺸﻤﺎﻝ ﻋﻨﺪ ﺗﺤﺮﻳﻚ ﺍﻟﻤﺮﻳﻀﺔ ﻣﻦ ﻭﺿﻌﻴﺔ‬
when moving the patient from the sitting position to the ‫ﺍﻟﺠﻠﻮﺱ ﺇﻟﻰ ﻭﺿﻌﻴﺔ ﺍﻻﺳﺘﻠﻘﺎء ﻣﻊ ﺗﺪﻟﻲ ﺍﻟﺮﺃﺱ ﺇﻟﻰ‬
supine position with head tilted downward to the left ‫ ﻟﻘﺪ ﺗﻮﻗﻒ ﺍﻟﺪﻭﺍﺭ ﻭﺍﻟﺮﺃﺭﺃﺓ‬.‫ﺍﻷﺳﻔﻞ ﻋﻠﻰ ﺍﻟﺠﺎﻧﺐ ﺍﻷﻳﺴﺮ‬
side. Her vertigo and nystagmus resolved after few :‫ ﻣﺎ ﻫﻮ ﺍﻟﺘﺸﺨﻴﺺ ﺍﻷﻛﺜﺮ ﺗﺮﺟﻴﺤﺎ؟‬.‫ﺑﻌﺪ ﺑﻀﻊ ﺛﻮﺍﻧﻲ‬
seconds. What is the most likely diagnosis? .‫ ﺩﺍء ﻣﻨﻴﻴﺮ‬-A
a) Meniere’s disease .‫ ﺍﻟﺘﻬﺎﺏ ﺍﻟﻌﺼﺐ ﺍﻟﺪﻫﻠﻴﺰﻱ ﺍﻷﻳﺴﺮ‬-B
b) Left vestibular neuritis ‫ ﺩﻭﺍﺭ ﻭﺿﻌﻲ ﺣﻤﻴﺪ ﻧﺎﺗﺞ ﻋﻦ ﺍﻟﻘﻨﺎﺓ ﻧﺼﻒ‬-C
c) Benign positional vertigo due to posterior .‫ﺍﻟﺪﺍﺋﺮﻳﺔ ﺍﻟﺨﻠﻔﻴﺔ‬
semicircular canal ‫ ﺩﻭﺍﺭ ﻭﺿﻌﻲ ﺣﻤﻴﺪ ﻧﺎﺗﺞ ﻋﻦ ﺍﻟﻘﻨﺎﺓ ﻧﺼﻒ‬-D
d) Benign positional vertigo due to horizontal .‫ﺍﻟﺪﺍﺋﺮﻳﺔ ﺍﻷﻓﻘﻴﺔ‬
semicircular canal .‫ ﻧﺰﻳﻒ ﻣﺨﻴﺨﻲ‬-E
e) Cerebellar hemorrhage

Une femme de à 30 ans se plaint de plusieurs épisodes de D


vertige. Son vertige est induit par la rotation de la tête sur
le côté gauche; il persiste pendant quelques secondes et
est associé à des nausées et des vomissements. Aucun
acouphène ou plénitude de l'oreille ne se produit pendant
les épisodes. À l'examen physique, tous les nerfs crâniens
sont intacts, et son test de Romberg était négatif. Un
nystagmus battant à gauche a été induit lors du
déplacement du patient de la position assise à la position
couchée avec la tête inclinée vers le bas sur le côté gauche.
Son vertige et son nystagmus se sont résolus après
quelques secondes. Quel est le diagnostic le plus
probable?
a) Maladie de Ménière
b) Névrite vestibulaire gauche
c) Vertige positionnel bénin dû au canal semi-
circulaire postérieur
d) Vertige positionnel bénin dû au canal semi-
circulaire horizontal
e) Hémorragie cérébelleuse

4A
11. Renal tubular acidosis: :‫ﺍﻟﺘﺤﻤﻀﻦ ﺍﻟﻨﺒﻴﺒﻲ ﺍﻟﻜﻠﻮﻱ‬
A. Is characterized by hypochloremic metabolic .‫ ﻳﺗﺻﻑ ﺑﺗﺣﻣﺿﻥ ﺍﺳﺗﻘﻼﺑﻲ ﻧﺎﺗﺞ ﻋﻥ ﻧﻘﺹ ﻛﻠﻭﺭﻳﺩ ﺍﻟﺩﻡ‬-
acidosis ‫ ﻳﺗﺻﻑ ﺑﺗﺣﻣﺿﻥ ﺍﺳﺗﻘﻼﺑﻲ ﻣﻊ ﻓﺟﻭﺓ ﺷﺭﺳﺑﻳﺔ ﻣﺻﻠﻳﺔ‬-
B. Is characterized by normal serum anion gap .‫ﻁﺑﻳﻌﻳﺔ‬
metabolic acidosis ‫ ﻳﺗﺻﻑ ﺑﺗﺣﻣﺿﻥ ﺍﺳﺗﻘﻼﺑﻲ ﻧﺎﺗﺞ ﻋﻥ ﻧﻘﺹ ﻛﻠﻭﺭﻳﺩ ﺍﻟﺩﻡ‬-
C. Is characterized by hyperchloremic high .‫ﻭﻓﺟﻭﺓ ﺷﺭﺳﺑﻳﺔ ﻋﺎﻟﻳﺔ‬
serum anion gap metabolic acidosis ‫ ﻳﺗﺻﻑ ﺑﺗﺣﻣﺿﻥ ﺍﺳﺗﻘﻼﺑﻲ ﻧﺎﺗﺞ ﻋﻥ ﻧﻘﺹ ﻛﻠﻭﺭﻳﺩ ﺍﻟﺩﻡ‬-
D. Is characterized by hypochloremic normal .‫ﻭﻓﺟﻭﺓ ﺷﺭﺳﺑﻳﺔ ﻁﺑﻳﻌﻳﺔ‬
serum anion gap metabolic acidosis ‫ ﻛﻝ ﺍﻷﺟﺎﺑﺎﺕ ﺧﻁﺄ‬-
E. None of the above 
B
Acidose tubulaire rénale:
A. est caractérisée par une acidose métabolique
hypochlorémique
B. est caractérisée par une acidose métabolique
avec trou anionique sérique normal
C.est caractérisée par une acidose métabolique
hyperchlorémique et trou anionique élevé
D. est caractérisée par une acidose métabolique
hypochlorémique et trou anionique normal
E. aucun des précédents
12. An 18 yo male presents with palpable purpura and ‫ ﻋﺎﻣﺎ ﻳﻌﺎﻧﻲ ﻣﻦ ﻓﺮﻓﺮﻳﺔ ﻣﻠﻤﻮﺳﺔ ﻭﺃﻟﻢ‬18 ‫ﺷﺎﺏ ﻳﺒﻠﻎ ﻣﻦ ﺍﻟﻌﻤﺮ‬
polyarthralgia. The following is true in Henoch-Schonlein -‫ ﺃﻱ ﻣﻦ ﺍﻟﺘﺎﻟﻲ ﺻﺤﻴﺢ ﻓﻲ ﺣﺎﻟﺔ ﻓﺮﻓﺮﻳﺔ ﻫﻴﻨﻮﺵ‬.‫ﻣﻔﺎﺻﻞ ﻣﺘﻌﺪﺩ‬
purpura: :‫ﺷﻮﻧﻠﺌﻴﻦ؟‬
A. The disease is more frequent in adults than in .‫ ﻳﺤﺪﺙ ﻫﺬﺍ ﺍﻟﻤﺮﺽ ﻋﻨﺪ ﺍﻟﻜﺒﺎﺭ ﺃﻛﺜﺮ ﻣﻦ ﺍﻟﺼﻐﺎﺭ‬-A
children .‫ ﻻ ﺗﺘﻌﺮﺽ ﻋﻀﻠﺔ ﺍﻟﻘﻠﺐ ﻟﻺﺻﺎﺑﺔ ﻋﻨﺪ ﺍﻟﻜﺒﺎﺭ‬-B
B. The myocardium is not involved in the adult ‫ ﺍﻧﺘﺎﻧﺎﺕ ﺍﻟﻄﺮﻕ ﺍﻟﺘﻨﻔﺴﻴﺔ ﺍﻟﻌﻠﻮﻳﺔ ﻭﺍﻟﺘﺤﺼﻴﻦ ﺍﻟﻤﻨﺎﻋﻲ ﻗﺪ‬-C
form .‫ﺗﺸﻜﻞ ﻣﻮﻟﺪ ﺿﺪ ﻣﺤﺮﺽ‬
C. Upper respiratory tract infections and .‫ ﺍﻟﺨﺰﻋﺔ ﺍﻟﻜﻠﻮﻳﺔ ﻫﻲ ﺃﺳﺎﺳﻴﺔ ﻟﻠﺘﺸﺨﻴﺺ‬-D
immunization may be an inciting antigen ‫ ﻫﻮ ﺍﻟﺠﺴﻢ ﺍﻟﻤﻀﺎﺩ ﺍﻷﻛﺜﺮ ﻣﺸﺎﻫﺪﺓ ﻓﻲ ﺍﻟﺘﻌﻘﻴﺪﺍﺕ‬IgM -E
D. Renal biopsy is essential to make the .‫ﺍﻟﻤﻨﺎﻋﻴﺔ‬
diagnosis
E. IgM is the antibody class most often seen in
the immune complexes
C
Un homme de 18 ans présente un purpura palpable et une
polyarthralgie. Laquelle des propositions suivantes est
vraie dans le purpura de Henoch-Schonlein:
A. La maladie est plus fréquente chez les adultes
que chez les enfants
B. Le myocarde n'est pas affecté dans la forme
adulte
C.Les infections des voies respiratoires
supérieures et la vaccination peuvent être un
antigène incitant
D. La biopsie rénale est essentielle pour établir le
diagnostic
E. IgM est la classe d'anticorps la plus souvent
observée dans les complexes immuns

5A
13. Where does Herpes Zoster virus reside during latency? :‫ﺃﻳﻦ ﻳﻘﻴﻢ ﻓﻴﺮﻭﺱ ﺍﻟﺤﻸ ﺍﻟﻤﻨﺎﻁﻘﻲ ﺧﻼﻝ ﻓﺘﺮﺓ ﺍﻟﻜﻤﻮﻥ؟‬
A. Lateral horn of spinal cord .‫ ﺍﻟﻘﺮﻥ ﺍﻟﻮﺣﺸﻲ ﻟﻠﻨﺨﺎﻉ‬-A
B. Ventral root ganglia .‫ ﺟﺬﺭ ﺍﻟﻌﻘﺪ ﺍﻟﺒﻄﻨﻴﺔ‬-B
C. Hypothalamus .‫ ﺗﺤﺖ ﺍﻟﺴﺮﻳﺮﻱ‬-C
D. Langerhans cells .‫ ﺧﻼﻳﺎ ﻻﻧﻐﺮﻫﺎﻧﺰ‬-D
E. None of the above .‫ ﻛﻞ ﺍﻷﺟﻮﺑﺔ ﺃﻋﻼﻩ ﻏﻴﺮ ﺻﺤﻴﺤﺔ‬-E
E
Où réside le virus herpès zoster pendant la latence?
A. Corne latérale de la moelle épinière
B. Ganglions de la racine ventrale
C. Hypothalamus
D. Cellules de Langerhans
E. Aucune des réponses ci-dessus
14. Which of the following is not a concerning feature of a ‫ﺃﻱ ﻣﻦ ﺍﻟﺘﺎﻟﻲ ﻫﻮ ﻅﺎﻫﺮﺓ ﻏﻴﺮ ﺫﺍﺕ ﺃﻫﻤﻴﺔ ﻓﻲ ﺁﻓﺔ ﺟﻠﺪﻳﺔ ﻋﻨﺪ‬
mole when considering a diagnosis of malignant :‫ﺗﺸﺨﻴﺺ ﻭﺭﻡ ﻣﻴﻼﻧﻴﻨﻲ ﺧﺒﻴﺚ؟‬
melanoma? .‫ ﻋﺪﻡ ﺍﻟﺘﻨﺎﺳﻖ‬-A
A. Asymmetry .‫ ﺍﻟﻨﺰﻳﻒ‬-B
B. Bleeding .‫ ﻋﺪﻡ ﺍﻧﺘﻈﺎﻡ ﺍﻟﺤﺪﻭﺩ‬-C
C. Border irregularity .‫ ﻟﻮﻥ ﻣﺨﺘﻠﻒ ﻋﻦ ﻏﻴﺮﻩ ﻣﻦ ﺍﻵﻓﺎﺕ ﺍﻟﺠﻠﺪﻳﺔ ﻋﻨﺪ ﺍﻟﻤﺮﻳﺾ‬-D
D. Colour different from other moles on patient .‫ ﺣﻜﺔ‬-E
E. Itching
D
Lequel des éléments suivants n'est pas une caractéristique
préoccupante d'une lésion cutanée lorsqu'on considère un
diagnostic de mélanome malin?
A. Asymétrie
B. Saignement
C.Irrégularité de la bordure
D. Couleur différente des autres lésions cutanées
chez le patient
E. Démangeaisons
15. The following is a cause of membranous :‫ﺃﻱ ﻣﻦ ﺍﻟﺘﺎﻟﻲ ﻫﻮ ﺍﻟﻌﺎﻣﻞ ﺍﻟﻤﺴﺒﺐ ﻻﻟﺘﻬﺎﺏ ﻛﺒﻴﺒﺎﺕ ﺍﻟﻜﻠﻰ ﺍﻟﻐﺸﺎﺋﻲ؟‬
glomerulonephritis: .B ‫ ﺍﻟﺘﻬﺎﺏ ﺍﻟﻜﺒﺪ ﺍﻟﻔﻴﺮﻭﺳﻲ‬-A
A. Hepatitis B .‫ ﺩﺍء ﺍﻟﺴﻔﻠﺲ‬-B
B. Syphilis .C ‫ ﺍﻟﺘﻬﺎﺏ ﺍﻟﻜﺒﺪ ﺍﻟﻔﻴﺮﻭﺳﻲ‬-C
C. Hepatitis C .‫ ﺍﻟﺬﺋﺒﺔ‬-D
D. Lupus .‫ ﻛﻞ ﺍﻷﺟﻮﺑﺔ ﺃﻋﻼﻩ ﺻﺤﻴﺤﺔ‬-E
E. All of the above
E
Ce qui suit est une cause de glomérulonéphrite
membraneuse:
A. Hépatite B
B. Syphilis
C. Hépatite C
D. Lupus
E. Tout ce qui précède

6A
16. The following extra-articular feature is associated with ‫ﻣﻔﺼﻠﻴﺔ ﺗﺘﺮﺍﻓﻖ ﻣﻊ ﺍﻟﺘﻬﺎﺏ ﺍﻟﻔﻘﺎﺭ‬-‫ﺃﻱ ﻣﻦ ﺍﻟﺘﺎﻟﻲ ﻫﻲ ﻅﺎﻫﺮﺓ ﺧﺎﺭﺝ‬
Ankylosing Spondylitis: :‫ﺍﻟﻤﻘﺴﻂ؟‬
A. Apical lung fibrosis .‫ ﺗﻠﻴﻒ ﺭﺋﻮﻱ ﻗﻤﻲ‬-A
B. Achilles tendinitis .‫ ﺍﻟﺘﻬﺎﺏ ﻭﺗﺮ ﺍﻟﻌﺮﻗﻮﺏ‬-B
C. Anterior uveitis .‫ ﺍﻟﺘﻬﺎﺏ ﺍﻟﻌﻨﺒﻴﺔ ﺍﻷﻣﺎﻣﻲ‬-C
D. Aortic regurgitation .‫ ﻗﻠﺲ ﺍﻷﺑﻬﺮﻱ‬-D
E. All of the above .‫ ﻛﻞ ﺍﻷﺟﻮﺑﺔ ﺃﻋﻼﻩ ﺻﺤﻴﺤﺔ‬-E
E
La caractéristique extra-articulaire suivante est associée à
la spondylarthrite ankylosante:
A. Fibrose pulmonaire apicale
B. Tendinite d'Achille
C. Uvéite antérieure
D. Régurgitation aortique
E. Tout ce qui précède
17. Atrial fibrillation can be caused by all the following :‫ﻛﻞ ﻣﺎ ﻳﻠﻲ ﻳﻤﻜﻦ ﺃﻥ ﻳﺴﺒﺐ ﺍﻟﺮﺟﻔﺎﻥ ﺍﻷﺫﻳﻨﻲ ﻣﺎ ﻋﺪﺍ‬
except: .‫ ﺇﻧﺘﺎﻥ ﺣﺎﺩ‬-A
A. Acute infection .‫ ﺍﻹﻓﺮﺍﻁ ﻓﻲ ﺷﺮﺏ ﺍﻟﻜﺤﻮﻝ‬-B
B. Binge drinking .‫ ﺍﺭﺗﻔﺎﻉ ﺍﻟﻀﻐﻂ‬-C
C. Hypertension .‫ ﺃﻡ ﺍﻟﺪﻡ ﺍﻷﺑﻬﺮﻳﺔ ﺍﻟﺒﻄﻨﻴﺔ‬-D
D. Abdominal aortic aneurysm
A
La fibrillation auriculaire peut être causée par par tout ce
qui suit sauf ::
A. Infection aiguë
B. Exces d’alcool
C. Hypertension
D. Anévrisme de l'aorte abdominale
18. Mobitz type I second-degree atrio-ventricular block is :‫ ﻳﺸﺎﻫﺪ‬،‫ ﻣﻮﺑﻴﺘﺰ‬I ‫ﺍﻟﺤﺠﺐ ﺍﻷﺫﻳﻨﻲ ﺍﻟﺒﻄﻴﻨﻲ ﻣﻦ ﺍﻟﺪﺭﺟﺔ ﺍﻟﺜﺎﻧﻴﺔ ﻧﻤﻂ‬
seen: .‫ ﻛﻌﻴﺐ ﻋﺎﺑﺮ ﻳﺘﺮﺍﻓﻖ ﻣﻊ ﺍﺣﺘﺸﺎء ﺍﻟﺠﺪﺍﺭ ﺍﻟﺴﻔﻠﻲ‬-A
A. As a transient abnormality with inferior wall .‫ ﻣﻊ ﺍﻟﺘﺴﻤﻢ ﺍﻟﺪﻭﺍﺋﻲ ﻟﺤﺎﺟﺒﺎﺕ ﺑﻴﺘﺎ‬-B
infarction .‫ ﻣﻊ ﺍﻟﺘﺴﻤﻢ ﺑﺎﻟﺪﻳﺠﻴﺘﺎﻝ‬-C
B. With drug intoxication such as beta-blockers .‫ ﻋﻨﺪ ﺍﻷﺷﺨﺎﺹ ﺍﻟﺴﻠﻴﻤﺔ ﺍﻟﺬﻳﻦ ﻟﺪﻳﻬﻢ ﺗﻮﺗﺮ ﻣﺒﻬﻤﻲ ﻣﺘﺰﺍﻳﺪ‬-D
C. With digitalis intoxication .‫ ﻛﻞ ﺍﻷﺟﻮﺑﺔ ﺃﻋﻼﻩ ﺻﺤﻴﺤﺔ‬-E
D. In normal persons with heightened vagal tone
E. All of the above

Bloc auriculo-ventriculaire de deuxième degré de Mobitz E


de type I survient:
A. Comme une anomalie transitoire avec infarctus de
la paroi inférieure
B. Avec l'intoxication médicamenteuse comme les
bêta-bloquants
C. Avec l'intoxication digitalique
D. Chez les personnes normales avec un tonus vagal
accru
E. Tout ce qui précède

7A
19. The following is a cause of hyperamylasuria: :‫ﺃﻱ ﻣﻦ ﺍﻟﺘﺎﻟﻲ ﻫﻮ ﻋﺎﻣﻞ ﻣﺴﺒﺐ ﻟﻔﺮﻁ ﺃﻣﻴﻼﺯ ﺍﻟﺒﻮﻝ؟‬
A. Morphine .‫ ﻣﻮﺭﻓﻴﻦ‬-A
B. Intestinal infarction .‫ ﺍﺣﺘﺸﺎء ﻣﻌﻮﻱ‬-B
C. Esophageal carcinoma .‫ ﻛﺎﺭﺳﻴﻨﻮﻣﺎ ﺍﻟﻤﺮﻳﺊ‬-C
D. Pancreatic necrosis .‫ ﺗﻨﺨﺮ ﺑﻨﻜﺮﻳﺎﺳﻲ‬-D
E. All of the above .‫ ﻛﻞ ﺍﻷﺟﻮﺑﺔ ﺃﻋﻼﻩ ﺻﺤﻴﺤﺔ‬-E
E
Ce qui suit est une cause d'hyperamylasurie:
A. Morphine
B. Infarctus intestinal
C. Carcinome œsophagien
D. Nécrose pancréatique
E. Tout ce qui précède
20. A 54-year-old woman undergoes a routine insurance .‫ ﻋﺎﻣﺎ ﺗﺨﻀﻊ ﻟﻔﺤﺺ ﺳﺮﻳﺮﻱ ﺭﻭﺗﻴﻨﻲ‬54 ‫ﺍﻣﺮﺃﺓ ﺗﺒﻠﻎ ﻣﻦ ﺍﻟﻌﻤﺮ‬
physical examination. Chest x-ray reveals bilateral hilar ‫ ﺗﺒﻴﻦ ﻣﻦ‬.‫ﺃﻅﻬﺮﺕ ﺍﻷﺷﻌﺔ ﺍﻟﺴﻴﻨﻴﺔ ﻟﻠﺼﺪﺭﻛﺘﻞ ﻧﻘﻴﺮﻳﺔ ﺛﻨﺎﺋﻴﺔ ﺍﻟﺠﺎﻧﺐ‬
masses. Biopsy of the masses shows granulomata, but ‫ ﺃﻣﺎ ﻟﻄﺎﺧﺔ ﺍﻟﻜﺸﻒ ﻋﻦ‬،‫ﺍﻟﺨﺰﻋﺔ ﻟﺘﻠﻚ ﺍﻟﻜﺘﻞ ﻭﺟﻮﺩ ﻭﺭﻡ ﺣﺒﻴﺒﻲ‬
acid-fast and fungal stains are negative for organisms. ‫ ﺃﻱ ﻣﻦ‬.‫ﺍﻟﻔﻄﻮﺭ ﻭﺍﻟﺠﺮﺍﺛﻴﻢ ﺍﻟﻤﻘﺎﻭﻣﺔ ﻟﻠﺤﻤﺾ ﻭﺍﻟﻐﻮﻝ ﻓﻬﻲ ﺳﻠﺒﻴﺔ‬
:‫ﺍﻟﺘﺎﻟﻲ ﻫﻮ ﺍﻟﺘﺸﺨﻴﺺ ﺍﻷﻛﺜﺮ ﺗﺮﺟﻴﺤﺎ؟‬
Which of the following is the most likely diagnosis?
.‫ ﺩﺍء ﻛﺎﺭﻭﻟﻲ‬-A
A. Caroli disease
.‫ ﺩﺍء ﺭﻳﻨﻮﺩ‬-B
B. Raynaud disease .‫ ﺳﺎﺭﻛﻮﻳﺪ‬-C
C. Sarcoidosis .‫ ﺗﺼﻠﺐ ﺍﻟﺠﻠﺪ‬-D
D. Scleroderma .‫ ﺫﺋﺒﺔ ﺣﻤﺎﻣﻴﺔ ﺟﻬﺎﺯﻳﺔ‬-E
E. Systemic lupus erythematosus
C
Une femme de 54 ans subit un examen médical
d’assurance de routine. La radiographie pulmonaire
révèle des masses hilaires bilatérales. La biopsie des
masses montre des granulomes, mais les colorations
acides et pour les champignons sont négatives pour ces
organismes. Lequel des diagnostics suivants est le plus
probable?
A. Maladie De Caroli
B. Maladie de Raynaud
C. Sarcoïdose
D. Sclérodermie
E. Lupus érythémateux systémique

8A
21. A 74-year-old man has had increasingly severe headaches ‫ ﻋﺎﻣﺎ ﻳﻌﺎﻧﻲ ﻣﻦ ﺻﺪﺍﻉ ﺷﺪﻳﺪ ﻣﺘﻔﺎﻗﻢ ﻣﻨﺬ‬74 ‫ﺭﺟﻞ ﻳﺒﻠﻎ ﻣﻦ ﺍﻟﻌﻤﺮ‬
for 2 months, centered on the right. On examination, he ‫ ﻛﺎﻧﺖ ﺣﺮﺍﺭﺗﻪ‬،‫ ﻋﻨﺪ ﺍﻟﻔﺤﺺ‬.‫ ﻣﺘﻤﺮﻛﺰ ﻋﻠﻰ ﺟﻬﺔ ﺍﻟﻴﻤﻴﻦ‬،‫ﺷﻬﺮﻳﻦ‬
has T 36.9°C, P 82/minute, RR 15/minute, and BP 130/85 ‫ ﻭﺿﻐﻂ ﺍﻟﺪﻡ‬،‫ﺩ‬/15 ‫ ﺳﺮﻋﺔ ﺍﻟﺘﻨﻔﺲ‬،‫ﺩ‬/82 ‫ ﺍﻟﻨﺒﺾ‬،‫ﻡ‬°36.9
mm Hg. There is a palpable tender cord-like area over his ‫ ﻫﻨﺎﻙ ﻣﺴﺎﺣﺔ ﻣﺆﻟﻤﺔ ﻣﺠﺴﻮﺳﺔ ﺗﺸﺒﻪ ﺍﻟﺤﺒﻞ ﻓﻮﻕ‬.‫ ﻣﻢ ﺯﺋﺒﻖ‬85/130
right temple. His heart rate is regular with no murmurs, ‫ ﺃﻭ‬،‫ ﺧﺒﺐ‬،‫ ﺳﺮﻋﺔ ﺍﻟﻘﻠﺐ ﻁﺒﻴﻌﻴﺔﺑﺪﻭﻥ ﻧﻔﺨﺔ‬.‫ﺍﻟﺼﺪﻍ ﺍﻷﻳﻤﻦ‬
gallops, or rubs. Pulses are equal and full in all ‫ﺃﻱ ﻣﻦ‬.‫ ﺍﻟﻨﺒﻀﺎﺕ ﻣﺘﺴﺎﻭﻳﺔ ﻭﻛﺎﻣﻠﺔ ﻓﻲ ﺟﻤﻴﻊ ﺍﻷﻁﺮﺍﻑ‬.‫ﺍﺣﺘﻜﺎﻛﺎﺕ‬
extremities. Which of the following laboratory test ‫ﻧﺘﺎﺋﺞ ﺍﻟﻔﺤﻮﺹ ﺍﻟﻤﺨﺒﺮﻳﺔ ﺍﻟﺘﺎﻟﻴﺔﻫﻲ ﺍﻷﻛﺜﺮ ﺗﺮﺟﻴﺤﺎ ﺃﻥ ﺗﻈﻬﺮ ﻣﻊ‬
findings is most likely to be present with this man's :‫ﻣﺮﺽ ﻫﺬﺍ ﺍﻟﺮﺟﻞ؟‬
disease? .‫ﺳﺎﻋﺔ‬/‫ ﻣﻢ‬110 ‫ ﺳﺮﻋﺔ ﺗﺮﺳﺐ ﺍﻟﻜﺮﻳﺎﺕ ﺍﻟﺤﻤﺮﺍء‬-A
A. Erythrocyte sedimentation rate of 110 mm/hr .‫ ﻣﻞ‬/‫ ﻭﺣﺪﺓ ﺩﻭﻟﻴﺔ‬80 ‫ ﺍﻟﻌﺎﻣﻞ ﺍﻟﺮﺛﻴﻮﻱ‬-B
B. Rheumatoid factor titer of 80 IU/mL .‫ ﺩﺳﻞ‬/‫ ﻣﻠﻎ‬15 HDL -C
C. HDL cholesterol of 15 mg/dL 1024 :1 DNA‫ ﺍﻟﻄﺎﻕ ﻏﻴﺮ ﺍﻟﻤﺰﺩﻭﺝ ﻝ‬-D
D. Anti-double stranded DNA titer of 1:1024 Panca 160 :1
pANCA titer of 1:160

Un homme de 74 ans a depuis 2 mois des maux de tête de A


plus en plus graves, centrés à droite. À l'examen, il a une
température de 36,9 ° C, une tension artérielle de 130/85
mmHg un pouls à 82/minutes et une fréquence
respiratoire de 15/minutes. Il y a une zone palpable
comme une corde sur sa tempe droite. Son rythme
cardiaque est régulier, sans murmures, galops ou
frottements. Les pouls sont égaux et normaux à toutes les
extrémités. Parmi les tests de laboratoire suivants, lequel
est le plus susceptible d'être présent avec la maladie de cet
homme?
A. Vitesse de sédimentation de 110 mm / h
B. Titre du facteur rhumatoïde de 80 UI / mL
C. cholestérol HDL de 15 mg / dL
D. Titre en ADN anti-double de 1: 1024 pANCA titre
de 1: 160
22. A 45 yo man, heavy smoker, is recently diagnosed with ‫ ﺗﻢ ﺍﻟﺘﺸﺨﻴﺺ ﻟﻪ‬،‫ ﻣﺪﺧﻦ ﺷﺮﻩ‬،‫ ﻋﺎﻣﺎ‬45 ‫ﺭﺟﻞ ﻳﺒﻠﻎ ﻣﻦ ﺍﻟﻌﻤﺮ‬
chronic obstructive pulmonary disease (COPD). He has ‫ ﻟﻢ ﻳﺴﺠﻞ ﺗﻔﺎﻗﻢ ﺣﺎﺩ ﻓﻲ‬.‫ﻣﺆﺧﺮﺍ ﺑﻤﺮﺽ ﺍﻻﻧﺴﺪﺍﺩ ﺍﻟﺮﺋﻮﻱ ﺍﻟﻤﺰﻣﻦ‬
no documented acute exacerbation in the past. Which of ‫ ﺃﻱ ﻣﻦ ﺍﻟﻤﻌﺎﻟﺠﺎﺕ ﺍﻟﺘﺎﻟﻴﺔ ﻫﻲ ﻏﻴﺮ ﻣﻨﺎﺳﺒﺔ ﻓﻲ ﺍﻟﺘﺪﺑﻴﺮ‬.‫ﺍﻟﻤﺎﺿﻲ‬
the following treatments is not suitable in the :‫( ﻋﻨﺪ ﻫﺬﺍ ﺍﻟﻤﺮﻳﺾ؟‬COPD)‫ﺍﻟﻌﻼﺟﻲ ﻝ‬
management of COPD in this patient? .‫ ﻟﻘﺎﺡ ﺍﻻﻧﻔﻠﻮﻧﺰﺍ ﻭﺍﻟﻤﻜﻮﺭﺍﺕ ﺍﻟﺮﺋﻮﻳﺔ ﺳﻨﻮﻳﺎ‬-A
A. Annual influenza and pneumococcal vaccination .‫ ﻛﻮﺭﺗﻴﻜﻮﺳﺘﻴﺮﻭﺋﻴﺪ ﺍﺳﺘﻨﺸﺎﻗﻲ‬-B
B. Inhaled corticosteroids .‫ ﻗﺼﻴﺮ ﺍﻟﻤﺪﻯ‬2‫ ﻣﻘﻠﺪ ﺑﻴﺘﺎ‬-C
C. Short acting ß2-agonist .‫ ﻣﻀﺎﺩ ﻛﻮﻟﻴﻦ ﻗﺼﻴﺮ ﺍﻟﻤﺪﻯ‬-D
D. Short –acting anti-cholinergic .‫ ﺇﻳﻘﺎﻑ ﺍﻟﺘﺪﺧﻴﻦ‬-E
E. Smoking cessation
B
Un homme de 45 ans, grand fumeur, vient d’avoir un
diagnostic de maladie pulmonaire obstructive chronique
(BPCO). Il n'a pas d'exacerbation aiguë dans le passé.
Lequel des traitements suivants ne convient pas à la
gestion de la BPCO chez ce patient?
A. Vaccination annuelle contre la grippe et le
pneumocoque
B. corticostéroïdes inhalés
C. Agoniste ß2 à courte durée d'action
D. Anticholinergique à action rapide
E. Arrêt du tabac

9A
23. A 19 yo footballer has collapsed during a game. His ‫ ﺍﻟﺴﺒﻞ ﺍﻟﻬﻮﺍﺋﻴﺔ‬.‫ ﻋﺎﻣﺎ ﺍﻧﻬﺎﺭ ﺧﻼﻝ ﺍﻟﻤﺒﺎﺭﺍﺓ‬19 ‫ﻻﻋﺐ ﻛﺮﺓ ﻗﺪﻡ ﻋﻤﺮﻩ‬
airway is clear and he is brought to the emergency room, ‫ ﺣﻴﺚ ﺑﺪﺃ ﺑﺎﺳﺘﺮﺩﺍﺩ ﻗﻮﺗﻪ‬،‫ﻭﺍﺿﺤﺔ ﻭﻗﺪ ﺃﺣﻀﺮ ﺇﻟﻰ ﻗﺴﻢ ﺍﻟﻄﻮﺍﺭﺉ‬
where he begins to recover and denies that he has chest ‫ ﺃﻱ ﻣﻦ‬.‫ ﻟﻢ ﻳﺼﺐ ﺑﺬﻟﻚ ﺳﺎﺑﻘﺎ‬.‫ﻭﻳﻨﻔﻲ ﺃﻥ ﻳﻜﻮﻥ ﻟﺪﻳﻪ ﺃﻟﻢ ﻓﻲ ﺍﻟﺼﺪﺭ‬
pain. He has never had anything like this before. Which of :‫ﺍﻟﺘﺎﻟﻲ ﻫﻮ ﺍﻟﺘﺸﺨﻴﺺ ﺍﻷﻛﺜﺮ ﺗﺮﺟﻴﺤﺎ؟‬
the following is the most likely diagnosis? .‫ ﺗﻀﻴﻖ ﺍﻟﺴﺒﺎﺗﻲ‬-A
A. Carotid stenosis .‫ ﺇﻋﺘﻼﻝ ﻋﻀﻠﺔ ﻗﻠﺐ ﺿﺨﺎﻣﻲ ﺍﻧﺴﺪﺍﺩﻱ‬-B
B. Hypertrophic obstructive cardiomyopathy .‫ ﺍﺣﺘﺸﺎء ﻋﻀﻠﺔ ﺍﻟﻘﻠﺐ‬-C
C. Myocardial infarction .‫ ﺣﻤﻰ ﺭﺛﻴﻮﻳﺔ‬-D
D. Rheumatic fever .‫ ﺍﻧﺴﻤﺎﻡ ﺩﺭﻗﻲ‬-E
E. Thyrotoxicosis
B
Un footballeur de 19 ans s'est effondré lors d'un match.
Ses voies respiratoires sont libres et il est conduit à
l'urgence où il commence à se rétablir et nie avoir mal à la
poitrine. Il n'a jamais eu de tel incident auparavant.
Lequel des diagnostics suivants est le plus probable?
A. Sténose carotidienne
B. Cardiomyopathie obstructive hypertrophique
C. Infarctus du myocarde
D. Rhumatisme articulaire aigu
E. Thyrotoxicose
24. You review an electrocardiogram in the emergency room. .‫ﻋﻨﺪ ﻣﺮﺍﺟﻌﺔ ﺗﺨﻄﻴﻂ ﺍﻟﻘﻠﺐ ﺍﻟﻜﻬﺮﺑﺎﺋﻲ ﻓﻲ ﻏﺮﻓﺔ ﺍﻟﻄﻮﺍﺭﺉ‬
Details are printed as follows: rate 88/min, regular - ‫ ﺍﻟﻤﺤﻮﺭ‬،‫ ﺍﻟﻨﻈﻢ ﻁﺒﻴﻌﻲ‬،‫ﺩ‬/88 ‫ ﺍﻟﻤﻌﺪﻝ‬:‫ﺍﻟﺘﻔﺎﺻﻴﻞ ﻣﺴﺠﻠﺔ ﻛﺎﻟﺘﺎﻟﻲ‬
rhythm, axis -20°, PR duration 0.26 seconds, QRS complex QT ‫ ﻓﺘﺮﺓ‬،‫ ﺛﺎﻧﻴﺔ‬0.08 QRS ‫ ﻣﻌﻘﺪ‬،‫ ﺛﺎﻧﻴﺔ‬0.26 PR ‫ ﻓﺘﺮﺓ‬،°20
0.08 seconds, QT interval 0.2 seconds. You note that p ‫ ﻣﻮﺟﻮﺩﺓ ﻓﻘﻂ ﺑﻌﺪ ﻛﻞ‬p ‫ ﻳﻤﻜﻦ ﻣﻼﺣﻈﺔ ﺃﻥ ﻣﻮﺟﺎﺕ‬.‫ ﺛﺎﻧﻴﺔ‬0.2
waves are only present before each QRS and that the :‫ ﺃﻱ ﻣﻦ ﺍﻟﺘﺎﻟﻲ ﻳﻌﺘﺒﺮ ﺃﻓﻀﻞ ﺧﻼﺻﺔ؟‬.‫ ﻭﺃﻥ ﺍﻟﻨﻈﻢ ﻁﺒﻴﻌﻲ‬QRS
rhythm is regular. Which of the following would be the .‫ ﺣﺠﺐ ﻗﻠﺒﻲ ﻣﻦ ﺍﻟﺪﺭﺟﺔ ﺍﻷﻭﻟﻰ‬-A
best summary? .‫ ﺍﻧﺤﺮﺍﻑ ﺍﻟﻤﺤﻮﺭ ﺍﻷﻳﺴﺮ‬-B
A. First degree heart block .‫ ﺣﺠﺐ ﻓﺮﻉ ﺍﻟﺤﺰﻣﺔ ﺍﻷﻳﺴﺮ‬-C
B. Left axis deviation .‫ ﺗﺴﺎﺭﻉ ﻗﻠﺐ ﺑﻄﻴﻨﻲ‬-D
C. Left bundle branch block .WPW ‫ ﻣﺘﻼﺯﻣﺔ‬-E
D. Ventricular tachycardia
E. WPW syndrome (Wolf Parkinson white)
A
Vous passez en revue un électrocardiogramme dans la
salle d'urgence. Les détails sont imprimés comme suit:
fréquence 88 / min, rythme régulier, axe -20 °, durée PR
de 0,26 seconde, complexe QRS de 0,08 seconde, intervalle
QT de 0,2 seconde. Vous notez que les ondes p ne sont
présentes qu'avant chaque QRS et que le rythme est
régulier. Lequel des suivants serait le meilleur résumé?
A. Bloc cardiaque du premier degré
B. Déviation de l'axe gauche
C. Bloc de branche gauche
D. Tachycardie ventriculaire
E. Syndrome WPW (Wolf Parkinson white)

10 A
25. A 65 yo man with a longstanding diagnosis of chronic ‫ ﻳﻌﺎﻧﻲ ﻣﻨﺬ ﺃﻣﺪ ﺑﻌﻴﺪ ﻣﻦ ﻣﺮﺽ ﺍﻻﻧﺴﺪﺍﺩ‬،‫ ﻋﺎﻣﺎ‬65 ‫ﺭﺟﻞ ﻋﻤﺮﻩ‬
obstructive pulmonary disease has deteriorating liver ‫ ﻟﻘﺪ ﺗﺪﻫﻮﺭﺕ ﻭﻅﺎﺋﻔﻪ ﺍﻟﻜﺒﺪﻳﺔ ﻭﺗﺸﻴﺮ ﺍﻟﻔﺤﻮﺹ‬.‫ﺍﻟﺮﺋﻮﻱ ﺍﻟﻤﺰﻣﻦ‬
function tests and clinical signs and symptoms of ‫ﻣﺎ ﻫﻮ ﺍﻟﺬﻱ ﻳﻨﺒﻐﻲ ﺍﻟﺘﺤﺮﻱ‬.‫ﺍﻟﺴﺮﻳﺮﻳﺔ ﻭﺍﻷﻋﺮﺍﺽ ﺇﻟﻰ ﺗﺸﻤﻊ ﺍﻟﻜﺒﺪ‬
cirrhosis. What investigation should be requested? :‫ﻋﻨﻪ؟‬
A. Alpha-1-antitrypsin serum levels .‫ ﺍﻟﻤﺼﻠﻴﺔ‬١-‫ ﻣﺴﺘﻮﻳﺎﺕ ﻣﻀﺎﺩ ﺗﺮﻳﺒﺴﻴﻦ ﺃﻟﻔﺎ‬-A
B. Alpha-feto protein level .‫ﻓﻴﺘﻮ ﺑﺮﻭﺗﻴﻦ‬-‫ ﻣﺴﺘﻮﻯ ﺃﻟﻔﺎ‬-B
C. Anti-smooth muscle antibodies .‫ ﺍﻷﺿﺪﺍﺩ ﺍﻟﻤﻀﺎﺩﺓ ﻟﻠﻌﻀﻼﺕ ﺍﻟﻤﻠﺴﺎء‬-C
D. Gamma GT levels .GT ‫ ﻣﺴﺘﻮﻳﺎﺕ ﻏﺎﻣﺎ‬-D
E. Hepatitis screen
.‫ ﺍﻟﺘﻬﺎﺏ ﺍﻟﻜﺒﺪ‬-E
A
Un homme de 65 ans ayant un diagnostic de longue date
de maladie pulmonaire obstructive chronique présente
une détérioration des tests de la fonction hépatique ainsi
que des signes cliniques et des symptômes de cirrhose.
Quelle enquête devrait être demandée?
A. Taux sériques de l'alpha-1-antitrypsine
B. Taux de protéine alpha-feto
C. Anticorps anti-muscle lisse
D. Taux Gamma GT
E. Dépistage de l'hépatite
26. A patient with nephrotic syndrome and normal creatinine ،‫ﻣﺮﻳﺾ ﻳﻌﺎﻧﻲ ﻣﻦ ﻣﺘﻼﺯﻣﺔ ﻧﻔﺮﻭﻧﻴﺔ ﻭﺗﺼﻔﻴﺔ ﺍﻟﻜﺮﻳﺎﺗﻨﻴﻦ ﻁﺒﻴﻌﻴﺔ‬
clearance, presents with sudden onset of left flank pain, .‫ ﻭﺑﻴﻠﺔ ﺩﻣﻮﻳﺔ‬،‫ﻳﺸﻜﻮ ﻣﻦ ﺑﺪء ﻣﻔﺎﺟﺊ ﻷﻟﻢ ﻓﻲ ﺍﻟﺨﺎﺻﺮﺓ ﺍﻟﻴﺴﺮﻯ‬
and hematuria. Laboratory tests show a rapid .‫ﺃﻅﻬﺮ ﺍﻟﻔﺤﺺ ﺍﻟﻤﺨﺒﺮﻱ ﺗﺪﻫﻮﺭ ﺳﺮﻳﻊ ﻓﻲ ﺍﻟﻮﻅﻴﻔﺔ ﺍﻟﻜﻠﻮﻳﺔ‬
deterioration of renal function. Ultrasound demonstrates ‫ﺃﻭﺿﺤﺖ ﺍﻟﻤﻮﺟﺎﺕ ﻓﻮﻕ ﺍﻟﺼﻮﺗﻴﺔ ﺃﻥ ﺍﻟﻜﻠﻴﺔ ﺍﻟﻴﺴﺮﻯ ﺃﻛﺒﺮ ﺣﺠﻤﺎ ﻣﻦ‬
the left kidney to be larger than the right. Which of the :‫ ﺃﻱ ﻣﻦ ﺍﻟﺘﺎﻟﻲ ﻫﻮ ﺍﻟﺘﺸﺨﻴﺺ ﺍﻷﻛﺜﺮ ﺗﺮﺟﻴﺤﺎ؟‬.‫ﺍﻟﻜﻠﻴﺔ ﺍﻟﻴﻤﻨﻰ‬
following is the most likely diagnosis? .‫ ﺍﻟﺘﻬﺎﺏ ﺍﻟﺤﻮﻳﻀﺔ ﻭﺍﻟﻜﻠﻴﺔ ﺍﻟﺤﺎﺩ‬-A
A. Acute pyelonephritis .‫ ﺍﻧﺴﺪﺍﺩ ﺍﻟﺸﺮﻳﺎﻥ ﺍﻟﻜﻠﻮﻱ‬-B
B. Renal artery occlusion .‫ ﺧﺜﺎﺭ ﻭﺭﻳﺪﻱ ﻛﻠﻮﻱ‬-C
C. Renal vein thrombosis .‫ ﺗﺤﺼﻲ ﺍﻟﻜﻠﻴﺔ‬-D
D. Nephrolithiasis .‫ ﺗﻤﺰﻕ ﻛﻴﺴﺔ ﻛﻠﻮﻳﺔ‬-E
E. Ruptured renal cyst
C
Un patient présentant un syndrome néphrotique et une
clairance de la créatinine normale, présente une douleur
soudaine au flanc gauche et une hématurie. Les tests de
laboratoire montrent une détérioration rapide de la
fonction rénale. L'échographie montre que le rein gauche
est plus gros que le droit. Lequel des diagnostics suivants
est le plus probable?
A. Pyélonéphrite aiguë
B. Occlusion de l'artère rénale
C. Thrombose veineuse rénale
D. Néphrolithiase
E. kyste rénal rompu

11 A
27. A 24-year-old woman with newly diagnosed systemic ‫ ﻋﺎﻣﺎ ﺗﻢ ﺗﺸﺨﻴﺼﻬﺎ ﺣﺪﻳﺜﺎ ﺑﺎﻟﺬﺋﺒﺔ ﺍﻟﺤﻤﺎﻣﻴﺔ‬24 ‫ﺍﻣﺮﺃﺓ ﻋﻤﺮﻫﺎ‬
lupus erythematosus (SLE) and lupus nephritis is ‫ ﻭﻣﻦ ﺍﻟﻤﻘﺮﺭ ﺃﻥ ﺗﺒﺪﺃ ﺍﻟﻌﻼﺝ‬،‫ﺍﻟﺠﻬﺎﺯﻳﺔ ﻭﺍﻟﺘﻬﺎﺏ ﺍﻟﻜﻠﻴﺔ ﺍﻟﺬﺋﺒﻲ‬
scheduled to begin receiving cyclophosphamide therapy. ‫ ﻛﻴﻒ ﻳﻨﺒﻐﻲ ﺍﻟﺘﺤﺮﻱ ﺑﺸﻜﻞ ﺃﻓﻀﻞ ﻋﻦ ﺳﺮﻁﺎﻥ‬.‫ﺑﺎﻟﺴﻴﻜﻠﻮﻓﻮﺳﻔﺎﻣﻴﺪ‬
How should she be best screened for bladder cancer in the :‫ﺍﻟﻤﺜﺎﻧﺔ ﻓﻲ ﺍﻟﻤﺴﺘﻘﺒﻞ؟‬
future? .‫ ﺍﻟﺘﺤﺮﻱ ﻋﻦ ﺑﻴﻠﺔ ﺩﻣﻮﻳﺔ ﺑﻌﺪ ﺳﻦ ﺍﻟﺨﻤﺴﻴﻦ‬-A
A. Hematuria screening only after age 50 .‫ ﺑﺎﻟﺘﺤﻠﻴﻞ ﺍﻟﻐﻤﻴﺴﻲ ﻟﻠﺒﻴﻠﺔ ﺍﻟﺪﻣﻮﻳﺔ‬-B
B. With dipstick analysis for hematuria .‫ ﺑﺎﻟﺘﺤﻠﻴﻞ ﺍﻟﻤﺠﻬﺮﻱ ﻟﻠﺒﻴﻠﺔ ﺍﻟﺪﻣﻮﻳﺔ‬-C
C. With microscopic analysis for hematuria .‫ ﺑﻤﻮﺟﺎﺕ ﻓﻮﻕ ﺍﻟﺼﻮﺕ ﻟﻠﻤﺜﺎﻧﺔ‬-D
D. With bladder ultrasound .‫ ﺗﻨﻈﻴﺮ ﺍﻟﻤﺜﺎﻧﺔ ﻭﺍﻟﺪﺭﺍﺳﺔ ﺍﻟﺨﻠﻮﻳﺔ ﺍﻟﺒﻮﻟﻴﺔ‬-E
E. Urine cytology and cystoscopy
E
Une femme de 24 ans souffrant de lupus érythémateux
systémique (SLE) et de néphrite lupique récemment
diagnostiqués doit commencer un traitement par
cyclophosphamide. Comment devrait-elle être mieux
dépistée pour le cancer de la vessie à l'avenir?
A. Dépistage de l’hématurie seulement après 50 ans
B. Analyse par bandelette pour hématurie
C. Analyse microscopique de l'hématurie
D. Echographie de la vessie
E. Cytologie urinaire et cystoscopie urinaires
28. A 22-year-old female with a history of generalized tonic ‫ ﻋﺎﻣﺎ ﺗﻌﺎﻧﻲ ﻣﻦ ﺍﺧﺘﻼﺟﺎﺕ ﻣﻘﻮﻳﺔ ﺧﻠﺠﺎﻧﻴﺔ‬22 ‫ﺍﻣﺮﺃﺓ ﺗﺒﻠﻎ ﻣﻦ ﺍﻟﻌﻤﺮ‬
and clonic convulsions has been receiving treatment for ‫ ﻫﻲ ﺍﻵﻥ‬.‫ﻣﻌﻤﻤﺔ ﻭﻫﻲ ﺗﺘﻠﻘﻰ ﻋﻼﺟﺎ ﻟﻠﺼﺮﻉ ﻣﻨﺬ ﺳﻨﺔ ﻭﻧﺼﻒ‬
epilepsy for the last 1 and a half years. She has now :‫ ﻣﺎ ﻫﻮ ﺍﻟﺪﻭﺍء ﺍﻟﺬﻱ ﺗﺘﻨﺎﻭﻟﻪ؟‬.‫ﺗﻌﺎﻧﻲ ﻣﻦ ﻓﺮﻁ ﺗﻨﺴﺞ ﻟﺜﻮﻱ ﻭﺷﻌﺮﺍﻧﻴﺔ‬
developed gum hypertrophy and facial hirsutism. What .‫ ﺇﻳﺜﻮﺳﻮﻛﺴﻴﻤﻴﺪ‬-A
drug is she most likely taking? .‫ ﻓﻴﻨﻮﺑﺎﺭﺑﻴﺘﺎﻝ‬-B
A. Ethosuximide .‫ ﻓﻨﻴﺘﻮﺋﻴﻦ‬-C
B. Phenobarbital .‫ ﻻﻣﻮﺗﺮﻳﺠﻴﻦ‬-D
C. Phenytoin .‫ ﻓﻴﻐﺎﺑﺎﺗﺮﻳﻦ‬-E
D. Lamotrigine
E. Vigabatrin
C
Une femme de 22 ans ayant des antécédents de
convulsions toniques et cloniques généralisées suit un
traitement contre l'épilepsie depuis un an et demi. Elle a
maintenant développé une hypertrophie gingivale et un
hirsutisme facial. Quel médicament prend-elle le plus
probablement?
A. Ethosuximide
B. Phénobarbital
C. Phénytoïne
D. Lamotrigine
E. Vigabatrin

12 A
29. A 30-year-old woman presents with small wounds on the ‫ ﻟﺪﻳﻬﺎ ﺷﻘﻮﻕ ﻋﻠﻰ ﺟﺎﻧﺒﻲ ﻓﻤﻬﺎ ﻣﻨﺬ‬،‫ ﻋﺎﻣﺎ‬30 ‫ﺍﻣﺮﺃﺓ ﺗﺒﻠﻎ ﻣﻦ ﺍﻟﻌﻤﺮ‬
sides of her mouth for the past 2 weeks. She recently ،‫ ﻋﻨﺪ ﺍﻟﻔﺤﺺ‬.‫ ﻟﻘﺪ ﺍﺗﺒﻌﺖ ﻣﺆﺧﺮﺍ ﻧﻈﺎﻡ ﺣﻤﻴﺔ ﻧﺒﺎﺗﻲ ﺻﺎﺭﻡ‬.‫ﺃﺳﺒﻮﻋﻴﻦ‬
adopted a strict vegetarian diet. On examination, you note ‫ ﻭﺷﻔﺎﻩ ﻣﺘﺂﻛﻠﺔ ﺑﺎﻹﺿﺎﻓﺔ‬،‫ ﻟﺴﺎﻥ ﺃﺭﺟﻮﺍﻧﻲ‬،‫ﻟﻮﺣﻆ ﺷﺤﻮﺏ ﺍﻟﻤﻠﺘﺤﻤﺔ‬
she has pale conjunctivae, a magenta tongue, and :‫ ﻣﺎ ﻫﻮ ﺍﻟﺘﺸﺨﻴﺺ ﺍﻷﻛﺜﺮ ﺗﺮﺟﻴﺤﺎ؟‬.‫ﺇﻟﻰ ﺍﻟﺘﻬﺎﺏ ﺍﻟﻔﻢ ﺍﻟﺰﺍﻭﻱ‬
macerated lips in addition to the angular stomatitis. What .‫ ﻋﻮﺯ ﺍﻟﺜﻴﺎﻣﻴﻦ‬-A
is the most likely diagnosis? .‫ ﻋﻮﺯ ﺭﻳﺒﻮﻓﻼﻓﻴﻦ‬-B
A. Thiamine deficiency .K ‫ ﻋﻮﺯ ﻓﻴﺘﺎﻣﻴﻦ‬-C
B. Riboflavin deficiency .D ‫ ﻋﻮﺯ ﻓﻴﺘﺎﻣﻴﻦ‬-D
C. Vitamin K deficiency .‫ ﻋﻮﺯ ﺍﻟﺤﺪﻳﺪ‬-E
D. Vitamin D deficiency
E. Iron deficiency
B
Une femme de 30 ans se présente depuis 2 semaines avec
de petites blessures sur les côtés de la bouche. Elle a
récemment adopté un régime végétarien strict. À
l'examen, vous remarquerez qu'elle a une conjonctive
pâle, une langue magenta et des lèvres macérées en plus
de la stomatite angulaire. Quel est le diagnostic le plus
probable?
A. Carence en thiamine
B. carence en riboflavine
C. Carence en vitamine K
D. Carence en vitamine D
E. Carence en fer
30. A 19 yo female presents with dry scaly skin and hair loss. .‫ ﻋﺎﻣﺎ ﺗﻌﺎﻧﻲ ﻣﻦ ﺟﻠﺪ ﻗﺸﺮﻱ ﺟﺎﻑ ﻭﺗﺴﺎﻗﻂ ﺷﻌﺮ‬19 ‫ﺃﻧﺜﻰ ﻋﻤﺮﻫﺎ‬
Anorexia, vomiting, and hyperostosis are also noted. ‫ ﻣﺎ ﻫﻮ ﻓﺮﻁ‬.‫ ﺍﻹﻗﻴﺎء ﻭﻓﺮﻁ ﺍﻟﺘﻌﻈﻢ‬،‫ﻟﻘﺪ ﻟﻮﺣﻆ ﺃﻳﻀﺎ ﻓﻘﺪﺍﻥ ﺍﻟﺸﻬﻴﺔ‬
Which hypervitaminosis can cause these symptoms? :‫ﺍﻟﻔﻴﺘﺎﻣﻴﻦ ﺍﻟﺬﻱ ﻳﻤﻜﻦ ﺃﻥ ﻳﺴﺒﺐ ﻫﺬﻩ ﺍﻷﻋﺮﺍﺽ؟‬
A. Hypervitaminosis A .A ‫ ﻓﺮﻁ ﻓﻴﺘﺎﻣﻴﻦ‬-A
B. Hypervitaminosis B6 .B6 ‫ ﻓﺮﻁ ﻓﻴﺘﺎﻣﻴﻦ‬-B
C. Hypervitaminosis C .C ‫ ﻓﺮﻁ ﻓﻴﺘﺎﻣﻴﻦ‬-C
D. Hypervitaminosis E .E ‫ ﻓﺮﻁ ﻓﻴﺘﺎﻣﻴﻦ‬-D
E. Hypervitaminosis B3
.B3 ‫ ﻓﺮﻁ ﻓﻴﺘﺎﻣﻴﻦ‬-E
A
Une femme de 19 ans présente une peau sèche et
squameuse et une perte de cheveux. Anorexie,
vomissements et hyperostose sont également notés.
Quelle hypervitaminose peut causer ces symptômes?
A. Hypervitaminose A
B. Hypervitaminose B6
C. Hypervitaminose C
D. Hypervitaminose E
E. Hypervitaminose B3

13 A
31. A middle aged male is brought to the emergency ‫ ﺃﺣﻀﺮ ﺇﻟﻰ ﻗﺴﻢ ﺍﻟﻄﻮﺍﺭﺉ ﺑﻌﺪ ﺃﻥ ﻋﺜﺮ‬،‫ﺭﺟﻞ ﻓﻲ ﻣﻨﺘﺼﻒ ﺍﻟﻌﻤﺮ‬
department after he was found wandering down the ‫ ﻭﻻ ﺃﻥ‬،‫ ﻻ ﻳﺴﺘﻄﻴﻊ ﺃﻥ ﻳﻠﻔﻆ ﺍﺳﻤﻪ‬.‫ﻋﻠﻴﻪ ﻳﺘﺠﻮﻝ ﻓﻲ ﺷﻮﺍﺭﻉ ﺍﻟﻤﺪﻳﻨﺔ‬
street in the city. He cannot tell you his name, nor provide ‫ ﻳﺒﺪﻭ ﺑﻤﻈﻬﺮ ﻏﻴﺮ‬،‫ ﻋﻨﺪ ﺍﻟﻔﺤﺺ‬.‫ﻳﻌﻄﻲ ﺃﻱ ﺳﻴﺮﺓ ﺫﺍﺗﻴﺔ ﻋﻦ ﻧﻔﺴﻪ‬
any history about himself. On exam he is rather drowsy, ‫ ﻳﺒﺮﺯ ﺍﻟﻔﺤﺺ ﺍﻟﻌﺼﺒﻲ ﺣﺎﻟﺔ ﺭﺃﺭﺃﺓ‬.‫ ﻭﺑﺤﺎﻟﺔ ﻧﻌﺎﺱ‬،‫ ﻧﺤﻴﻒ‬،‫ﻣﺮﺗﺐ‬
thin, and quite disheveled appearing. Neurologic exam is ‫ ﻫﻮ ﻳﺘﺮﻧﺢ ﻭﻻ ﻳﺴﺘﻄﻴﻊ ﺃﻥ ﻳﺤﺎﻓﻆ‬.‫ﻭﺭﻋﺸﺔ ﻅﺎﻫﺮﺓ ﻓﻲ ﻛﻼ ﺍﻟﻴﺪﻳﻦ‬
significant for nystagmus and a slight tremor is apparent ‫ ﺗﺒﻴﻦ ﻣﻦ ﻓﺤﺺ ﺍﻟﺠﻠﺪ ﺃﻧﻪ ﻭﺳﺦ ﻭﺫﺍﺕ‬.‫ﻋﻠﻰ ﺗﻮﺍﺯﻧﻪ ﺑﺪﻭﻥ ﻣﺴﺎﻋﺪﺓ‬
in both hands. He is ataxic and cannot maintain his ‫ ﻣﺎ ﺗﺒﻘﻰ ﻣﻦ ﻓﺤﺺ ﺳﺮﻳﺮﻱ‬.‫ ﻭﻟﻜﻦ ﺩﻭﻥ ﻧﻤﻮ ﻵﻓﺎﺕ‬،‫ﺭﺍﺋﺤﺔ ﺣﻤﻀﻴﺔ‬
balance without assistance. Skin exam reveals dirt and a :‫ﻣﺎﺫﺍ ﻳﺠﺐ ﻓﻌﻠﻪ ﺑﻌﺪ ﺫﻟﻚ؟‬.‫ﻛﺎﻥ ﺳﻠﺒﻴﺎ‬
sour odor, but no breaks in the skin or gross lesions. The ‫ ﻣﻦ ﻣﺤﻠﻮﻝ ﺩﻛﺴﺘﺮﻭﺯ‬³‫ﺳﻢ‬50 ‫ ﻓﺤﺺ ﺳﻜﺮ ﺍﻟﺪﻡ ﻭﺇﻋﻄﺎء‬-A
rest of his physical exam is negative. What should you do .‫ ﺟﺮﻋﺔ ﻭﺭﻳﺪﻳﺔ ﻓﻮﺭﺍ‬%50
next? .‫ﻣﻠﻎ ﻭﺭﻳﺪﻳﺎ ﻓﻮﺭﺍ‬100 ‫ ﺇﻋﻄﺎء ﺟﺮﻋﺔ ﻣﻦ ﺍﻟﺘﻴﺎﻣﻴﻦ‬-B
A. Draw serum blood sugar and give 50cc bolus of a ‫ ﺍﻧﺘﻈﺎﺭ ﻧﺘﺎﺋﺞ ﺗﺤﺮﻱ ﺍﻟﺴﻤﻮﻡ ﻗﺒﻞ ﺍﻟﺒﺪء ﺑﺄﻱ ﻋﻼﺝ‬-C
50% dextrose solution IV immediately .‫ﺗﺠﺮﻳﺒﻲ‬
B. Give a bolus of thiamine 100 mg IV immediately .‫ﻣﻠﻎ ﻭﺭﻳﺪﻱ ﻓﻮﺭﺍ‬2 ‫ ﺇﻋﻄﺎء ﻧﺎﻟﻮﻛﺴﻮﻥ‬-D
C.Await results of a toxicology screen before initiating ‫ ﻣﻠﻴﻮﻥ ﻭﺣﺪﺓ‬2.4 G ‫ ﻭﺇﻋﻄﺎء ﺑﻨﺴﻠﻠﻴﻦ‬VDRL ‫ ﻓﺤﺺ‬-E
any empiric treatment .‫ﺑﺎﻟﻌﻀﻞ ﻓﻮﺭﺍ‬
D. Give naloxone (Narcan) 2 mg IV immediately
E. Draw a VDRL and give penicillin G 2.4 million
units IM immediately
B
Un homme est amené au service des urgences après avoir
été retrouvé errant dans la rue. Il ne peut pas vous dire
son nom, ni fournir aucune histoire. À l'examen, il est
plutôt somnolent, maigre et échevelé. L'examen
neurologique est significatif pour le nystagmus et un léger
tremblement est visible aux deux mains. Il est ataxique et
ne peut maintenir son équilibre sans assistance. L'examen
de la peau révèle de la saleté et une odeur aigre, mais pas
de fractures ni de lésions macroscopiques. Le reste de son
examen physique est négatif. Que devrait-on faire
ensuite?
A. Tirer le sucre dans le sang et donner immédiatement
50 ml de solution de dextrose à 50%
B. Donnez un bolus de thiamine 100 mg IV
immédiatement
C. Attendre les résultats d'un examen toxicologique
avant de commencer tout traitement empirique
D. Administrer immédiatement 2 mg de naloxone
(Narcan) par voie intraveineuse
E. Demander un VDRL et donner immédiatement 2,4
millions d’unités IM à la pénicilline G

14 A
32. A 23-year-old presents to a dermatologist with a red ‫ ﻟﻘﺪ ﺿﻐﻂ ﻋﻠﻴﻬﺎ‬.‫ ﻋﺎﻣﺎ ﻳﺸﻜﻮ ﻣﻦ ﺁﻓﺔ ﺟﻠﺪﻳﺔ ﺣﻤﺮﺍء‬23 ‫ﺭﺟﻞ ﻋﻤﺮﻩ‬
lesion. The dermatologist applies direct pressure and ‫ ﻣﺎ‬.‫ﻁﺒﻴﺐ ﺍﻟﺠﻠﺪ ﺑﺸﻜﻞ ﻣﺒﺎﺷﺮ ﻭﻻﺣﻆ ﺃﻥ ﺍﻵﻓﺔ ﺍﺑﻴﻀﺖ ﺑﺸﻜﻞ ﻛﺎﻣﻞ‬
notices that the lesion blanches completely. What is the :‫ﻫﻮ ﺍﻟﺘﺸﺨﻴﺺ ﺍﻷﻛﺜﺮ ﺗﺮﺟﻴﺤﺎ؟‬
most likely diagnosis? .‫ ﻭﺭﻡ ﻭﻋﺎﺋﻲ ﻛﺮﺯﻱ‬-A
A. Cherry angioma .‫ ﻧﺠﻤﺔ ﻭﺭﻳﺪﻳﺔ‬-B
B. Venous star .‫ ﻛﺪﻣﺔ‬-C
C. Ecchymosis .‫ ﻭﺭﻡ ﻭﻋﺎﺋﻲ ﻋﻨﻜﺒﻲ‬-D
D. Spider angioma .‫ ﺣﺒﺮﺓ‬-E
E. Petechia
D
Un jeune homme de 23 ans se présente chez un
dermatologue avec une lésion rouge. Le dermatologue
applique une pression directe et remarque que la lésion
blanchit complètement. Quel est le diagnostic le plus
probable?
A. Angiome à la cerise
B. Etoile veineuse
C. Ecchymose
D. Angiome stellaire
E. Petechia
33. A 34-year-old man complains to his internist that his ‫ ﻋﺎﻣﺎ ﻳﺸﻜﻮ ﻟﻄﺒﻴﺒﻪ ﻣﻦ ﺗﺸﻮﺵ ﻓﻲ ﺍﻟﺮﺅﻳﺔ ﻋﻨﺪ‬34 ‫ﺭﺟﻞ ﻋﻤﺮﻩ‬
vision becomes blurry when he looks to the left. On ‫ ﻻﺣﻆ ﺃﻥ ﻟﺪﻳﻪ ﺣﻤﻠﻘﺔ ﺃﻣﺎﻣﻴﺔ‬،‫ ﻋﻨﺪ ﺍﻟﻔﺤﺺ‬.‫ﺍﻟﻨﻈﺮ ﻧﺤﻮ ﺍﻟﺸﻤﺎﻝ‬
examination, he is noted to have parallel forward and ،‫ ﻋﻨﺪ ﺍﻟﺤﻤﻠﻘﺔ ﻋﻠﻰ ﺍﻟﺠﺎﻧﺐ ﺍﻷﻳﺴﺮ‬.‫ﻣﺘﻮﺍﺯﻳﺔ ﻭﻋﻠﻰ ﺍﻟﺠﺎﻧﺐ ﺍﻷﻳﻤﻦ‬
right lateral gazes. On left lateral gaze, his right eye ‫ ﻭﻟﻜﻦ ﻋﻴﻨﻪ ﺍﻟﺒﺴﺮﻯ ﻻ ﺗﺘﺤﺮﻙ‬،‫ﺗﺘﺤﺮﻙ ﻋﻴﻨﻪ ﺍﻟﻴﻤﻨﻰ ﺑﺸﻜﻞ ﻁﺒﻴﻌﻲ‬
moves normally, but his left eye does not move outward. :‫ ﺃﻳﻦ ﺗﻜﻤﻦ ﺍﻵﻓﺔ؟‬.‫ﻧﺤﻮ ﺍﻟﺨﺎﺭﺝ‬
Where is the lesion? .‫ ﺍﻟﻌﺼﺐ ﺍﻟﺮﺍﺑﻊ ﺍﻷﻳﻤﻦ‬-A
A. Right 4th nerve .‫ ﺍﻟﻌﺼﺐ ﺍﻟﺜﺎﻟﺚ ﺍﻷﻳﺴﺮ‬-B
B. Left 3rd nerve .‫ ﺍﻟﻌﺼﺐ ﺍﻟﺴﺎﺩﺱ ﺍﻷﻳﻤﻦ‬-C
C. Right 6th nerve .‫ ﺍﻟﻌﺼﺐ ﺍﻟﺮﺍﺑﻊ ﺍﻷﻳﺴﺮ‬-D
D. Left 4th nerve .‫ ﺍﻟﻌﺼﺐ ﺍﻟﺴﺎﺩﺱ ﺍﻷﻳﺴﺮ‬-E
E. Left 6th nerve
E
Un homme de 34 ans se plaint à son interniste que sa
vision devient floue quand il regarde à gauche. À
l'examen, on remarque qu'il a des regards latéraux avant
et droit parallèles. Au regard latéral gauche, son oeil droit
bouge normalement, mais son oeil gauche ne bouge pas
vers l'extérieur. Où est la lésion?
A. 4ème nerf droit
B. 3ème nerf gauche
C. 6ème nerf droit
D. 4ème nerf gauche
E. 6ème nerf gauche

15 A
34. A 77 years old man with known coronary artery diesease ‫ ﺣﻀﺮ ﻣﻦ ﺃﺟﻞ‬،‫ ﻋﺎﻣﺎ ﻳﻌﺎﻧﻲ ﻣﻦ ﺩﺍء ﺷﺮﻳﺎﻧﻲ ﺗﺎﺟﻲ‬77 ‫ﺭﺟﻞ ﻋﻤﺮﻩ‬
presents for hypertension evaluation and progressive ‫ ﺍﻷﺩﻭﻳﺔ ﺗﺘﻀﻤﻦ‬.‫ﺗﻘﻴﻴﻢ ﻟﻀﻐﻄﻪ ﻭﺗﺪﻫﻮﺭ ﻣﺘﻔﺎﻗﻢ ﻟﻮﻅﻴﻔﺔ ﺍﻟﻜﻠﻴﺔ‬
deterioration of kidney function. Medications include .‫ ﻣﻴﺘﻮﺑﺮﻭﻟﻮﻝ ﻭﺃﻣﻠﻮﺩﺑﻴﻦ‬،‫ ﻛﻠﻮﺑﻴﺪﻭﺟﺮﻝ‬،‫ﺃﺳﺒﻴﺮﻳﻦ‬
aspirin , clopidogrel , metoprolol , and amlodipine. .‫ ﺿﻐﻂ ﺍﻟﺪﻡ ﻣﺮﺗﻔﻊ‬:‫ﺍﻟﻔﺤﺺ ﺍﻟﺴﺮﻳﺮﻱ‬
Physical exam : high BP, Normal chemistry with ‫ﺩﺳﻞ‬/‫ ﻣﻠﻎ‬2.2=‫ﺍﻟﺘﺤﻠﻴﻞ ﺍﻟﻜﻴﻤﻴﺎﺋﻲ ﻁﺒﻴﻌﻲ ﻣﻊ ﻛﺮﻳﺎﺗﻨﻴﻦ‬
Creatinine = 2.2 mg/dl and BUN = 35 mg/dl. Negative ‫ ﺗﺼﻮﻳﺮ ﺍﻷﻭﻋﻴﺔ‬.‫ ﺗﺤﻠﻴﻞ ﺍﻟﺒﻮﻝ ﺳﻠﺒﻲ‬.‫ﺩﺳﻞ‬/‫ﻣﻠﻎ‬35=BUN‫ﻭ‬
urinalysis MRA renal arteries: left renal artery stenosis ‫ ﺗﻀﻴﻖ ﺍﻟﺸﺮﻳﺎﻥ‬:‫ﺍﻟﺪﻣﻮﻳﺔ ﺑﺎﻟﺮﻧﻴﻦ ﺍﻟﻤﻐﻨﺎﻁﻴﺴﻲ ﻟﻠﺸﺮﺍﻳﻴﻦ ﺍﻟﻜﻠﻮﻳﺔ‬
80%. Which of the following statements about ACE ‫ ﺃﻱ ﻣﻦ ﺍﻟﻌﺒﺎﺭﺍﺕ ﺍﻟﺘﺎﻟﻴﺔ ﺑﺎﻟﻨﺴﺒﺔ‬.%80 ‫ﺍﻟﻜﻠﻮﻱ ﺍﻷﻳﺴﺮ ﺑﻨﺴﺒﺔ‬
inhibitors in this patient is false? ‫ﻟﻤﺜﺒﻄﺎﺕ ﺧﻤﻴﺮﺓ ﺃﻧﺠﻴﻮﺗﻨﺴﻴﻦ ﺍﻟﻤﻨﻘﻠﺒﺔ ﻋﻨﺪ ﻫﺬﺍ ﺍﻟﻤﺮﻳﺾ ﻏﻴﺮ‬
A. They may initially worsen renal failure :‫ﺻﺤﻴﺤﺔ؟‬
B. They are preferred antihypertensive .‫ ﻗﺪ ﺗﺰﻳﺪ ﺍﻟﻘﺼﻮﺭ ﺍﻟﻜﻠﻮﻱ ﺳﻮءﺍ‬-A
C. They are contraindicated because of renal .‫ ﻫﻲ ﻣﻦ ﺧﺎﻓﻀﺎﺕ ﺍﻟﻀﻐﻂ ﺍﻟﻤﻔﻀﻠﺔ‬-B
artery stenosis .‫ ﻫﻲ ﻣﻀﺎﺩ ﺍﺳﺘﻄﺒﺎﺏ ﺑﺴﺒﺐ ﺗﻀﻴﻖ ﺍﻟﺸﺮﻳﺎﻥ ﺍﻟﻜﻠﻮﻱ‬-C
D. They are secondary therapy if the patient ‫ ﻫﻲ ﺍﻟﻌﻼﺝ ﺍﻟﺜﺎﻧﻮﻱ ﺇﺫﺍ ﺭﻓﺾ ﺍﻟﻤﺮﻳﺾ ﺍﻟﺘﺪﺧﻞ‬-D
.‫ﺍﻟﺠﺮﺍﺣﻲ‬
refuses intervention
‫ ﻛﻞ ﺍﻹﺟﺎﺑﺎﺕ ﻫﻲ ﺧﻄﺄ‬-E
E. All the answers are false

Un homme de 77 ans atteint de coronaropathie connue se C


présente pour une évaluation de l'hypertension et une
détérioration progressive de la fonction rénale. Les
médicaments comprennent l'aspirine, le clopidogrel, le
métoprolol et l'amlodipine. Examen physique: TA élevée
biochimie normale avec créatinine = 2,2 mg / dl et BUN =
35 mg / dl. Analyse d'urine négative artères rénales MRA:
sténose de l'artère rénale gauche 80%. Parmi les
affirmations suivantes sur les inhibiteurs ACE
(angiotensine conversing enzyme) chez ce patient,
laquelle est fausse?
A. Ils peuvent initialement aggraver l'insuffisance
rénale
B. Ils sont les antihypertenseurs préférés
C. Ils sont contre-indiqués en raison d'une sténose
de l'artère rénale
D. Ils sont un traitement secondaire si le patient
refuse une intervention
E. Toutes les résonses sont fausses
35. Four days after the onset of clinical symptoms of acute ‫ﺑﻌﺪ ﺃﺭﺑﻌﺔ ﺃﻳﺎﻡ ﻣﻦ ﺑﺪء ﺍﻷﻋﺮﺍﺽ ﺍﻟﺴﺮﻳﺮﻳﺔ ﻻﻟﺘﻬﺎﺏ ﺍﻟﺠﻴﻮﺏ ﺍﻷﻧﻔﻴﺔ‬
sinusitis, a 22 year old man is seeking your advice. Fever ‫ ﻟﻘﺪ ﺍﺧﺘﻔﺖ ﺍﻟﺤﺮﺍﺭﺓ‬.‫ ﻋﺎﻣﺎﻳﻄﻠﺐ ﺍﻟﻨﺼﻴﺤﺔ‬22 ‫ ﺭﺟﻞ ﻋﻤﺮﻩ‬،‫ﺍﻟﺤﺎﺩ‬
has disappeared after two days, but the nasal secretions ‫ ﻭﻟﻜﻦ ﺍﻹﻓﺮﺍﺯﺍﺕ ﺍﻷﻧﻔﻴﺔ ﺗﻐﻴﺮ ﻟﻮﻧﻬﺎﻣﻦ ﺍﻟﻠﻮﻥ ﺍﻟﺼﺎﻓﻲ‬،‫ﺑﻌﺪ ﻳﻮﻣﻴﻦ‬
have changed in color from clear to yellow. What should :‫ ﻣﺎ ﻫﻮ ﺍﻟﻌﻤﻞ ﺍﻟﻤﻄﻠﻮﺏ؟‬.‫ﺇﻟﻰ ﺍﻟﻠﻮﻥ ﺍﻷﺻﻔﺮ‬
be done? .‫ﺣﻤﺾ ﺍﻟﻜﻼﻓﻮﻧﻴﻚ‬-‫ ﺍﻟﺒﺪء ﺑﺄﻣﻮﻛﺴﻴﺴﻴﻠﻠﻴﻦ‬-A
A. Start Amoxicillin-clavulinic acid .‫ ﺍﻟﺒﺪء ﺑﺴﻴﺒﺮﻭﻓﻠﻮﻛﺴﺎﺳﻴﻦ‬-B
B. Start Ciprofloxacin .‫ ﺍﻷﻧﻒ ﻭﺍﻟﺤﻨﺠﺮﺓ‬،‫ ﺍﺳﺘﺸﺎﺭﺓ ﻁﺒﻴﺐ ﺍﻷﺫﻥ‬-C
C. Refer for ENT .‫ ﺍﻟﺒﺪء ﺑﺴﻔﺘﺮﻳﺎﻛﺴﻮﻥ‬-D
D. Start ceftriaxone .‫ ﻣﻌﺎﻟﺠﺔ ﺍﻷﻋﺮﺍﺽ‬-E
E. Symptomatic therapy
E
Quatre jours après le début des symptômes cliniques de la
sinusite aiguë, un homme de 22 ans demande votre avis.
La fièvre a disparu au bout de deux jours, mais la couleur
des sécrétions nasales est passée de claire à jaune. Qu'est-
ce qui devrait être fait?
A. Démarrer amoxicilline-acide clavulinique
B. Démarrer la ciprofloxacine
C. Se référer pour un ORL
D. Démarrer la ceftriaxone
E. Thérapie symptomatique

16 A
36. A 65-year-old woman presents to the emergency ‫ ﻧﻮﺑﺎﺕ‬5 ‫ ﻋﺎﻣﺎ ﺣﻀﺮﺕ ﺇﻟﻰ ﻗﺴﻢ ﺍﻟﻄﻮﺍﺭﺉ ﺑﺴﺒﺐ‬65 ‫ﺍﻣﺮﺃﺓ ﻋﻤﺮﻫﺎ‬
department for five episodes of melena. Her hemoglobin .‫ﺩﺳﻞ‬/‫ﻍ‬6.3 ‫ ﻋﻨﺪ ﻭﺻﻮﻟﻬﺎ ﻛﺎﻥ ﺍﻟﻬﻴﻤﻮﻏﻠﻮﺑﻴﻦ‬.‫ﻣﻦ ﺍﻟﺘﻐﻮﻁ ﺍﻷﺳﻮﺩ‬
upon arrival is 6.3 g/dL. An urgent EGD ‫ﺗﻢ ﺇﺟﺮﺍء ﺗﻨﻈﻴﺮ ﻣﺮﻳﺌﻲ ﻣﻌﺪﻱ ﻣﻌﻮﻱ ﻋﺎﺟﻞ ﺣﻴﺚ ﺗﺒﻴﻦ ﻭﺟﻮﺩ ﺛﻼﺙ‬
(Esogastroduedonoscopy) is performed and three 3 ‫ﺳﻢ ﺇﻟﻰ‬1 ‫ ﺍﺑﺘﺪﺍء ﻣﻦ‬،‫ﺁﻓﺎﺕ ﻣﺘﻘﺮﺣﺔ ﻓﻲ ﺟﺴﻢ ﺍﻟﻤﻌﺪﺓ ﻭﺍﻟﺘﺠﻮﻳﻒ‬
ulcerated mass-like lesions were found in the gastric body ‫ ﺃﻅﻬﺮﺕ ﺍﻟﺨﺰﻋﺔ ﻅﻮﺍﻫﺮ ﻧﺴﺠﻴﺔ ﺧﺒﻴﺜﺔ ﻻ ﺗﺘﻔﻖ ﻣﻊ ﻛﺎﺭﺳﻴﻨﻮﻣﺎ‬.‫ﺳﻢ‬
and antrum, ranging from 1 cm to 3 cm. Biopsy showed ‫ ﻣﺎ ﻫﻮ ﺍﻟﻤﻮﻗﻊ ﺍﻷﺳﺎﺳﻲ ﺍﻷﻛﺜﺮ ﺷﻴﻮﻋﺎ ﻟﻠﺪﺍء ﺍﻟﻨﻘﻴﻠﻲ‬.‫ﻏﺪﻳﺔ ﻣﻌﺪﻳﺔ‬
malignant histologic features that are not consistent with :‫ﻟﻠﻤﻌﺪﺓ؟‬
gastric adenocarcinoma. What is the most common .‫ ﺍﻟﻤﺒﻴﺾ‬-A
primary site of metastatic disease to the stomach? .‫ ﺍﻟﺮﺋﺔ‬-B
A. Ovary .‫ ﺍﻟﻘﻮﻟﻮﻥ‬-C
B. Lung .‫ ﺍﻟﻜﺒﺪ‬-D
C. Colon .‫ ﺍﻟﺜﺪﻱ‬-E
D. Liver
E. Breast
E
Une femme de 65 ans se présente au service des urgences
pour cinq épisodes de méléna. À son arrivée, son
hémoglobine est de 6,3 g / dL. Une oesogastro-
duodenoscopie urgente est réalisée et trois lésions
ulcéreuses ressemblant à une masse ont été trouvées dans
le corps gastrique et l'antre, allant de 1 cm à 3 cm. La
biopsie a montré des caractéristiques histologiques
malignes non compatibles avec un adénocarcinome
gastrique. Quel est le site primaire de la maladie
métastatique le plus commun à l’estomac?
A. Ovaire
B. poumon
C. Colon
D. Foie
E. Seins
37. A 67 yo woman who is a 30 pack-year smoker complains ‫ ﺗﺸﻜﻮ ﻣﻦ ﻧﻔﺚ‬،‫ ﻋﻠﺒﺔ ﻓﻲ ﺍﻟﺴﻨﺔ‬30 ‫ ﻋﺎﻣﺎ ﺗﺪﺧﻦ‬67 ‫ﺍﻣﺮﺃﺓ ﻋﻤﺮﻫﺎ‬
of hemoptysis and chest oppression. Chest X-ray is ‫ ﺗﺒﻴﻦ ﻣﻦ ﺍﻷﺷﻌﺔ ﺍﻟﺴﻴﻨﻴﺔ ﻟﻠﺼﺪﺭ ﻭﺟﻮﺩ ﺁﻓﺔ‬.‫ﺍﻟﺪﻡ ﻭﺿﻴﻖ ﻓﻲ ﺍﻟﺼﺪﺭ‬
obtained and reveals a central mass lesion. Cavitations are ‫ ﺃﻅﻬﺮﺕ ﺍﻟﺼﻮﺭﺓ ﺍﻟﻤﻘﻄﻌﻴﺔ‬.‫ ﻻ ﻭﺟﻮﺩ ﻟﻠﻜﻬﻮﻑ‬.‫ﺫﺍﺕ ﻛﺘﻠﺔ ﻣﺮﻛﺰﻳﺔ‬
not seen. Chest CT scan reveals widespread metastasis to ‫ﻟﻠﺼﺪﺭ ﻧﻘﻴﻠﺔ ﺫﺍﺕ ﺍﻧﺘﺸﺎﺭ ﻭﺍﺳﻊ ﺇﻟﻰ ﺳﻠﺴﻠﺔ ﺍﻟﻌﻘﺪﺓ ﺍﻟﻠﻴﻤﻔﺎﻭﻳﺔ ﻧﻈﻴﺮﺓ‬
the paratracheal lymph node chain and the ‫ ﻟﻘﺪ ﺗﻢ ﺇﺟﺮﺍء ﺗﻨﻈﻴﺮ ﻗﺼﺒﻲ‬.‫ﺍﻟﺮﻏﺎﻣﻰ ﻭﺍﻟﻠﻤﻔﺎﻭﻳﺎﺕ ﻓﻮﻕ ﺍﻟﺘﺮﻗﻮﺓ‬
supraclavicular lymphatics. Bronchoscopy and biopsy are :‫ ﻣﺎ ﻫﻮ ﺍﻟﺘﻔﺴﻴﺮ ﺍﻷﻛﺜﺮ ﺗﺮﺟﻴﺤﺎ ﻟﻬﺬﻩ ﺍﻟﻨﺘﺎﺋﺞ؟‬.‫ﻭﺃﺧﺬ ﺧﺰﻋﺔ‬
undertaken. What is the most likely explanation for these .‫ ﻛﺎﺭﺳﻴﻨﻮﻣﺎ ﻛﺒﻴﺮﺓ ﺍﻟﺨﻠﻴﺔ‬-A
findings: .‫ ﺍﻧﺨﻤﺎﺹ‬-B
A. Large cell carcinoma .‫ ﻛﺎﺭﺳﻴﻨﻮﻣﺎ ﻏﺪﻳﺔ‬-C
B. Atelectasis .‫ ﻛﺎﺭﺳﻴﻨﻮﻣﺎ ﺍﻟﺨﻠﻴﺔ ﺍﻟﺤﺮﺷﻔﻴﺔ‬-D
C. Adenocarcinoma .‫ ﻛﺎﺭﺳﻴﻨﻮﻣﺎ ﺻﻐﻴﺮﺓ ﺍﻟﺨﻠﻴﺔ‬-E
D. Squamous cell carcinoma
E. Small cell carcinoma
E
Une femme de 67 ans qui fume depuis 30 ans se plaint
d'hémoptysie et d'oppression thoracique. La radiographie
pulmonaire est obtenue et révèle une masse centrale. Les
cavitations ne sont pas vues. Le scanner thoracique révèle
une métastase étendue à la chaîne des ganglions
lymphatiques paratrachéaux et des lymphatiques
supraclaviculaires. Bronchoscopie et des biopsies sont
entrepris. Quelle est l'explication la plus probable ?
A. Cancer à grandes cellules
B. Atélectasie
C. Adénocarcinome
D. Carcinome épidermoïde
E. Cancer à petites cellules

17 A
38. A 19 yo male college student presents for hemoptysis. He ‫ ﻛﺎﻥ ﻣﺼﺎﺑﺎ‬.‫ ﻋﺎﻣﺎ ﻳﺸﻜﻮ ﻣﻦ ﻧﻔﺚ ﺍﻟﺪﻡ‬19 ‫ﻁﺎﻟﺐ ﺟﺎﻣﻌﻲ ﻋﻤﺮﻩ‬
has had a cold and sore throat for the last 2 week and 4 ‫ ﺃﻳﺎﻡ ﺭﻛﺾ ﻓﻲ‬4 ‫ ﻭﻣﻨﺬ‬،‫ﺑﻨﺰﻟﺔ ﺑﺮﺩ ﻭﺍﻟﺘﻬﺎﺏ ﺍﻟﺤﻠﻖ ﻣﻨﺬ ﺃﺳﺒﻮﻋﻴﻦ‬
days ago ran in a 10-Km race despite being told not to by ‫ ﻟﻘﺪ ﻧﻔﻰ ﺍﻟﺘﻌﺮﻕ ﺍﻟﻠﻴﻠﻲ ﺃﻭ‬.‫ﻛﻢ ﺑﺎﻟﺮﻏﻢ ﻣﻦ ﺭﻓﺾ ﻭﺍﻟﺪﻳﻪ‬10 ‫ﺳﺒﺎﻕ‬
his parents. He denies night sweats or weight loss but ‫ ﺗﺒﻴﻦ ﻣﻦ ﺍﻟﻔﺤﺺ‬.‫ﺧﺴﺎﺭﺓ ﺍﻟﻮﺯﻥ ﻭﻟﻜﻨﻪ ﻳﻌﺎﻧﻲ ﻣﻦ ﺣﺮﺍﺭﺓ ﺫﺍﺗﻴﺔ‬
does have a subjective fever. Physical exam reveals some ‫ﺍﻟﺴﺮﻳﺮﻱ ﺑﻌﺾ ﺍﻟﺨﺮﺧﺮﺓ ﻓﻲ ﺣﻘﻞ ﺍﻟﺮﺋﺔ ﺍﻟﻤﺘﻮﺳﻂ ﺍﻷﻳﻤﻦ ﻭﻟﻜﻦ ﻻ‬
rales in the right mid-lung field but no evidence of ronchi ‫ ﻣﺎ ﻫﻮ ﺍﻟﺘﺸﺨﻴﺺ‬.‫ ﺃﺻﻮﺍﺕ ﺍﻟﺘﻨﻔﺲ ﻣﺘﺴﺎﻭﻳﺔ‬.‫ﻳﻮﺟﺪ ﺃﺛﺮ ﻷﺯﻳﺰ‬
or wheeze. Breath sounds are equal. What is the most :‫ﺍﻷﻛﺜﺮ ﺗﺮﺟﻴﺤﺎ؟‬
likely diagnosis? .‫ ﻛﺎﺭﺳﻴﻨﻮﻣﺎ ﻓﻲ ﺍﻟﺮﺋﺔ‬-A
A. Carcinoma of the lung .‫ ﺫﺍﺕ ﺭﺋﺔ‬-B
B. Pneumonia .‫ ﺍﻟﺘﻬﺎﺏ ﻟﺴﺎﻥ ﺍﻟﻤﺰﻣﺎﺭ ﺍﻟﻔﻴﺮﻭﺳﻲ‬-C
C. Viral epiglottitis .‫ ﺍﻟﺘﻬﺎﺏ ﻗﺼﺒﺎﺕ‬-D
D. Bronchitis .‫ ﺩﺍء ﺍﻟﺴﻞ‬-E
E. Tuberculosis
D
Un étudiant de 19 ans se présente pour une hémoptysie. Il
a eu un rhume et un mal de gorge depuis 2 semaines et il
y a 4 jours, il a couru une course de 10 km malgré le fait
que ses parents lui aient interdit de le faire. Il nie les
sueurs nocturnes ou la perte de poids mais a une fièvre.
L'examen physique révèle des résonances dans le champ
mi-pulmonaire droit, mais aucune trace de ronchi ou de
respiration sifflante. Les bruits respiratoires sont
symétriques. Quel est le diagnostic le plus probable?
A. Carcinome pulmonaire
B. Pneumonie
C. épiglottite virale
D. bronchite
E. Tuberculose

18 A
39. A 65 yo obese woman presents for check up. She admits ‫ ﻟﻘﺪ ﺃﻓﺎﺩﺕ‬.‫ ﻋﺎﻣﺎ ﺣﻀﺮﺕ ﻣﻦ ﺃﺟﻞ ﻓﺤﺺ ﺷﺎﻣﻞ‬65 ‫ﺍﻣﺮﺃﺓ ﻋﻤﺮﻫﺎ‬
to weight loss, worsening vision, and several yeast ‫ ﻭﻋﺪﺓ ﺇﺻﺎﺑﺎﺕ ﻓﻄﺮﻳﺔ ﺧﻼﻝ‬،‫ ﺭﺅﻳﺔ ﺗﺰﺩﺍﺩ ﺳﻮءﺍ‬،‫ﻋﻦ ﺧﺴﺎﺭﺓ ﻭﺯﻥ‬
infections over the previous year. She has smoked 1 ‫ ﻫﻲ‬.‫ ﺳﻨﺔ‬35 ‫ ﻫﻲ ﺗﺪﺧﻦ ﻋﻠﺒﺔ ﻭﺍﺣﺪﺓ ﺑﺎﻟﻴﻮﻡ ﻣﻨﺬ‬.‫ﺍﻟﺴﻨﺔ ﺍﻟﻤﺎﺿﻴﺔ‬
pack/day for the last 35 years. She drinks alcohol ‫ ﺳﻴﺮﺗﻬﺎ ﺍﻟﻄﺒﻴﺔ ﺍﻟﻘﺪﻳﻤﺔ ﺗﺘﻀﻤﻦ ﺍﻛﺘﺌﺎﺏ ﺗﻢ‬.‫ﺗﺸﺮﺏ ﻛﺤﻮﻝ ﺑﺎﻟﻤﻨﺎﺳﺒﺎﺕ‬
occasionally. Her past medical history is positive for ‫ ﺳﻨﺔ ﺑﺴﺒﺐ‬40 ‫ ﻭﺍﺳﺘﺌﺼﺎﻝ ﺭﺣﻢ ﺟﺬﺭﻱ ﻓﻲ ﻋﻤﺮ‬،‫ ﺭﺑﻮ‬،‫ﻣﻌﺎﻟﺠﺘﻪ‬
resolved depression, asthma, and a radical hysterectomy ،‫ﺃﻅﻬﺮ ﺗﻨﻈﻴﺮ ﻗﺎﻉ ﺍﻟﻌﻴﻦ ﻧﺰﻭﻑ ﺷﺒﻜﻴﺔ ﻣﺘﻔﺮﻗﺔ‬.‫ﺍﻟﻠﻴﻔﺎﻧﻴﺎﺕ ﺍﻟﺮﺣﻤﻴﺔ‬
at the age of 40 years for uterine fibroids. Fundoscopy ‫ ﻣﺎ ﺗﺒﻘﻰ ﻣﻦ ﺍﻟﻔﺤﺺ ﺍﻟﺴﺮﻳﺮﻱ‬.‫ ﻭﺇﻓﺮﺍﺯﺍﺕ ﺻﻠﺒﺔ‬،‫ﺗﻮﺳﻊ ﻭﺭﻳﺪﻱ‬
reveals scattered retinal hemorrhages, venous dilation, :‫ ﺃﻱ ﻣﻦ ﺍﻟﺘﺎﻟﻲ ﻫﻮ ﺍﻟﺨﻄﻮﺓ ﺍﻟﺘﺎﻟﻴﺔ ﺍﻷﻓﻀﻞ ﻓﻲ ﺍﻟﻤﻌﺎﻟﺠﺔ؟‬.‫ﻛﺎﻥ ﺳﻠﻴﻤﺎ‬
and hard exudates. The remainder of the physical exam is .‫ ﺇﺟﺮﺍء ﻓﺤﺺ ﻏﻠﻮﻛﻮﺯ ﺍﻟﺒﻼﺯﻣﺎ ﺻﻴﺎﻣﻲ‬-A
normal. Which of the following is the next best step in .‫ ﻏﻤﻴﺴﺔ ﺑﻮﻟﻴﺔ‬-B
management? .‫ ﺍﻟﺘﺸﺠﻴﻊ ﻋﻠﻰ ﺧﺴﺎﺭﺓ ﺇﺿﺎﻓﻴﺔ ﻟﻠﻮﺯﻥ‬-C
A. Perform a fasting plasma glucose .‫ ﻟﻄﺎﺧﺔ ﺑﺎﺑﺎ ﻧﻴﻜﻮﻻﻭ‬-D
B. Urine dipstick .‫ ﺇﺣﺎﻟﺘﻬﺎ ﺇﻟﻰ ﻁﺒﻴﺐ ﻋﻴﻮﻥ‬-E
C. Encourage additional weight loss
D. Pap smear
E. Referral to an ophthalmologist

Une femme obèse de 65 ans se présente pour un examen. A


Elle admet avoir perdu du poids, avoir une vision
détériorée et avoir contracté plusieurs infections à levures
au cours de l'année précédente. Elle a fumé 1 paquet par
jour au cours des 35 dernières années. Elle boit de l'alcool
occasionnellement. Ses antécédents médicaux sont positifs
pour la résolution de la dépression, l'asthme et une
hystérectomie à l'âge de 40 ans pour les fibromes utérins.
La fondoscopie révèle des hémorragies rétiniennes
éparses, une dilatation veineuse et des exsudats larges. Le
reste de l'examen physique est normal. Lequel des
éléments suivants constitue la prochaine meilleure étape
de la gestion?
A. Effectuer une glycémie à jeun
B. Bandelette urinaire
C. Encourager une perte de poids supplémentaire
D. frottis
E. Référer à un ophtalmologue
40. A 28 yo woman with known neurofibromatosis type I I ‫ ﻋﺎﻣﺎ ﺗﻌﺎﻧﻲ ﻣﻦ ﻭﺭﻡ ﻟﻴﻔﻲ ﻋﺼﺒﻲ ﻧﻤﻂ‬28 ‫ﺍﻣﺮﺃﺓ ﻋﻤﺮﻫﺎ‬
presents with palpitations and recurrent headaches that ‫ﺣﻀﺮﺕ ﻭﻫﻲ ﺗﺸﻜﻮ ﻣﻦ ﺧﻔﻘﺎﻥ ﻭﺻﺪﺍﻉ ﻣﺘﻜﺮﺭ ﻻ ﻳﻬﺪﺃ ﺑﺎﺳﺘﻌﻤﺎﻝ‬
are not relieved by paracetamol. She also feels dizzy and ‫ ﺿﻐﻂ‬.‫ ﻫﻲ ﺃﻳﻀﺎ ﺗﺸﻌﺮ ﺑﺪﻭﺍﺭ ﻭﺗﻌﺮﻕ ﺃﺛﻨﺎء ﺍﻟﻨﻮﺑﺎﺕ‬.‫ﺍﻟﺒﺎﺭﺍﺳﻴﺘﺎﻣﻮﻝ‬
becomes sweaty during these episodes. Her blood :‫ ﻣﺎ ﻫﻮ ﺍﻟﺘﺸﺨﻴﺺ ﺍﻷﻛﺜﺮ ﺗﺮﺟﻴﺤﺎ؟‬.‫ ﻣﻠﻢ ﺯﺋﺒﻖ‬120/220 ‫ﺍﻟﺪﻡ‬
pressure is found to be 220/120mmHg. What is the most .‫ ﺿﺨﺎﻣﺔ ﺃﻁﺮﺍﻑ‬-A
likely diagnosis? .‫ ﻣﺘﻼﺯﻣﺔ ﻛﺎﺭﺳﻴﻨﻮﻳﺪ‬-B
A. Acromegaly .‫ ﺍﻧﺴﻤﺎﻡ ﺩﺭﻗﻲ‬-C
B. Carcinoid syndrome .‫ ﻭﺭﻡ ﺍﻟﻘﻮﺍﺗﻢ‬-D
C. Thyrotoxicosis
D. Phaeochromocytoma
D
Une femme de 28 ans atteinte d'une neurofibromatose de
type I présente des palpitations et des maux de tête
récurrents non soulagés par le paracétamol. Elle se sent
également étourdie et transpire pendant ces épisodes. Sa
tension artérielle est de 220 / 120mmHg. Quel est le
diagnostic le plus probable?
A. Acromégalie
B. syndrome carcinoïde
C. Thyrotoxicose
D. Phéochromocytome

19 A
41. One year after bone marrow transplantation from a ‫ﺑﻌﺪ ﻣﺮﻭﺭ ﺳﻨﺔ ﻋﻠﻰ ﺯﺭﻉ ﻧﺨﺎﻉ ﻋﻈﻤﻲ ﻣﻦ ﻣﺘﺒﺮﻉ ﻟﻌﻼﺝ ﻻ‬
matched donor to treat non-Hodgkin’s lymphoma, a ‫ ﺣﻀﺮ ﺍﻟﻤﺮﻳﺾ ﻭﻫﻮ ﻳﺸﻜﻮ ﻣﻦ ﺗﻬﻴﺞ ﺟﻠﺪﻱ‬،‫ﻫﻮﺩﺟﻜﻴﻦ ﻟﻤﻔﻮﻣﺎ‬
patient presents complaining of widespread skin ‫ ﺍﻟﺠﻠﺪ ﻣﺘﻮﺳﻒ ﻓﻲ ﻣﺴﺎﺣﺎﺕ ﻭﻫﻨﺎﻙ ﻁﻔﺢ ﺑﻘﻌﻲ‬.‫ﻭﺍﺳﻊ ﺍﻻﻧﺘﺸﺎﺭ‬
irritation. The skin is exfoliated in areas and there is a ‫ ﻟﻢ ﻳﺸﻌﺮ ﺑﺄﻧﻪ ﻋﻠﻰ‬.‫ ﻓﻲ ﺣﻴﻦ ﺃﻥ ﺍﻷﺻﺎﺑﻊ ﺃﺻﺒﺤﺖ ﻣﺘﺼﻠﺒﺔ‬،‫ﺟﻠﺪﻱ‬
maculopapular rash, while the fingers are becoming ‫ﻣﺎ ﻳﺮﺍﻡ ﻟﻔﺘﺮﺓ ﻣﻊ ﻭﺟﻮﺩ ﻧﻮﺑﺎﺕ ﻣﻦ ﺍﻹﺳﻬﺎﻝ ﻭﺍﻟﺘﻬﺎﺑﺎﺕ ﺻﺪﺭ‬
sclerotic. He has not been feeling well for a while with :‫ ﻣﺎ ﻫﻮ ﺍﻟﺴﺒﺐ ﺍﻷﻛﺜﺮ ﺗﺮﺟﻴﺤﺎ ﻟﻬﺬﻩ ﺍﻟﺤﺎﻟﺔ؟‬.‫ﻣﺘﻜﺮﺭﺓ‬
episodes of diarrhea and recurrent chest infections. What .‫ ﺭﻓﺾ ﺯﺭﻉ ﺣﺎﺩ‬-A
is the most likely reason for this? .‫ ﺇﺻﺎﺑﺔ ﺑﺪﺍء ﺍﻟﻤﻨﻄﻘﺔ‬-B
A. Acute graft rejection .‫ ﺑﺪء ﺟﺪﻳﺪ ﻟﺘﺼﻠﺐ ﺍﻟﺠﻠﺪ‬-C
B. Herpes zoster infection .‫ ﻣﺮﺽ ﻣﺰﻣﻦ ﻣﻦ ﻧﻮﻉ ﺍﻟﻄﻌﻢ ﻣﻘﺎﺑﻞ ﺍﻟﻤﻀﻴﻒ‬-D
C. New-onset scleroderma .‫ ﺗﺄﺛﻴﺮﺍﺕ ﺟﺎﻧﺒﻴﺔ ﻟﻤﺜﺒﻄﺎﺕ ﺍﻟﻤﻨﺎﻋﺔ‬-E
D. Chronic graft-versus-host disease
E. Side effects of immunosuppressants
D
Un an après la greffe de moelle osseuse à partir d’un
donneur compatible pour traiter un lymphome non
hodgkinien, un patient se plaint d’une irritation cutanée
généralisée. La peau est exfoliée par endroits et il y a une
éruption maculo-papuleuse, tandis que les doigts
deviennent sclérosés. Il ne se sent pas bien depuis un
moment avec des épisodes de diarrhée et des infections
thoraciques récurrentes. Quelle est la raison la plus
probable pour cela?
A. Rejet aigu du greffon
B. Infection par l'herpès zoster
C. Sclérodermie récente
D. Maladie chronique du greffon v/s hôte
E. Effets secondaires des immunosuppresseurs
42. A 59 yo woman with known polycythemia vera presents ،‫ ﻋﺎﻣﺎ ﺗﻌﺎﻧﻲ ﻣﻦ ﻛﺜﺮﺓ ﺍﻟﺨﻼﻳﺎ ﺍﻟﺤﻤﺮﺍء ﺍﻟﺤﻘﻴﻘﻴﺔ‬59 ‫ﺍﻣﺮﺃﺓ ﻋﻤﺮﻫﺎ‬
to the emergency department with right upper quadrant ‫ﺣﻀﺮﺕ ﺇﻟﻰ ﻗﺴﻢ ﺍﻟﻄﻮﺍﺭﺉ ﻭﻫﻲ ﺗﺸﻜﻮ ﻣﻦ ﺃﻟﻢ ﻓﻲ ﺍﻟﻤﺮﺑﻊ ﺍﻟﻌﻠﻮﻱ‬
pain, tender hepatomegaly and gross ascites, which has ‫ ﻻ‬.‫ ﻭﺍﻟﺬﻱ ﺣﺪﺙ ﻓﺠﺄﺓ‬،‫ ﺿﺨﺎﻣﺔ ﻛﺒﺪ ﻣﺆﻟﻤﺔ ﻭﺍﺳﺘﺴﻘﺎء ﻛﺒﻴﺮ‬،‫ﺍﻷﻳﻤﻦ‬
come on suddenly. There is no jaundice. :‫ ﻣﺎ ﻫﻮ ﺍﻟﺘﺤﺮﻱ ﺍﻟﺘﺎﻟﻲ ﺍﻷﻛﺜﺮ ﻣﻼءﻣﺔ؟‬.‫ﻳﻮﺟﺪ ﻳﺮﻗﺎﻥ‬
What is the next most appropriate investigation? .‫ ﺍﻟﺘﺤﺮﻱ ﻋﻦ ﻓﻴﺮﻭﺱ ﺍﻟﺨﻼﻳﺎ ﺍﻟﻌﺮﻁﻞ‬-A
A. Cytomegalovirus screen .‫ﻏﻠﻮﺗﺎﻣﻴﻞ ﺗﺮﺍﻧﺴﻔﺮﺍﺯ‬-‫ ﻣﺴﺘﻮﻳﺎﺕ ﻏﺎﻣﺎ‬-B
B. Gamma-glutamyl transferase levels .‫ ﺍﻟﺘﺤﺮﻱ ﻋﻦ ﺍﻟﺘﻬﺎﺏ ﺍﻟﻜﺒﺪ ﻓﻲ ﺍﻟﻤﺼﻞ‬-C
C. Hepatitis serology .‫ ﺗﺨﻄﻴﻂ ﺍﻟﺼﺪﻯ ﺍﻟﺪﻭﺑﻠﺮﻱ ﻟﻠﻮﺭﻳﺪ ﺍﻟﻜﺒﺪﻱ‬-D
D. Hepatic vein Doppler ultrasound scan .‫ ﺍﻟﺘﺤﺮﻱ ﻋﻦ ﻓﻴﺮﻭﺱ ﻧﻘﺺ ﺍﻟﻤﻨﺎﻋﺔ ﺍﻟﺒﺸﺮﻱ‬-E
E. Human immunodeficiency virus testing
D
Une femme de 59 ans atteinte d'une polycythémie vraie se
présente au service des urgences en raison d'une douleur
dans le quadrant supérieur droit, d'une hépatomégalie
douloureuse et d'une ascite soudaine. Il n'y a pas de
jaunisse. Quelle est la prochaine étape la plus appropriée?
A. Dépistage du cytomégalovirus
B. Niveaux de gamma-glutamyl transférase
C. Sérologie de l'hépatite
D. Échographie Doppler veineuse hépatique
E. Test du virus de l'immunodéficience humaine

20 A
43. A 78 yo woman presents with palpitations that began an ‫ ﻋﺎﻣﺎ ﺣﻀﺮﺕ ﻭﻫﻲ ﺗﺸﻜﻮ ﻣﻦ ﺧﻔﻘﺎﻥ ﺑﺪﺃ ﻣﻨﺬ‬78 ‫ﺍﻣﺮﺃﺓ ﻋﻤﺮﻫﺎ‬
hour ago. She feels clammy to the touch. On the monitor, ‫ ﺿﺮﺑﺔ‬230 ‫ ﺗﺒﻴﻦ ﻣﻦ ﺟﻬﺎﺯ ﺍﻟﻤﺮﺍﻗﺒﺔ ﺃﻥ ﺳﺮﻋﺔ ﺍﻟﻘﻠﺐ ﻫﻲ‬.‫ﺳﺎﻋﺔ‬
her heart rate is 230bpm and regular. The QRS complexes .P-‫ ﺿﻴﻘﺔ ﻭﻟﻢ ﻳﻼﺣﻆ ﻣﻮﺟﺎﺕ‬QRS ‫ ﺍﻟﻤﺮﻛﺒﺎﺕ‬.‫ﺑﺎﻟﺪﻗﻴﻘﺔ ﻭﻣﻨﺘﻈﻤﺔ‬
are narrow and no P-waves are noted. What is the most :‫ﻣﺎ ﻫﻲ ﺍﻟﻼﻧﻈﻤﻴﺔ ﺍﻷﻛﺜﺮ ﺗﺮﺟﻴﺤﺎ؟‬
likely arrhythmia? .‫ ﺭﺟﻔﺎﻥ ﺃﺫﻳﻨﻲ‬-A
A. Atrial fibrillation .‫ ﺭﻓﺮﻓﺔ ﺃﺫﻳﻨﻴﺔ‬-B
B. Atrial flutter .‫ ﺗﺴﺎﺭﻉ ﻗﻠﺐ ﺑﺎﻟﻌﻘﺪﺓ ﺍﻷﺫﻳﻨﻴﺔ ﺍﻟﺒﻄﻴﻨﻴﺔ‬-C
C. Atrio-ventricular node re-entry tachycardia .‫ ﺗﺴﺎﺭﻉ ﻗﻠﺐ ﺟﻴﺒﻲ‬-D
D. Sinus tachycardia .‫ ﺗﺴﺎﺭﻉ ﻗﻠﺐ ﺑﻄﻴﻨﻲ‬-E
E. Ventricular tachycardia
C
Une femme de 78 ans présente des palpitations depuis
une heure. Elle se sent moite au toucher. Sur le moniteur,
sa fréquence cardiaque est de 230 /mn et régulière. Les
complexes QRS sont étroits et aucune onde P n'est notée.
Quelle est l'arythmie la plus probable?
A. Finrillation auriculaire
B. Flutter auriculaire
C. Tachycardie avec ré-entrée du noeud atrio-
ventriculaire
D. Tachycardie sinusale
E. Tachycardie ventriculaire
44. A 56 yo man with a long history of alcohol abuse presents ،‫ ﻋﺎﻣﺎ ﺫﺍﺕ ﺗﺎﺭﻳﺦ ﻁﻮﻳﻞ ﻣﻦ ﺍﻹﺩﻣﺎﻥ ﻋﻠﻰ ﺍﻟﻜﺤﻮﻝ‬56 ‫ﺭﺟﻞ ﻋﻤﺮﻩ‬
to the emergency department with abdominal pain. On ،‫ ﻋﻨﺪ ﺍﻟﻔﺤﺺ‬.‫ﺣﻀﺮ ﺇﻟﻰ ﻗﺴﻢ ﺍﻟﻄﻮﺍﺭﺉ ﻭﻫﻮ ﻳﺸﻜﻮ ﻣﻦ ﺃﻟﻢ ﺑﻄﻨﻲ‬
examination, he has a distended abdomen with shifting ‫ﻛﺎﻥ ﺍﻟﺒﻄﻦ ﻣﻨﺘﻔﺦ ﻣﻊ ﺃﺻﻤﻴﺔ ﻣﺘﻨﻘﻠﺔ ﻭﻛﺎﻧﺖ ﺩﺭﺟﺔ ﺣﺮﺍﺭﺗﻪ‬
dullness and has a temperature of 38.2ºC. What is the .‫ﻡ‬°38.2
most likely diagnosis? :‫ﻣﺎ ﻫﻮ ﺍﻟﺘﺸﺨﻴﺺ ﺍﻷﻛﺜﺮ ﺗﺮﺟﻴﺤﺎ؟‬
A. Bowel obstruction .‫ ﺍﻧﺴﺪﺍﺩ ﺇﻣﻌﺎء‬-A
B. Liver cirrhosis .‫ ﺗﺸﻤﻊ ﻛﺒﺪ‬-B
C. Mallory-Weiss syndrome .‫ﻭﻳﺲ‬-‫ ﻣﺘﻼﺯﻣﺔ ﻣﺎﻟﻮﺭﻱ‬-C
D. Perforated peptic ulcer .‫ ﻗﺮﺣﺔ ﻫﻀﻤﻴﺔ ﻣﺜﻘﺒﺔ‬-D
E. Spontaneous bacterial peritonitis .‫ ﺍﻟﺘﻬﺎﺏ ﺍﻟﺼﻔﺎﻕ ﺍﻟﺠﺮﺛﻮﻣﻲ ﺍﻟﺘﻠﻘﺎﺋﻲ‬-E
E
Un homme de 56 ans ayant de longs antécédents d'abus
d'alcool se présente au service des urgences avec des
douleurs abdominales. À l'examen, il a un abdomen
distendu avec une matité changeante et une température
de 38,2ºC. Quel est le diagnostic le plus probable?
A. obstruction intestinale
B. Cirrhose du foie
C. Syndrome de Mallory-Weiss
D. Ulcère peptique perforé
E. Péritonite bactérienne spontanée

21 A
45. A 24 yo woman presents with a 3 months history of vague ‫ ﺃﺷﻬﺮ ﻣﻦ ﺁﻻﻡ‬3 ‫ ﻋﺎﻣﺎ ﺃﻓﺎﺩﺕ ﺑﺄﻧﻬﺎ ﺗﻌﺎﻧﻲ ﻣﻨﺬ‬24 ‫ﺍﻣﺮﺃﺓ ﻋﻤﺮﻫﺎ‬
right upper quadrant pains. She has also noticed an ‫ ﻟﻘﺪ ﻻﺣﻈﺖ ﺃﻳﻀﺎ ﺇﻓﺮﺍﺯﺍﺕ‬.‫ﻣﺒﻬﻤﺔ ﻓﻲ ﺍﻟﻤﺮﺑﻊ ﺍﻟﻌﻠﻮﻱ ﺍﻷﻳﻤﻦ‬
abnormal vaginal discharge since starting a new sexual .‫ ﺃﺷﻬﺮ‬4 ‫ﻣﻬﺒﻠﻴﺔ ﻏﻴﺮ ﻁﺒﻴﻌﻴﺔ ﺑﻌﺪ ﺃﻥ ﺑﺪﺃﺕ ﺑﻌﻼﻗﺔ ﺟﻨﺴﻴﺔ ﺟﺪﻳﺪﺓ ﻣﻨﺬ‬
relationship 4 months ago. On examination, there is a ‫ ﻣﺎ ﻫﻮ ﺍﻟﺘﺸﺨﻴﺺ ﺍﻷﻛﺜﺮ‬.‫ ﺗﺒﻴﻦ ﺃﻥ ﻫﻨﺎﻙ ﺍﺣﺘﻜﺎﻙ ﻛﺒﺪﻱ‬،‫ﻋﻨﺪ ﺍﻟﻔﺤﺺ‬
hepatic friction rub. What is the most likely diagnosis? :‫ﺗﺮﺟﻴﺤﺎ؟‬
A. Fitz-Hugh-Curtis syndrome .‫ﻛﻮﺭﺗﻴﺲ‬-‫ﻫﻴﻮﻍ‬-‫ ﻣﺘﻼﺯﻣﺔ ﻓﻴﺘﺰ‬-A
B. Hepatitis C .C ‫ ﺍﻟﺘﻬﺎﺏ ﻛﺒﺪﻱ ﻓﻴﺮﻭﺳﻲ‬-B
C. HIV .‫ ﻓﻴﺮﻭﺱ ﻧﻘﺺ ﺍﻟﻤﻨﺎﻋﺔ ﺍﻟﺒﺸﺮﻱ‬-C
D. Liver abscess .‫ ﺧﺮﺍﺝ ﻛﺒﺪﻱ‬-D
E. Liver infarct .‫ ﺍﺣﺘﺸﺎء ﻛﺒﺪﻱ‬-E
A
Une femme de 24 ans présente depuis 3 mois des
douleurs vagues dans le quadrant supérieur droit. Elle a
également remarqué des pertes vaginales anormales
depuis le début d'une nouvelle relation sexuelle il y a 4
mois. À l'examen, il y a un frottement hépatique. Quel est
le diagnostic le plus probable?
A. Syndrome Fitz-Hugh-Curtis
B. Hépatite C
C. VIH
D. Abcès du foie
E. infarctus du foie
46. Which ONE of the following is a risk factor for developing ‫ﺃﻱ ﻣﻦ ﺍﻟﺘﺎﻟﻲ ﻫﻮ ﺍﻟﻌﺎﻣﻞ ﺍﻟﻤﻬﺪﺩ ﻟﺘﻄﻮﺭ ﺍﻟﻔﺸﻞ ﺍﻟﻜﻠﻮﻱ ﺍﻟﺤﺎﺩ ﻋﻨﺪ‬
acute renal failure in a patient about to receive intravenous ‫ﻣﺮﻳﺾ ﻋﻠﻰ ﻭﺷﻚ ﺃﻥ ﻳﺘﻠﻘﻰ ﺻﺒﺎﻍ ﻅﻠﻴﻠﻲ ﻭﺭﻳﺪﻱ ﺃﺛﻨﺎء ﺇﺟﺮﺍء‬
contrast dye during a surgery act? :‫ﺟﺮﺍﺣﺔ؟‬
a) Dehydration .‫ ﺍﻟﺘﺠﻔﺎﻑ‬-A
b) A peak of hypertension .‫ ﺍﺭﺗﻔﺎﻉ ﺿﻐﻂ ﺍﻟﺪﻡ‬-B
c) The use of fentanyl .‫ ﺍﺳﺘﻌﻤﺎﻝ ﻓﻨﺘﺎﻧﻴﻞ‬-C
d) Old age .‫ ﺍﻟﺴﻦ‬-D
e) A long induction period .‫ ﻓﺘﺮﺓ ﺗﺤﺮﻳﺾ ﻁﻮﻳﻠﺔ‬-E
A
Lequel des facteurs suivants est un facteur de risque pour
le développement d’une insuffisance rénale aiguë chez un
patient sur le point de recevoir une injection intraveineuse
de contraste pendant un acte chirurgical?
a) déshydratation
b) Un pic d'hypertension
c) L'utilisation de fentanyl
d) la vieillesse
e) Une longue période d'induction
47. Post operatively , the worsening situations of ‫ ﺗﻔﺎﻗﻢ ﺣﺎﻻﺕ ﻧﻘﺺ ﺻﻮﺩﻳﻮﻡ ﺍﻟﺪﻡ ﺍﻟﺘﻲ ﻳﻤﻜﻦ‬،‫ﺑﻌﺪ ﺍﻟﻌﻤﻞ ﺍﻟﺠﺮﺍﺣﻲ‬
hyponatremia which can occur include all of these :‫ﺃﻥ ﺗﺤﺪﺙ ﺗﺘﻀﻤﻦ ﻛﻞ ﻣﺎ ﻳﻠﻲ ﻣﺎ ﻋﺪﺍ‬
EXCEPT: .‫ ﻗﺼﻮﺭ ﻛﻠﻮﻱ ﻣﺰﻣﻦ ﻣﺘﻮﺳﻂ ﻣﻮﺟﻮﺩ ﻣﺴﺒﻘﺎ‬-A
a)Pre existing moderate chronic renal insufficiency .‫ ﺃﻣﺮﺍﺽ ﻗﻠﺒﻴﺔ ﻭﺫﻣﻴﺔ‬-B
b)Edematous cardiac pathologies .‫ ﺍﺭﺗﻔﺎﻉ ﺿﻐﻂ ﻣﻮﺟﻮﺩ ﻣﺴﺒﻘﺎ‬-C
c)Pre existing hypertension .‫ ﺟﻮﻉ ﻳﻨﺘﺞ ﻋﻨﻪ ﺑﻴﻠﺔ ﺻﻮﺩﻳﻮﻡ‬-D
d)Starvation producing natriuresis .‫ ﺍﺳﺘﺨﺪﺍﻡ ﻣﺤﺎﻟﻴﻞ ﻣﻨﺨﻔﻀﺔ ﺍﻟﺘﻮﺗﺮ‬-E
e)The use of hypotonic solutions
C
En post opératoire, l’aggravation des situations
d’hyponatrémie qui peut se produire inclut tout ce qui
suit SAUF:
a- Insuffisance rénale chronique modérée
préexistante
b- Pathologies cardiaques oedémateuses
c- Hypertension préexistante
d- La natriurèse dûe au jeûn
e- L'utilisation de solutions hypotoniques

22 A
48. You have completed suturing a laceration on the cheek of ‫ﻟﻘﺪ ﺃﺗﻤﻤﺖ ﺧﻴﺎﻁﺔ ﺗﻬﺘﻚ ﻓﻲ ﺍﻟﺨﺪ ﻟﻤﺮﺍﻫﻖ ﻭﺗﺴﺄﻝ ﻭﺍﻟﺪﺗﻪ ﻣﺘﻰ ﻳﻨﺒﻐﻲ‬
a teenager and his mother is asking when he should ‫ ﻓﻲ ﺃﻱ ﻣﻦ ﺍﻷﻭﻗﺎﺕ ﺍﻟﺘﺎﻟﻴﺔ ﻳﻨﺒﻐﻲ ﺇﺯﺍﻟﺔ‬.‫ﺃﻥ ﻳﻌﻮﺩ ﻹﺯﺍﻟﺔ ﺍﻟﺨﻴﻮﻁ‬
return to have the sutures removed. You inform her that :‫ﺧﻴﻮﻁ ﺍﻟﻮﺟﻪ؟‬
facial sutures should be removed at which one of the .‫ ﺳﺎﻋﺔ‬24 -A
following times after placement ? .‫ ﺃﻳﺎﻡ‬5 ‫ ﺇﻟﻰ‬3 -B
A- 24 hours .‫ ﺃﻳﺎﻡ‬10 ‫ ﺇﻟﻰ‬7 -C
B- 3 to 5 days .‫ ﻳﻮﻡ‬14 -D
C- 7 to 10 days ‫ ﺍﻟﺨﻴﻮﻁ ﺍﻟﻘﺎﺑﻠﺔ ﻟﻼﻣﺘﺼﺎﺹ ﻳﻨﺒﻐﻲ ﺍﺳﺘﺨﺪﺍﻣﻬﺎ ﻋﻠﻰ ﺍﻟﻮﺟﻪ‬-E
D- 14 days .‫ﻓﻘﻂ ﻭﻻ ﺗﺤﺘﺎﺝ ﺇﻟﻰ ﺇﺯﺍﻟﺔ‬
E- Only absorbable sutures should be used on the
face and do not need removal
B
Vous avez terminé la suture d'une lacération sur la joue
d'un adolescent et sa mère demande quand il devrait
revenir pour faire enlever les sutures. Vous l'informez que
les points de suture du visage doivent être retirés à quel
moment après?
A- 24 heures
B- 3 à 5 jours
C- 7 à 10 jours
D- 14 jours
E- Seules les sutures résorbables doivent être
utilisées sur le visage et n'ont pas besoin d'être
enlevées
49. A young patient is being evaluated for a cut on his face. ‫ ﻗﺒﻞ ﺇﺻﻼﺡ ﺃﻱ‬.‫ﻳﺘﻢ ﺗﻘﻴﻴﻢ ﻣﺮﻳﺾ ﺷﺎﺏ ﺑﺴﺒﺐ ﻗﻄﻊ ﻋﻠﻰ ﻭﺟﻬﻪ‬
Before repairing any facial laceration, it is important to ‫ ﻣﻦ ﺍﻟﻤﻬﻢ ﺍﻟﺘﺄﻛﺪ ﻣﻦ ﺃﻥ ﺍﻟﻔﺮﻭﻉ ﺍﻟﺨﻤﺴﺔ ﻟﻠﻌﺼﺐ‬،‫ﺗﻬﺘﻚ ﻭﺟﻬﻲ‬
ensure that the five branches of the facial nerve were not ‫ ﺃﻱ ﻣﻦ ﺍﻟﻤﻨﺎﻭﺭﺍﺕ ﺍﻟﺘﺎﻟﻴﺔ ﺳﻮﻑ ﺗﻄﻠﺐ ﻣﻦ‬.‫ﺍﻟﻮﺟﻬﻲ ﻏﻴﺮ ﻣﺘﺄﺛﺮﺓ‬
affected. Asking the patient to do which one of the :‫ﺍﻟﻤﺮﻳﺾ ﻻﺧﺘﺒﺎﺭ ﻭﻅﻴﻔﺔ ﺍﻟﻔﺮﻉ ﺍﻟﻮﺟﻨﻲ ﻟﻠﻌﺼﺐ ﺍﻟﻮﺟﻬﻲ؟‬
following maneuvers would test the function of the .‫ ﻋﻘﺪ ﺍﻟﺠﺒﻴﻦ ﻭﺭﻓﻊ ﺍﻟﺤﺎﺟﺒﻴﻦ‬-A
zygomatic branch of the facial nerve ? .‫ ﻋﻘﺪ ﺍﻟﻌﻀﻠﺔ ﺍﻟﺠﻠﺪﻳﺔ ﻟﻠﻌﻨﻖ‬-B
A- Contract the forehead and elevate the eyebows .‫ ﻋﺒﻮﺱ‬-C
B- Contract the platysma (neck) muscles .‫ ﻓﺘﺢ ﻭﺇﻏﻼﻕ ﺍﻟﻌﻴﻮﻥ‬-D
C- Frown .‫ ﺍﺑﺘﺴﺎﻣﺔ‬-E
D- Open and shut eyes
E- Smile
D
Un jeune patient est évalué pour une coupure au visage.
Avant de réparer une lacération du visage, il est
important de s'assurer que les cinq branches du nerf facial
ne sont pas affectées. Demander au patient de faire
laquelle des manœuvres suivantes testerait la fonction de
la branche zygomatique du nerf facial?
A- Contractez le front et élevez les sourcils
B- Contractez les muscles platysma (cou)
C- froncer les sourcils
D- Ouvrir et fermer les yeux
E- Sourire

23 A
50. Transfer Criteria to a Burn Center (All the answers are :(‫ﻣﻌﺎﻳﻴﺮ ﺍﻹﺣﺎﻟﺔ ﺇﻟﻰ ﻣﺮﻛﺰ ﺍﻟﺤﺮﻭﻕ )ﻛﻞ ﺍﻷﺟﻮﺑﺔ ﺻﺤﻴﺤﺔ ﻣﺎ ﻋﺪﺍ‬
right except one): ‫ ﻣﻦ‬%10 ‫ ﺣﺮﻭﻕ ﺫﺍﺕ ﺳﻤﺎﻛﺔ ﺟﺰﺋﻴﺔ ﺗﻐﻄﻲ ﺃﻗﻞ ﻣﻦ‬-A
a. Partial thickness burns less than 10% total body .‫ﻣﺴﺎﺣﺔ ﺳﻄﺢ ﺍﻟﺠﺴﻢ ﺍﻟﻜﻠﻲ‬
surface area ‫ ﺍﻷﻋﻀﺎء‬،‫ ﺍﻟﻘﺪﻣﻴﻦ‬،‫ ﺍﻟﻴﺪﻳﻦ‬،‫ ﺍﻟﺤﺮﻭﻕ ﺍﻟﺘﻲ ﺗﺸﻤﻞ ﺍﻟﻮﺟﻪ‬-B
b. Burns that involve the face, hands, feet, genitalia, .‫ ﺍﻟﻌﺠﺎﻥ ﺃﻭ ﺍﻟﻤﻔﺎﺻﻞ ﺍﻟﺮﺋﻴﺴﻴﺔ‬،‫ﺍﻟﺘﻨﺎﺳﻠﻴﺔ‬
perineum or major joints .‫ ﺗﺸﻤﻞ ﺍﻟﺼﻮﺍﻋﻖ‬،‫ ﺣﺮﻭﻕ ﻛﻬﺮﺑﺎﺋﻴﺔ‬-C
c. Electrical burns, including lightening injury ‫ ﻣﻦ ﻣﺴﺎﺣﺔ ﺳﻄﺢ‬%25 ‫ ﺣﺮﻭﻕ ﺩﺭﺟﺔ ﺛﺎﻟﺜﺔ – ﺗﻐﻄﻲ‬-D
d. Third degree burns – evaluated at 25% of the total .‫ﺍﻟﺠﺴﻢ ﺍﻟﻜﻠﻲ‬
body surface area

Les critères de transfert vers un centre de traitement des A


brûlures (toutes les réponses sont exactes sauf une):
A- Brûlûres d’épaisseur partielle moins de 10% de la
surface totale corporelle
B- Brûlures touchant le visage, les mains, les pieds,
les organes génitaux, le périné ou les articulations
majeures
C- Brûlures électriques, y compris blessures dues à la
foudre
D- Brûlûres de 3ème degré - prenant 25% de la surface
corporelle
51. meningeal stiffness in cranial trauma: :‫ﺍﻟﺘﺼﻠﺐ ﺍﻟﺴﺤﺎﺋﻲ ﻋﻨﺪ ﺍﻟﻤﺼﺎﺑﻴﻦ ﺑﻜﺪﻣﺎﺕ ﺑﺎﻟﺠﻤﺠﻤﺔ‬
A. is always caused by meningee infection .‫ ﻳﺤﺪﺙ ﺩﺍﺋﻤﺎ ﺑﺘﺤﺮﻳﺾ ﻣﻦ ﺍﻟﺘﻬﺎﺏ ﺳﺤﺎﺋﻲ‬-A
B. must be searched for on the scene of the .‫ ﻳﺠﺐ ﺍﻟﺒﺤﺚ ﻋﻦ ﻣﻜﺎﻥ ﺍﻟﺤﺎﺩﺙ‬-B
accident .‫ ﺿﺮﻭﺭﺓ ﺇﺟﺮﺍء ﺑﺰﻝ ﻗﻄﻨﻲ ﻋﺎﺟﻞ‬-C
C. requires urgent lumbar puncture .‫ ﻳﺘﻄﻮﺭ ﺑﺸﻜﻞ ﺇﻳﺠﺎﺑﻲ ﺑﻄﺮﻳﻘﺔ ﻋﻔﻮﻳﺔ‬-D
D. to evolve favorably in a spontaneous way ‫ ﻛﻞ ﺍﻹﺟﺎﺑﺎﺕ ﺻﺤﻴﺤﺔ‬-E
E. All is correct
D
une raideur méningée chez les traumatisés craniens:
A. est toujous provoqué par une infection méningée
B. doit être recherchée au lieu de l’accident
C. necessite une ponction lombaire urgente
D. peur évoluer favorablement spontanement
E. tous sont corrects
52. Early complications of cranial trauma are all of the :‫ﺍﻟﻤﻀﺎﻋﻔﺎﺕ ﺍﻟﻤﺒﻜﺮﺓ ﻻﺭﺗﺠﺎﺝ ﺍﻟﺪﻣﺎﻍ ﻫﻲ ﻛﻞ ﻣﺎ ﻳﻠﻲ ﻣﺎ ﻋﺪﺍ؟‬
following except: .‫ ﻭﺭﻡ ﺩﻣﻮﻱ ﺧﺎﺭﺝ ﺍﻟﺠﺎﻓﻴﺔ‬-A
A. extra dural hematoma .‫ ﻣﻮﻩ ﺍﻟﺮﺃﺱ‬-B
B. hydrocephalus .‫ ﺍﺷﺘﺒﺎﻙ ﺩﻣﺎﻏﻲ‬-C
C. cerebral herniation .‫ ﻧﺎﺳﻮﺭ ﺍﻟﺴﺎﺋﻞ ﺍﻟﺪﻣﺎﻏﻲ ﺍﻟﺸﻮﻛﻲ‬-D
D. LCR fistula ‫ ﻛﻞ ﺍﻹﺟﺎﺑﺎﺕ ﺻﺤﻴﺤﺔ‬-E
E. All are correct
B
les complications précoces d'un traumatisme cranien sont
tous les suivants sauf :
A. hématome extra dural
B. hydrocéphalie
C. engagement cérébral
D. fistule LCR
E. tous sont corrects

24 A
53. The risk of surgical site infection is increased with: :‫ﻳﺰﺩﺍﺩ ﺧﻄﺮ ﺍﻹﺻﺎﺑﺔ ﺍﻟﺠﺮﺛﻮﻣﻴﺔ ﻓﻲ ﻣﻮﻗﻊ ﺍﻟﺠﺮﺍﺣﺔ ﻣﻊ‬
A. Diabetes .‫ ﺩﺍء ﺍﻟﺴﻜﺮﻱ‬-A
B. Inadequate vaccination status of the patient .‫ ﺍﻟﺘﻠﻘﻴﺢ ﻏﻴﺮ ﻛﺎﻑ ﻟﻠﻤﺮﻳﺾ‬-B
C. Asthma .‫ ﺍﻟﺮﺑﻮ‬-C
D. All of the above .‫ ﻛﻞ ﺍﻷﺟﻮﺑﺔ ﺃﻋﻼﻩ ﺻﺤﻴﺤﺔ‬-D
E. Only A and B are true .‫ ﺻﺤﻴﺤﺘﺎﻥ‬B‫ ﻭ‬A ‫ ﺍﻹﺟﺎﺑﺘﺎﻥ‬-E
A
Le risque d'infection du site opératoire est augmenté avec:
A. Diabète
B. Statut de vaccination inadéquat du patient
C. l'asthme
D. Tout ce qui précède
E. Seuls A et B sont vrais
54. Choose the FALSE answer :‫ﺇﺧﺘﺮ ﺍﻹﺟﺎﺑﺔ ﺍﻟﺨﻄﺄ‬
A. Hypovolemic shock is associated with low .‫ ﺻﺪﻣﺔ ﻧﻘﺺ ﺣﺠﻢ ﺍﻟﺪﻡ ﺗﺘﺮﺍﻓﻖ ﻣﻊ ﺍﻧﺨﻔﺎﺽ ﻧﺘﺎﺝ ﺍﻟﻘﻠﺐ‬-A
cardiac output ‫ ﺍﺣﺘﺸﺎء‬،‫ ﺍﻟﺼﺪﻣﺔ ﺫﺍﺕ ﺍﻟﻤﻨﺸﺄ ﺍﻟﻘﻠﺒﻲ ﺗﺘﺮﺍﻓﻖ ﻣﻊ ﻻﻧﻈﻤﻴﺔ‬-B
B. Cardiogenic shock is associated with .‫ ﻭﺍﻟﺘﻬﺎﺏ ﻋﻀﻠﺔ ﺍﻟﻘﻠﺐ‬،‫ﻋﻀﻠﺔ ﺍﻟﻘﻠﺐ‬
arrhythmia, myocardial infarction, and ‫ ﺍﻟﺼﺪﻣﺔ ﺍﻟﺘﺄﻗﻴﺔ ﺗﻌﺎﻟﺞ ﻓﻮﺭﺍ ﺑﺎﻟﺘﻠﻘﻴﻢ ﺍﻟﻮﺭﻳﺪﻱ ﻟﻠﻔﻨﻴﻞ‬-C
myocarditis .‫ﺇﻓﺮﻳﻦ‬
C. Anaphylactic shock is treated immediately ‫ ﺍﻟﺼﺪﻣﺔ ﺍﻻﻧﺴﺪﺍﺩﻳﺔ ﺧﺎﺭﺝ ﺍﻟﻘﻠﺐ ﺗﺘﺮﺍﻓﻖ ﻣﻊ ﺍﻧﺪﺣﺎﺱ‬-D
with Phenylephrine IV drip. .‫ ﻭﺍﻧﺼﻤﺎﻡ ﺭﺋﻮﻱ‬،‫ ﺍﺳﺘﺮﻭﺍﺡ ﺍﻟﺼﺪﺭ‬،‫ﻗﻠﺒﻲ‬
D. Extracardiac obstructive shock is associated
with cardiac tamponade, pneumothorax, and
pulmonary emboli.
C
Choisissez la réponse FAUSSE
A- Le choc hypovolémique est associé à un faible
débit cardiaque
B- Le choc cardiogénique est associé à une arythmie,
un infarctus du myocarde et une myocardite
C- Le choc anaphylactique est traité immédiatement
par une perfusion intraveineuse de phényléphrine
IV.
D- Un choc obstructif extracardiaque est associé à
une tamponnade cardiaque, à un pneumothorax
et à des emboles pulmonaires.
55. Anaphylactic shock is due to what type of sensitivity: :‫ﻋﻦ ﺃﻱ ﻧﻮﻉ ﻣﻦ ﺍﻟﺤﺴﺎﺳﻴﺔ ﺗﻨﺘﺞ ﺍﻟﺼﺪﻣﺔ ﺍﻟﺘﺄﻗﻴﺔ؟‬
A. Type I, immediate Hypersensitivity Disorder .‫ ﺍﺿﻄﺮﺍﺏ ﻓﺮﻁ ﺗﺤﺴﺲ ﻓﻮﺭﻱ‬،I ‫ ﻧﻤﻂ‬-A
B. Type II, Antibody-mediated Disorder .‫ ﺍﺿﻄﺮﺍﺏ ﻣﺘﻮﺍﺳﻂ ﺍﻷﺿﺪﺍﺩ‬،II‫ ﻧﻤﻂ‬-B
C. Type III, Immune Complex-Mediated .‫ ﺍﺿﻄﺮﺍﺏ ﻣﺘﻮﺍﺳﻂ ﺑﻤﻌﻘﺪ ﻣﻨﺎﻋﻲ‬،III‫ ﻧﻤﻂ‬-C
Disorder .‫ ﺍﺿﻄﺮﺍﺏ ﻓﺮﻁ ﺗﺤﺴﺲ ﻣﺘﻮﺍﺳﻂ ﺑﺎﻟﺨﻼﻳﺎ‬،IV‫ ﻧﻤﻂ‬-D
D. Type IV, Cell Mediated Hypersensitivity
Disorder
A
Le choc anaphylactique est dû à quel type de sensibilité:
A- Trouble d'hypersensibilité immédiate de type I
B- Trouble à médiation anticorps de type II
C- Trouble à médiation complexe immunitaire de
type III
D- Trouble d'hypersensibilité à médiation cellulaire
de type IV

25 A
56. You are asked to see a 65 year old unconscious patient in ‫ ﻋﺎﻣﺎ ﻏﺎﺋﺐ ﻋﻦ ﺍﻟﻮﻋﻲ ﻓﻲ‬65 ‫ﻁﻠﺐ ﻣﻨﻚ ﻣﻌﺎﻳﻨﺔ ﻣﺮﻳﺾ ﻋﻤﺮﻩ‬
the Emergency Department. He was brought by ‫ ﻳﺒﺪﻭ ﺷﺎﺣﺒﺎ‬.‫ ﻟﻘﺪ ﺃﺣﻀﺮ ﺑﻮﺍﺳﻄﺔ ﺳﻴﺎﺭﺓ ﺍﻹﺳﻌﺎﻑ‬.‫ﻗﺴﻢ ﺍﻟﻄﻮﺍﺭﺉ‬
ambulance. He looks pale and his peripheries are cold and 140 ‫ ﺗﺒﻴﻦ ﻋﻨﺪ ﺍﻟﻔﺤﺺ ﺃﻥ ﻣﻌﺪﻝ ﺍﻟﻨﺒﺾ‬.‫ﻭﺃﻁﺮﺍﻓﻪ ﺑﺎﺭﺩﺓ ﻭﻧﺪﻳﺔ‬
clammy. On examination his pulse rate is 140 Bpm, BP ‫ ﻓﻲ‬%96 ‫ ﻭﺇﺷﺒﺎﻉ ﺍﻷﻭﻛﺴﺠﻴﻦ‬45/70 ‫ ﺿﻐﻂ ﺍﻟﺪﻡ‬،‫ﻧﺒﻀﺔ ﺑﺎﻟﺪﻗﻴﻘﺔ‬
70/45 and Sat 96% on Room Air. His central venous ‫ ﺳﻢ ﻣﻦ‬١ ‫ ﺍﻟﻀﻐﻂ ﺍﻟﻮﺭﻳﺪﻱ ﺍﻟﻤﺮﻛﺰﻱ ﻫﻮ‬.‫ﺩﺭﺟﺔ ﺣﺮﺍﺭﺓ ﺍﻟﻐﺮﻓﺔ‬
pressure CVP is 1 Cm of H2O. the MOST LIKELY :‫ ﺇﻥ ﺍﻟﺘﺸﺨﻴﺺ ﺍﻷﻛﺜﺮ ﺗﺮﺟﻴﺤﺎ ﻫﻮ‬.H2O
diagnosis is: .‫ ﺻﺩﻣﺔ ﺇﻧﺗﺎﻧﻳﺔ‬-
A. Septic shock .‫ ﺻﺩﻣﺔ ﻗﻠﺑﻳﺔ‬-
B. Cardiogenic shock .‫ ﺻﺩﻣﺔ ﻧﻘﺹ ﺣﺟﻡ ﺍﻟﺩﻡ‬-
C. Hypovolemic shock .‫ ﺻﺩﻣﺔ ﺗﺄﻗﻳﺔ‬-
D. Anaphylactic shock .‫ ﻛﻝ ﺍﻷﺟﻭﺑﺔ ﺃﻋﻼﻩ ﻏﻳﺭ ﺻﺣﻳﺣﺔ‬-
E. None of the above
C
On vous demande de voir un patient inconscient de 65
ans aux urgences. Il a été amené en ambulance. Il a l'air
pâle et ses extrémités sont froides et moites. À l'examen,
son pouls est de 140 /mn, TA 70/45 saturation d’O2 96% à
l'air ambiant. Sa pression veineuse centrale CVP est de 1
Cm de H2O. le diagnostic le plus probable est:
A- Choc septique
B- Choc cardiogénique
C- Choc hypovolémique
D- Choc anaphylactique
E- Aucune de ces réponses
57. What is the primary cause of total parenteral nutrition ‫ﻣﺎ ﻫﻮ ﺍﻟﺴﺒﺐ ﺍﻟﺮﺋﻴﺴﻲ ﻹﻧﺘﺎﻥ ﺍﻟﺪﻡ ﺍﻟﻤﺘﻌﻠﻖ ﺑﺎﻟﺘﻐﺬﻳﺔ ﺍﻟﻮﺭﻳﺪﻳﺔ‬
related sepsis? :‫ﺍﻟﺸﺎﻣﻠﺔ؟‬
A. Fibrin sheath formation. .‫ ﺗﺸﻜﻞ ﺍﻟﻐﻤﺪ ﺍﻟﻠﻴﻔﻲ‬-A
B. Intrinsic contamination of nutrient solution. .‫ ﺗﻠﻮﺙ ﺩﺍﺧﻠﻲ ﺍﻟﻤﻨﺸﺄ ﻟﻠﻤﺤﻠﻮﻝ ﺍﻟﻐﺬﺍﺋﻲ‬-B
C. Migration of micro-organisms along the .‫ ﻫﺠﺮﺓ ﺍﻟﻌﻀﻮﻳﺎﺕ ﺍﻟﺪﻗﻴﻘﺔ ﻋﺒﺮ ﺍﻟﻘﺜﻄﺮﺓ‬-C
catheter. .‫ ﺑﺬﻭﺭ ﻣﻦ ﻣﺼﺪﺭ ﺇﻧﺘﺎﻧﻲ ﺁﺧﺮ ﻓﻲ ﺍﻟﺠﺴﻢ‬-D
D. Seeding from other infection source in the
body.
C
Quelle est la principale cause de la sepsie liée à
l’alimentation parentérale totale?
A- Formation d’une gaine de fibrine.
B- Contamination intrinsèque de la solution
nutritive.
C- Migration de micro-organismes le long du
cathéter.
D- Point de départ d'une autre source
d'infection dans le corps.

26 A
58. An 18-year-old male patient has been in the clinic for ‫ ﻟﻘﺪ ﻋﻮﻟﺞ‬.‫ ﻋﺎﻣﺎ ﻳﻌﺎﻧﻲ ﻣﻦ ﺇﻓﺮﺍﺯﺍﺕ ﺇﺣﻠﻴﻠﻴﺔ‬18 ‫ﻣﺮﻳﺾ ﻋﻤﺮﻩ‬
urethral discharge. He is treated with IM ceftriaxone, but ‫ ﻭﻟﻜﻦ ﻟﻢ ﺗﺤﻞ ﻣﺸﻜﻠﺔ ﺍﻹﻓﺮﺍﺯﺍﺕ ﺑﺎﻟﺮﻏﻢ ﻣﻦ‬،‫ﺑﺴﻔﺘﺮﻳﺎﻛﺴﻮﻥ ﺣﻘﻨﺎ ﺑﺎﻟﻌﻀﻞ‬
the discharge has not resolved and the culture has ‫ ﺃﻱ ﻣﻦ ﺍﻟﺘﺎﻟﻲ ﻫﻮ ﺍﻟﻌﺎﻣﻞ ﺍﻟﻤﺴﺒﺐ‬.‫ﺃﻥ ﻧﺘﻴﺠﺔ ﺍﻟﺰﺭﻉ ﺃﺛﺒﺘﺖ ﻋﺪﻡ ﺍﻟﻨﻤﻮ‬
returned as no growth. Which of the following is the most :‫ﺍﻷﻛﺜﺮ ﺗﺮﺟﻴﺤﺎ ﻟﻬﺬﻩ ﺍﻹﺻﺎﺑﺔ؟‬
likely etiologie agent to cause this infection? .‫ ﻣﻜﻮﺭﺍﺕ ﺑﻨﻴﺔ ﻣﻘﺎﻭﻣﺔ ﻟﻠﺴﻔﺘﺮﻳﺎﻛﺴﻮﻥ‬-A
A- Ceftriaxone-resistant gonococci .‫ ﺍﻟﻤﺘﺪﺛﺮﺓ ﺍﻟﺒﺒﻐﺎﺋﻴﺔ‬-B
B- Chlamydia psittaci .‫ ﺍﻟﻤﺘﺪﺛﺮﺓ ﺍﻟﺤﺜﺮﻳﺔ‬-C
C- Chlamydia trachomatis .‫ ﻓﻴﺮﻭﺱ ﺍﻟﺤﻸ ﺍﻟﺒﺴﻴﻂ‬-D
.‫ ﺍﻟﻤﺘﺪﺛﺮﺓ ﺍﻟﺮﺋﻮﻳﺔ‬-E
D- Herpes simplex virus
E- Chlamydia pneumonia
C
Un patient de sexe masculin âgé de 18 ans s'est rendu à la
clinique pour un écoulement de l'urètre. Il est traité avec
de la ceftriaxone par voie IM, mais la décharge n'a pas été
résolue et la culture est revenue sans croissance. Lequel
des agents étiologiques, parmi les suivantes, est le plus
susceptible de causer cette infection?
A. Gonocoques résistants à la ceftriaxone
B. Chlamydia psittaci
C. Chlamydia Trachomatis
D. Virus de l'herpès simplex
E. Chlamydia pneumonia
59. A 27-year-old man has fever, macular rash, and ‫ ﻁﻔﺢ ﺟﻠﺪﻱ ﻭﺗﻀﺨﻢ ﺍﻟﻌﻘﺪ‬،‫ ﻋﺎﻣﺎ ﻳﻌﺎﻧﻲ ﻣﻦ ﺍﺭﺗﻔﺎﻉ ﺣﺮﺍﺭﺓ‬27 ‫ﺭﺟﻞ ﻋﻤﺮﻩ‬
lymphadenopathy. He had unprotected sex with a male ‫ ﻟﻘﺪ ﻣﺎﺭﺱ ﺍﻟﺠﻨﺲ ﺑﻄﺮﻳﻘﺔ ﻏﻴﺮ ﻣﺄﻣﻮﻧﺔ ﻣﻊ ﺷﺮﻳﻚ ﺫﻛﺮ ﻗﺒﻞ‬.‫ﺍﻟﻠﻤﻔﻴﺔ‬
partner 2 weeks before the onset of these symptoms and ‫ﺇﺳﺒﻮﻋﻴﻦ ﻣﻦ ﺑﺪء ﻅﻬﻮﺭ ﻫﺬﻩ ﺍﻷﻋﺮﺍﺽ ﻭﻟﻘﺪ ﻋﻠﻢ ﻟﻠﺘﻮ ﺃﻥ ﺍﻟﺸﺮﻳﻚ ﻣﺼﺎﺏ‬
has just learned that the partner is, infected with human ‫ ﺍﻟﺴﺮﻳﻊ ﻟﻠﻤﺮﻳﺾ‬HIV ‫ ﻓﺤﺺ‬.(HIV) ‫ﺑﻔﻴﺮﻭﺱ ﻧﻘﺺ ﺍﻟﻤﻨﺎﻋﺔ ﺍﻟﻤﻜﺘﺴﺒﺔ‬
immunodeficiency virus (HIV). The patient’s rapid HIV :‫( ﻋﻨﺪ ﻫﺬﺍ ﺍﻟﻤﺮﻳﺾ؟‬HIV) ‫ ﻣﺎ ﻫﻮ ﺃﻓﻀﻞ ﺇﺧﺘﺒﺎﺭ ﻟﺘﻘﻴﻴﻢ ﻋﺪﻭﻯ‬.‫ﻛﺎﻥ ﺳﻠﺒﻴﺎ‬
test is negative. What is the best test to evaluate this .(ELISA) (HIV) ‫ ﻣﻘﺎﻳﺴﺔ ﺍﻟﻤﻤﺘﺰ ﺍﻟﻤﻨﺎﻋﻲ ﺍﻟﻤﺮﺗﺒﻂ ﺑﺄﻧﺰﻳﻢ‬-A
patient for HIV infection? HIV RNA ‫ ﺗﻔﺎﻋﻞ ﺳﻠﺴﻠﺔ ﺍﻟﺒﻮﻟﻴﻤﻴﺮﺍﺯ‬-B
A- HIV enzyme-linked immunosorbent assay .‫ ﺇﺧﺘﺒﺎﺭ ﻭﺳﺘﺮﻥ‬-C
(ELISA) .120 ‫ ﺇﺧﺘﺒﺎﺭ ﺇﻟﻴﺰﺍ ﻏﻠﻴﻜﻮﺑﺮﻭﺗﻴﻦ‬-D
B- PCR for HIV RNA HIV DNA ‫ ﺗﻔﺎﻋﻞ ﺳﻠﺴﻠﺔ ﺍﻟﺒﻮﻟﻴﻤﻴﺮﺍﺯ‬-E
C- Eastern blot testing
D- Glycoprotein 120 ELISA testing
E- PCR for HIV DNA
B
Un homme de 27 ans a de la fièvre, une éruption
maculaire et une lymphadénopathie. Il a eu des relations
sexuelles non protégées avec un partenaire masculin deux
semaines avant l'apparition de ces symptômes et vient
d'apprendre que son partenaire est infecté par le virus de
l'immunodéficience humaine (VIH). Le test VIH rapide du
patient est négatif. Quel est le meilleur test pour évaluer
l’infection par le VIH chez ce patient?
A. Essai immuno-absorbant lié à une enzyme VIH
(ELISA)
B. PCR pour l'ARN du VIH
C. Test de transfert Eastern
D. Test ELISA Glycoprotéine 120
E. PCR pour l'ADN du VIH

27 A
60. A 49-year-old man has a history of athlete’s foot but is ‫ ﻋﺎﻣﺎ ﻳﻌﺎﻧﻲ ﻣﻦ ﺩﺍء ﺍﻟﻘﺪﻡ ﺍﻟﺮﻳﺎﺿﻲ ﻭﻫﻮ ﻳﺘﻤﺘﻊ ﺑﺼﺤﺔ‬49 ‫ﺭﺟﻞ ﻋﻤﺮﻩ‬
otherwise healthy when he develops sudden onset of ‫ ﺗﺒﻴﻦ ﻣﻦ‬.‫ ﻓﺠﺄﺓ ﺑﺪﺃ ﻳﺸﻌﺮ ﺑﺤﺮﺍﺭﺓ ﻭﺃﻟﻢ ﻓﻲ ﻗﺪﻣﻪ ﻭﺳﺎﻗﻪ ﺍﻷﻳﻤﻦ‬،‫ﺟﻴﺪﺓ‬
fever and pain in the right foot and leg. On physical ‫ﺍﻟﻔﺤﺺ ﺍﻟﺴﺮﻳﺮﻱ ﺃﻥ ﻗﺪﻣﻪ ﻭﺳﺎﻗﻪ ﺫﺍﺕ ﻟﻮﻥ ﺃﺣﻤﺮ ﻧﺎﺭﻱ ﻣﻊ ﻫﺎﻣﺶ ﺫﺍﺕ‬
examination, the foot and leg are fiery red with a well- ‫ ﻫﻨﺎﻙ ﺗﻀﺨﻢ ﻣﺆﻟﻢ ﻓﻲ ﺍﻟﻌﻘﺪ ﺍﻟﻠﻤﻔﻴﺔ‬.‫ﻗﺴﺎﻭﺓ ﻭﺍﺿﺤﺔ ﻭﺍﻟﺬﻱ ﻳﺘﻄﻮﺭ ﺑﺴﺮﻋﺔ‬
defined indurated margin that appears to be rapidly :‫ ﻣﺎ ﻫﻮ ﺍﻟﻌﺎﻣﻞ ﺍﻟﻤﺴﺒﺐ ﺍﻷﻛﺜﺮ ﺗﺮﺟﻴﺤﺎ ﻟﻬﺬﻩ ﺍﻹﺻﺎﺑﺔ؟‬.‫ﺍﻹﺭﺑﻴﺔ‬
advancing. There is tender inguinal lymphadenopathy. .‫ ﺍﻟﻤﻜﻮﺭﺍﺕ ﺍﻟﻌﻨﻘﻮﺩﻳﺔ ﺍﻟﺒﺸﺮﻭﻳﺔ‬-A
Which organism is the most likely cause of this infection? .‫ ﺳﻌﻔﺔ ﺍﻟﻘﺪﻡ‬-B
.‫ ﺍﻟﻤﻜﻮﺭﺍﺕ ﺍﻟﻌﻘﺪﻳﺔ ﺍﻟﻤﻘﻴﺤﺔ‬-C
A- Staphylococcus epidermidis
.‫ ﺇﺻﺎﺑﺔ ﺟﺮﺛﻮﻣﻴﺔ ﻻﻫﻮﺍﺋﻴﺔ ﻣﺨﺘﻠﻄﺔ‬-D
B- Tinea pedids
.‫ﺃﻟﻔﺎ‬-‫ ﻣﻜﻮﺭﺍﺕ ﻋﻘﺪﻳﺔ ﺍﻧﺤﻼﻟﻴﺔ‬-E
C- Streptococcus pyogenes
D- Mixed anaerobic infection
E- Alpha-hemolytic streptococci
C
Un homme de 49 ans a des antécédents de pied d’athlète,
par ailleurs en bonne il présente subitement une fièvre et
une douleur au pied et à la jambe droite. À l'examen
physique, le pied et la jambe sont rouge ardent avec une
marge indurée bien définie qui semble progresser
rapidement. Il y a une adénopathie inguinale
douloureuse. Quel organisme est la cause la plus probable
de cette infection?
A- Staphylococcus epidermidis
B- Tinea Pedids
C- Streptococcus pyogenes
D- Infection anaérobie mixte
E- Streptocoques alpha-hémolytiques
61. An 18-year-old woman presents with a 2-day history of ‫ ﻣﺎ‬.‫ ﻋﺎﻣﺎ ﺗﻌﺎﻧﻲ ﻣﻦ ﺍﻟﺘﻬﺎﺏ ﺣﻠﻖ ﻣﻨﺬ ﻳﻮﻣﻴﻦ‬18 ‫ﺍﻣﺮﺃﺓ ﻋﻤﺮﻫﺎ‬
sore throat. Which of the following constellation of ‫ﻫﻲ ﺍﻷﻋﺮﺍﺽ ﻭﺍﻹﺷﺎﺭﺍﺕ ﺍﻟﺘﻲ ﺗﻌﺘﺒﺮ ﻣﻦ ﻣﻜﻮﻧﺎﺕ ﺍﻟﺘﻬﺎﺏ‬
symptoms and signs is most consistent with group-A
:‫؟‬A ‫ﺍﻟﺒﻠﻌﻮﻡ ﺑﻤﺠﻤﻮﻋﺔ ﺍﻟﻤﻜﻮﺭﺍﺕ ﺍﻟﻌﻘﺪﻳﺔ‬
streptococcal pharyngitis ?
A- Fever, anorexia, dysphagia and hoarseness .‫ ﻋﺴﺮ ﺑﻠﻊ ﻭﺑﺤﺔ‬،‫ ﻓﻘﺪﺍﻥ ﺷﻬﻴﺔ‬،‫ ﺣﺮﺍﺭﺓ‬-A
B- Fever, runny nose, cough, myalgia and poor ‫ ﺃﻟﻢ ﻋﻀﻠﻲ ﻭﺿﻌﻒ‬،‫ ﺳﻌﺎﻝ‬،‫ ﺳﻴﻼﻥ ﺃﻧﻔﻲ‬،‫ ﺣﺮﺍﺭﺓ‬-B
appetite .‫ﺷﻬﻴﺔ‬
C- Fever, no cough, tonsillar exudates and tender ‫ ﺃﻓﺮﺍﺯﺍﺕ ﻣﻦ ﺍﻟﻠﻮﺯﺗﻴﻦ‬،‫ ﺑﺪﻭﻥ ﺳﻌﺎﻝ‬،‫ ﺣﺮﺍﺭﺓ‬-C
anterior cervical lymphadenopathy .‫ﻭﺗﻀﺨﻢ ﻋﻘﺪ ﻟﻤﻔﻴﺔ ﺭﻗﺒﻴﺔ ﺃﻣﺎﻣﻴﺔ ﻣﺆﻟﻢ‬
D- Fever, cough, pharyngeal erythema and .‫ ﺍﺣﻤﺮﺍﺭ ﺍﻟﺒﻠﻌﻮﻡ ﻭﻋﺴﺮ ﺑﻠﻊ‬،‫ ﺳﻌﺎﻝ‬،‫ ﺣﺮﺍﺭﺓ‬-D
dysphagia
.‫ ﻋﺴﺮ ﺑﻠﻊ ﻭﺗﻮﺭﻡ ﻓﻲ ﺍﻟﻌﻨﻖ‬،‫ ﺿﺰﺯ‬،‫ ﺣﺮﺍﺭﺓ‬-E
E- Fever, trismus, dysphagia and neck swelling
C
Une femme de 18 ans présente une histoire de maux de
gorge depuis 2 jours. Lequel des symptômes et des signes
suivants est le plus compatible avec la pharyngite à
streptocoque du groupe A?
A. Fièvre, anorexie, dysphagie et enrouement
B. Fièvre, nez qui coule, toux, myalgie et manque
d'appétit
C. Fièvre, pas de toux, exsudats des amygdales et
adénopathies cervicales antérieures douloureuses
D. Fièvre, toux, érythème pharyngé et dysphagie
E. Fièvre, trismus, dysphagie et gonflement du cou

28 A
62. A 65-year-old man with mild congestive heart failure is ‫ ﻗﺪ ﺗﻢ ﻭﺿﻌﻪ ﻋﻠﻰ‬،‫ ﻋﺎﻣﺎ ﻳﻌﺎﻧﻲ ﻣﻦ ﻗﺼﻮﺭ ﻗﻠﺐ ﺍﺣﺘﻘﺎﻧﻲ‬65 ‫ﺭﺟﻞ ﻋﻤﺮﻩ‬
scheduled to receive total hip replacement. He has no ‫ﻫﻮ ﻻ ﻳﻌﺎﻧﻲ ﻣﻦ ﺃﻣﺮﺍﺽ ﻛﺎﻣﻨﺔ ﻭﻻ‬.‫ﺍﻟﺠﺪﻭﻝ ﻹﺟﺮﺍء ﺍﺳﺘﺒﺪﺍﻝ ﻛﺎﻣﻞ ﻟﻠﻮﺭﻙ‬
other underlying diseases and no history of hypertension, ‫ ﺃﻱ ﻣﻦ ﺍﻟﺘﺎﻟﻲ‬.‫ ﺃﻭ ﺍﺿﻄﺮﺍﺏ ﻧﺰﻓﻲ‬،‫ ﻋﻤﻞ ﺟﺮﺍﺣﻲ ﺣﺪﻳﺚ‬،‫ﺍﺭﺗﻔﺎﻉ ﺿﻐﻂ‬
recent surgery, or bleeding disorder. Which of the :‫ﻫﻮﺃﻓﻀﻞ ﺇﺟﺮﺍء ﻟﻠﻮﻗﺎﻳﺔ ﻣﻦ ﺍﻧﺼﻤﺎﻡ ﺭﺋﻮﻱ ﻋﻨﺪ ﻫﺬﺍ ﺍﻟﻤﺮﻳﺾ؟‬
following is the best approach to prevention of .‫ﻳﻮﻡ‬/‫ﻣﻠﻎ‬75 ‫ ﺃﺳﺒﻴﺮﻳﻦ‬-A
pulmonary embolus in this patient ? .‫ﻳﻮﻡ‬/325 ‫ ﺃﺳﺒﻴﺮﻳﻦ‬-B
A- Aspirin 75mg/d .‫ ﻣﻠﻎ ﺗﺤﺖ ﺍﻟﺠﻠﺪ ﻣﺮﺗﻴﻦ ﻳﻮﻣﻴﺎ‬30 ‫ ﺇﻧﻮﻛﺴﺎﺑﺎﺭﻳﻦ‬-C
B- Aspirin 325mg/d .‫ ﺍﻟﺘﻤﺸﻲ ﺍﻟﻤﺒﻜﺮ‬-D
C- Enoxaparin 30mg subcutaneously bid .‫ ﺟﻮﺍﺭﺏ ﺿﻐﻂ ﻣﺮﻧﺔ‬-E
D- Early ambulation
E- Craded compression elastic stocking
C
Un homme de 65 ans souffrant d'insuffisance cardiaque
congestive légère devrait subir une prothèse totale de la
hanche. Il n'a aucune autre maladie sous-jacente et aucun
antécédent d'hypertension, de chirurgie récente ou de
trouble de la coagulation. Quelle est la meilleure approche
de prévention de l'embolie pulmonaire chez ce patient?
A. Aspirine 75 mg / j
B. Aspirine 325 mg / j
C. Enoxaparine 30 mg en sous cutanée chaque 12h
D. ambulation précoce
E. Bas élastique de compression
63. A 60-year-old obese man complains of excessive daytime ‫ ﻫﻮ ﻳﺘﻤﺘﻊ ﺑﺼﺤﺔ‬.‫ ﻋﺎﻣﺎ ﻳﺸﻜﻮ ﻣﻦ ﻧﻮﻡ ﻧﻬﺎﺭﻱ ﻣﺘﺰﺍﻳﺪ‬60 ‫ﺭﺟﻞ ﺑﺪﻳﻦ ﻋﻤﺮﻩ‬
sleepiness. He has been in good health except for mild ‫ ﻳﺸﺮﺏ ﻛﺤﻮﻝ ﺑﺸﻜﻞ‬.‫ﺟﻴﺪﺓ ﺑﺎﺳﺘﺜﻨﺎء ﺃﻧﻪ ﻳﻌﺎﻧﻲ ﻣﻦ ﺍﺭﺗﻔﺎﻉ ﺿﻐﻂ ﺧﻔﻴﻒ‬
hypertension. He drinks alcohol in moderation. The ‫ ﺍﻟﻔﺤﺺ‬.‫ ﻟﻘﺪ ﺃﻓﺎﺩﺕ ﺯﻭﺟﺘﻪ ﺍﻧﻪ ﻳﺸﺨﺮ ﻟﻴﻼ ﻭﻳﺴﺘﻴﻘﻆ ﺑﺸﻜﻞ ﻣﺘﻜﺮﺭ‬.‫ﻣﺘﻮﺳﻂ‬
patient’s wife states that he snores at night and awakens :‫ ﺃﻱ ﻣﻦ ﺍﻟﺪﺭﺍﺳﺎﺕ ﺍﻟﺘﺎﻟﻴﺔ ﻫﻲ ﺍﻷﻧﺴﺐ؟‬.‫ﺍﻟﻔﻤﻮﻱ ﺍﻟﺒﻠﻌﻮﻣﻲ ﻁﺒﻴﻌﻲ‬
frequently. Examination of the oropharynx is normal. .‫ ﺗﺨﻄﻴﻂ ﺩﻣﺎﻏﻲ ﻛﻬﺮﺑﺎﺋﻲ ﻟﺘﻘﻴﻴﻢ ﻧﻤﻂ ﺍﻟﻨﻮﻡ‬-A
Which of the following studies is most appropriate ? .‫ ﻧﻤﻂ ﺍﻟﺘﻬﻮﻳﺔ ﻟﻠﺘﺤﺮﻱ ﻋﻦ ﺗﻮﻗﻒ ﺍﻟﺘﻨﻔﺲ ﺍﻟﻔﺠﺎﺋﻲ ﺃﺛﻨﺎء ﺍﻟﻨﻮﻡ‬-B
A- Electroencephalogram (EEG) to assess sleep .‫ ﺇﺷﺒﺎﻉ ﺍﻷﻭﻛﺴﺠﻴﻦ ﺍﻟﺸﺮﻳﺎﻧﻲ‬-C
patterns .‫ ﺩﺭﺍﺳﺔ ﻋﻀﻼﺕ ﺍﻟﺘﻨﻔﺲ ﺃﺛﻨﺎء ﺍﻟﻨﻮﻡ‬-D
.‫ ﺩﺭﺍﺳﺔ ﺍﻟﻨﻮﻡ‬-E
B- Ventilation pattern to detect apnea
C- Arterial O2 saturation
D- Study of muscles of respiration during sleep
E- Polysomnography

Un homme obèse de 60 ans se plaint de somnolence E


excessive. Il a été en bonne santé, à l'exception d'une
hypertension légère. Il boit de l'alcool avec modération.
L’épouse du patient déclare qu’il ronfle la nuit et se
réveille fréquemment. L'examen de l'oropharynx est
normal. Laquelle des études suivantes est la plus
appropriée?
A. Electroencéphalogramme (EEG) pour évaluer les
paramètres du sommeil
B. Paramètre de ventilation pour détecter l'apnée
C. Saturation artérielle en O2
D. Etude des muscles de la respiration pendant le
sommeil
E. Polysomnographie

29 A
64. A 21-year-old man presents in the emergency room with ‫ ﻋﺎﻣﺎ ﺣﻀﺮ ﺇﻟﻰ ﻏﺮﻓﺔ ﺍﻟﻄﻮﺍﺭﺉ ﻭﻫﻮ ﻳﺸﻜﻮ ﻣﻦ ﺑﺪء‬21 ‫ﺭﺟﻞ ﻋﻤﺮﻩ‬
new onset of slurred speech and right hemiparesis. On ‫ ﻋﻨﺪ ﺍﻟﺘﺴﻤﻊ ﺗﺒﻴﻦ ﻭﺟﻮﺩ‬.‫ﺣﺪﻳﺚ ﻟﻠﺘﻠﻌﺜﻢ ﻭﺷﻠﻞ ﻧﺼﻔﻲ ﻋﻠﻰ ﺍﻟﺠﺎﻧﺐ ﺍﻷﻳﻤﻦ‬
auscultation the patient has a systolic murmur at the ‫ﻧﻔﺨﺔ ﺍﻧﻘﺒﺎﺿﻴﺔ ﻓﻲ ﺍﻟﻤﻨﻄﻘﺔ ﺍﻟﺮﺋﻮﻳﺔ ﻣﻊ ﺩﻣﺪﻣﺔ ﺍﻧﺒﺴﺎﻁﻴﺔ ﻋﻠﻰ ﻁﻮﻝ ﺍﻟﺤﺪﻭﺩ‬
pulmonic region with a diastolic rumble along the left ‫ﻣﺎ‬.‫ ﺻﻮﺕ ﺍﻟﻘﻠﺐ ﺍﻟﺜﺎﻧﻲ ﻫﻮ ﻣﻨﻘﺴﻢ ﻭﺛﺎﺑﺖ ﺑﺎﻟﻨﺴﺒﺔ ﻟﻠﺘﻨﻔﺲ‬.‫ﺍﻟﻘﺼﻴﺔ ﺍﻟﻴﺴﺮﻯ‬
sternal border. The second heart sound is split and fixed :‫ﻫﻮ ﺍﻟﺴﺒﺐ ﺍﻷﻛﺜﺮ ﺗﺮﺟﻴﺤﺎ ﻷﻋﺮﺍﺽ ﺍﻟﻤﺮﻳﺾ؟‬
relative to respiration. What is the likely cause of patient’s .‫ ﻋﻴﺐ ﺍﻟﺤﺎﺟﺰ ﺍﻟﺒﻄﻴﻨﻲ‬-A
symptom ? .‫ ﻋﻴﺐ ﺍﻟﺤﺎﺟﺰ ﺍﻷﺫﻳﻨﻲ‬-B
.‫ ﺍﻟﻘﻨﺎﺓ ﺍﻟﺸﺮﻳﺎﻧﻴﺔ ﺍﻟﺴﺎﻟﻜﺔ‬-C
A- Ventricular septal defect
.‫ ﻗﺼﻮﺭ ﺍﻷﺑﻬﺮ‬-D
B- Atrial septal defect
.‫ ﺗﻀﻴﻖ ﺍﻷﺑﻬﺮ‬-E
C- Patent ductus arteriosus
D- Aortic insufficiency
E- Coarctation of the aorta
B
Un homme de 21 ans se présente à la salle d'urgence avec
l'apparition récente de troubles de la parole et
d'hémiparésie droite. À l'auscultation, le patient a un
souffle systolique dans la région pulmonaire avec un
roulement diastolique le long du bord sternal gauche. Le
deuxième bruit cardiaque est dédoublé et fixe par rapport
à la respiration. Quelle est la cause probable du symptôme
du patient?
A- Défaut septal ventriculaire
B- Anomalie septale auriculaire
C-canal artériel
D- Insuffisance aortique
E- Coarctation de l'aorte
65. An active 78-year-old woman with hypertension presents ‫ ﻋﺎﻣﺎ ﺗﻌﺎﻧﻲ ﻣﻦ ﺍﺭﺗﻔﺎﻉ ﺿﻐﻂ ﺣﻀﺮﺕ ﺑﺤﺎﻟﺔ‬78 ‫ﺍﻣﺮﺃﺓ ﻧﺸﻄﺔ ﻋﻤﺮﻫﺎ‬
with a new left hemiparesis. Cardiac monitoring reveals ‫ ﺃﻅﻬﺮﺕ ﻣﺮﺍﻗﺒﺔ ﺍﻟﻘﻠﺐ ﻭﺟﻮﺩ‬.‫ﺷﻠﻞ ﻧﺼﻔﻲ ﻣﻦ ﺍﻟﺠﺎﻧﺐ ﺍﻷﻳﺴﺮ ﺣﺪﻳﺚ‬
atrial fibrillation. She had been in sinus rhythm 3 months ‫ ﺇﻧﻬﺎ ﺗﺘﻨﺎﻭﻝ‬.‫ ﺃﺷﻬﺮ‬3 ‫ ﻫﻲ ﻛﺎﻧﺖ ﻋﻠﻰ ﺍﻟﻨﻈﻢ ﺍﻟﺠﻴﺒﻲ ﻣﻨﺬ‬.‫ﺭﺟﻔﺎﻥ ﺃﺫﻳﻨﻲ‬
go. She takes a beta-blocker for her blood pressure. Aside ‫ ﺑﺼﺮﻑ ﺍﻟﻨﻈﺮ ﻋﻦ ﺿﻐﻂ ﺍﻟﺪﻡ ﻭﻣﺮﺍﻗﺒﺔ‬.‫ﺣﺎﺟﺐ ﺑﻴﺘﺎ ﻣﻦ ﺃﺟﻞ ﺿﻐﻄﻬﺎ‬
from blood pressure and heart rate control, which of the :‫ ﺃﻱ ﻣﻦ ﺍﻟﺘﺎﻟﻲ ﻫﻮ ﺍﻟﻤﻨﺎﺳﺐ؟‬،‫ﻣﻌﺪﻝ ﺿﺮﺑﺎﺕ ﺍﻟﻘﻠﺐ‬
following is appropriate ? .‫ ﻣﻨﻈﻢ ﺿﺮﺑﺎﺕ ﺍﻟﻘﻠﺐ ﺩﺍﺋﻢ‬-A
A- Permanent pacemaker .‫ ﺗﻴﺎﺭ ﻣﺒﺎﺷﺮ ﻓﻮﺭﻱ ﻟﺘﻘﻮﻳﻢ ﻧﻈﻢ ﺍﻟﻘﻠﺐ‬-B
B- Immediate direct-current cardioversion .‫ ﻣﻠﻎ ﻳﻮﻣﻴﺎ‬81 ‫ ﺃﺳﺒﻴﺮﻳﻦ‬-C
C- Aspirin 81mg daily INR ‫ ﺍﻟﻤﻌﺎﻟﺠﺔ ﺑﻤﻀﺎﺩ ﺗﻜﺪﺱ ﺻﻔﻴﺤﺎﺕ ﻣﻊ ﻭﺍﺭﻓﺎﺭﻳﻦ ﺑﻬﺪﻑ‬-D
D- Antiplatelet therapy plus warfarin with a target .1.5
.3-2 INR ‫ ﻭﺍﺭﻓﺎﺭﻳﻦ ﻣﻊ ﻫﺪﻑ‬-E
INR of 1.5
E- Warfarin with a target INR of 2.0 to 3.0
E
Une femme active de 78 ans souffrant d'hypertension se
présente avec une nouvelle hémiparésie gauche. La
surveillance cardiaque révèle une fibrillation auriculaire.
Elle était en rythme sinusal depuis 3 mois. Elle prend un
bêta-bloquant pour sa tension artérielle. Mis à part le
contrôle de la pression artérielle et du rythme cardiaque,
lequel des éléments suivants est approprié ?
A. Pacemaker permanent
B. Cardioversion à courant continu immédiat
C. Aspirine 81 mg par jour
D. Traitement antiplaquettaire plus warfarine avec
un INR cible de 1,5
E. warfarine avec un INR cible de 2,0 à 3,0

30 A
66. A 60-year-old woman develops chest pain, respiratory ‫ ﻭﺍﺭﺗﺒﺎﻙ‬،‫ ﺿﺎﺋﻘﺔ ﺗﻨﻔﺴﻴﺔ‬،‫ ﻋﺎﻣﺎ ﺗﻌﺎﻧﻲ ﻣﻦ ﺃﻟﻢ ﻓﻲ ﺍﻟﺼﺪﺭ‬60 ‫ﺍﻣﺮﺃﺓ ﻋﻤﺮﻫﺎ‬
distress, and confusion after right hip replacement ‫ ﺍﺳﺘﺠﺎﺑﺎﺗﻬﺎ ﻫﻲ ﻓﻲ ﺍﻟﺤﺪ ﺍﻷﺩﻧﻰ‬.‫ﺑﻌﺪ ﺟﺮﺍﺣﺔ ﺍﺳﺘﺒﺪﺍﻝ ﻣﻔﺼﻞ ﺍﻟﻮﺭﻙ ﺍﻷﻳﻤﻦ‬
surgery. She is minimally responsive and appears in ‫ ﻭﻣﻌﺪﻝ ﺿﺮﺑﺎﺕ‬50 /80 ‫ ﺿﻐﻂ ﺍﻟﺪﻡ‬.‫ﻭﺗﻈﻬﺮ ﻓﻲ ﺣﺎﻟﺔ ﺿﺎﺋﻘﺔ ﺗﻨﻔﺴﻴﺔ‬
respiratory distress. Blood pressure is 80/50 and heart ‫ ﺃﻅﻬﺮ ﺗﺨﻄﻴﻂ‬.‫ ﺑﺎﺭﺩﺓ ﻭﻣﺮﻗﺸﺔ‬،‫ ﺃﻁﺮﺍﻓﻬﺎ ﺫﺍﺕ ﺗﺮﻭﻳﺔ ﺳﻴﺌﺔ‬.‫ﺩ‬/155 ‫ﺍﻟﻘﻠﺐ‬
rate is 155/min. Her extremities are poorly perfused, ‫ ﺃﻱ‬.‫ﺍﻟﻘﻠﺐ ﺍﻟﻜﻬﺮﺑﺎﺋﻲ ﺑﺪء ﺣﺪﻳﺚ ﻟﺮﺟﻔﺎﻥ ﺃﺫﻳﻨﻲ ﻣﻊ ﺍﺳﺘﺠﺎﺑﺔ ﺑﻄﻴﻨﻴﺔ ﺳﺮﻳﻌﺔ‬
cold, and mottled. ECG reveals new-onset atrial :‫ﻣﻦ ﺍﻟﺘﺎﻟﻲ ﻫﻮ ﺍﻟﺘﺪﺑﻴﺮ ﺍﻷﻓﻀﻞ ﻟﻼﻧﻈﻤﻴﺔ ﻋﻨﺪ ﻫﺬﺍ ﺍﻟﻤﺮﻳﺾ؟‬
fibrillation with rapid ventricular response. Which of the .J360 ‫ ﻧﺰﻉ ﺭﺟﻔﺎﻥ ﻓﻮﺭﻱ ﻣﻊ‬-A
following is the best management of this patient’s .‫ ﺃﻣﻴﻮﺩﺍﺭﻭﻥ ﻭﺭﻳﺪﻱ‬-B
arrhythmia ? .‫ ﻣﺘﻮﺑﺮﻭﻟﻮﻝ ﻭﺭﻳﺪﻱ‬-C
A- Immediate defibrillation with 360 J .‫ ﺃﺩﻧﻮﺯﻳﻦ ﻭﺭﻳﺪﻱ‬-D
B- Intravenous amiodarone .J120 ‫ ﺗﻘﻮﻳﻢ ﻧﻈﻢ ﻗﻠﺐ ﻛﻬﺮﺑﺎﺋﻲ ﻓﻮﺭﻱ ﻣﻊ‬-E
C- Intravenous metoprolol
D- Intravenous adenosine
E- Immediate electric cardioversion with 120 J
E
Une femme de 60 ans développe une douleur à la
poitrine, une détresse respiratoire et une confusion après
une chirurgie de remplacement de la hanche droite. Elle
est minimalement réactive et apparaît en détresse
respiratoire. La pression artérielle à 80 /50 et la fréquence
cardiaque est de 155 / min. Ses extrémités sont mal
perfusées, froides et marbrées. L'ECG révèle une
fibrillation auriculaire d'apparition récente avec une
réponse ventriculaire rapide. Quelle est la meilleure
gestion de l’arythmie de ce patient?
A. Défibrillation immédiate avec 360 J
B. Amiodarone intraveineuse
C. Métoprolol par voie intraveineuse
D. Adénosine intraveineuse
E. Cardioversion électrique immédiate avec 120 J

31 A
67. A 30-year-old man is evaluated for a thyroid nodule. The ‫ ﻟﻘﺪ ﺃﻓﺎﺩ ﺍﻟﻤﺮﻳﺾ ﺃﻥ‬.‫ ﻋﺎﻣﺎ ﻳﺘﻢ ﺗﻘﻴﻴﻤﻪ ﻟﻠﻌﻘﻴﺪﺍﺕ ﺍﻟﺪﺭﻗﻴﺔ‬30 ‫ﺭﺟﻞ ﻋﻤﺮﻩ‬
patient reports that his father died from thyroid cancer ‫ﻭﺍﻟﺪﻩ ﻗﺪ ﺗﻮﻓﻲ ﺑﺴﺒﺐ ﺳﺮﻁﺎﻥ ﺍﻟﺪﺭﻕ ﻭﺃﻥ ﺷﻘﻴﻘﻪ ﻟﺪﻳﻪ ﺗﺎﺭﻳﺦ ﻣﻦ ﺍﻟﺤﺼﻴﺎﺕ‬
and that a brother had a history of recurrent renal stones. ‫ ﻣﻞ‬/‫ﺑﻴﻜﻮﻏﺮﺍﻡ‬2000 ‫ ﺗﺮﻛﻴﺰ ﺍﻟﻜﺎﻟﺴﻴﺘﻮﻧﻴﻦ ﺍﻟﺪﻣﻮﻱ‬.‫ﺍﻟﻜﻠﻮﻳﺔ ﺍﻟﻤﺘﻜﺮﺭﺓ‬
Blood calcitonin concentration is 2000pg/mL (normal is .‫ ﻣﺴﺘﻮﻳﺎﺕ ﺍﻟﻜﺎﻟﺴﻴﻮﻡ ﻭﺍﻟﻔﻮﺳﻔﺎﺕ ﺍﻟﻤﺼﻠﻴﺔ ﻁﺒﻴﻌﻴﺔ‬،(100> ‫)ﺍﻟﻄﺒﻴﻌﻲ ﻫﻮ‬
<100) ; serum calcium and phosphate levels are normal. ‫ ﺃﻱ ﻣﻦ‬.‫ﻟﻘﺪ ﺃﺣﻴﻞ ﺍﻟﻤﺮﻳﺾ ﺇﻟﻰ ﻁﺒﻴﺐ ﺟﺮﺍﺡ ﻣﺨﺘﺺ ﺑﺎﻟﻐﺪﺓ ﺍﻟﺪﺭﻗﻴﺔ‬
The patient is referred to a thyroid surgeon. Which of the :‫ﺍﻟﺪﺭﺍﺳﺎﺕ ﺍﻟﺘﺎﻟﻴﺔ ﻳﻨﺒﻐﻲ ﺍﻻﺳﺘﺤﺼﺎﻝ ﻋﻠﻴﻬﺎ؟‬
following studies should also be obtained ? .‫ ﺍﺟﺮﺍء ﻣﺴﺢ ﺿﻮﺋﻲ ﻟﻠﻜﺒﺪ‬-A
A- Obtain a liver scan .‫ ﻗﻴﺎﺱ ﻣﺴﺘﻮﻯ ﺟﺎﺭﺍﺕ ﺍﻟﺪﺭﻕ‬-B
B- Measure parathormone level .‫ ﻗﻴﺎﺱ ﻣﺘﺎﻧﻔﺮﻳﻦ ﺍﻟﺒﻮﻟﻲ‬-C
C- Measure urinary metanephrines ‫ ﺇﻋﻄﺎءﺟﺮﻋﺎﺕ ﻗﻤﻌﻴﺔ ﻣﻦ ﺍﻟﺘﻴﺮﻭﻛﺴﻴﻦ ﻭﻗﻴﺎﺱ ﻣﺴﺘﻮﻳﺎﺕ‬-D
. TSH‫ﺍﻟﻬﺮﻣﻮﻥ ﺍﻟﻤﻨﺒﻪ ﻟﻠﺪﺭﻕ‬
D- Administer suppressive doses of thyroxine and
.‫ ﻣﻌﺎﻟﺠﺔ ﺍﻟﻤﺮﻳﺾ ﺑﺎﻟﻴﻮﺩ ﺍﻟﻤﺸﻊ‬-E
measure levels of thyroid-simulating hormone
TSH
E- Treat the patient with radioactive iodine

Un homme de 30 ans est évalué pour un nodule C


thyroïdien. Le patient rapporte que son père est décédé
d'un cancer de la thyroïde et qu'un frère avait des
antécédents de calculs rénaux récurrents. La concentration
sanguine de calcitonine est de 2000 pg / ml (la normale
est <100); les taux sériques de calcium et de phosphate
sont normaux. Le patient est référé à un chirurgien de la
thyroïde. Laquelle des études suivantes devrait également
être obtenue?
A. Obtenir un scanner du foie
B. Mesurer le niveau de parathormone
C. Mesurer les métanéphrines urinaires
D. Administrer des doses suppressives de thyroxine
et mesurer les taux d'hormone simulant la
thyroïde TSH
E. Traiter le patient avec de l'iode radioactif
68. A 45-year-old patient presents with a history of weaknes, ‫ ﻭﻓﺮﻁ‬،‫ ﺍﻧﺨﻔﺎﺽ ﺗﻮﺗﺮ ﺍﻧﺘﺼﺎﺑﻲ‬،‫ ﻋﺎﻣﺎ ﻳﻌﺎﻧﻲ ﻣﻦ ﺗﻌﺐ‬45 ‫ﻣﺮﻳﺾ ﻋﻤﺮﻩ‬
orthostatic hypotension, and hyperpigmentation. Which ‫ ﺃﻱ ﻣﻦ ﺍﻟﻔﺤﻮﺹ ﺍﻟﻤﺨﺒﺮﻳﺔ ﺍﻟﺘﺎﻟﻴﺔ ﺳﺘﺴﺎﻋﺪ ﻓﻲ ﺇﺟﺮﺍء ﺍﻟﺘﺸﺨﻴﺺ‬.‫ﺗﺼﺒﻎ‬
of the following laboratory tests ould assist in making the :‫ﺍﻟﺼﺤﻴﺢ؟‬
correct diagnosis ? .‫ ﺳﺎﻋﺔ ﻟﻠﺘﺤﺮﻱ ﻋﻦ ﺍﻟﻜﺎﺗﻴﻜﻮﻻﻣﻴﻦ‬24 ‫ ﻓﺤﺺ ﺑﻮﻝ‬-A
A- 24-hour urine test for catecholamines .‫ ﻓﺤﺺ ﺗﺤﻤﻞ ﺍﻟﻐﻠﻮﻛﻮﺯ‬-B
B- Glucose tolerance test .ACTH ‫ ﻣﺴﺘﻮﻯ‬-C
C- ACTH level .GH ‫ ﻣﺴﺘﻮﻯ‬-D
D- GH level .TSH -E
E- TSH
C
Un patient de 45 ans présente des antécédents de faiblesse,
d'hypotension orthostatique et d'hyperpigmentation. Lequel des
tests de laboratoire suivants pourrait aider à poser le bon
diagnostic?
A. Test urinaire des catécholamines sur 24 heures
B. Test de tolérance au glucose
C. niveau ACTH
D. Niveau GH
E. TSH

32 A
69. A 55-year-old patient presents with severe palpitations .‫ ﻋﺎﻣﺎ ﺣﻀﺮ ﻭﻫﻮ ﻳﺸﻜﻮ ﻣﻦ ﺧﻔﻘﺎﻥ ﺷﺪﻳﺪ ﻭﺣﺮﺍﺭﺓ‬55 ‫ﻣﺮﻳﺾ ﻋﻤﺮﻩ‬
and fever. Thyroid studies reveal an elevated free ‫ ﻭﻣﺴﺘﻮﻯ‬،‫ﺃﻅﻬﺮﺕ ﺍﻟﺪﺭﺍﺳﺎﺕ ﺍﻟﺪﺭﻗﻴﺔ ﺍﺭﺗﻔﺎﻋﺎ ﻓﻲ ﻫﺮﻣﻮﻥ ﺍﻟﺪﺭﻕ ﺍﻟﺤﺮ‬
Thyroxin, and a TSH level that is undertectable. Which of ‫ ﺃﻱ ﻣﻦ ﺍﻟﺘﺎﻟﻲ ﻫﻮ ﺍﻟﺘﺪﺑﻴﺮ ﺍﻟﻌﻼﺟﻲ ﺍﻷﻓﻀﻞ ﻟﻬﺬﺍ‬.‫ ﻏﻴﺮ ﻣﻠﺤﻮﻅ‬TSH
the following is the most appropriate management of this :‫ﺍﻟﻤﺮﻳﺾ ﺑﺎﻧﺘﻈﺎﺭ ﺍﺟﺮﺍء ﺍﻟﻤﺰﻳﺪ ﻣﻦ ﺍﻟﺘﻘﻴﻴﻢ؟‬
patient pending further evaluation ? .‫ ﺃﺳﺒﻴﺮﻳﻦ‬-A
A- Aspirin .‫ ﻫﺮﻣﻮﻥ ﺩﺭﻕ ﺍﺻﻄﻨﺎﻋﻲ‬-B
B- Synthroid .(‫ ﺑﺮﻭﺑﺮﺍﻧﻮﻟﻮﻝ )ﺇﻧﺪﻳﺮﺍﻝ‬-C
C- Propranolol (Inderal) .‫ ﻣﻨﻈﻢ ﻗﻠﺐ‬-D
D- Cardiac pacing .‫ ﺍﻷﻣﺮ ﻻ ﻳﺴﺘﺪﻋﻲ ﺍﻟﻌﻼﺝ‬-E
E- No treatment is indicated
C
Un patient de 55 ans présente des palpitations sévères et
de la fièvre. Les études sur la thyroïde révèlent un taux de
Thyroxine libre élevé et un taux de TSH
incommensurable. Quelle est la gestion la plus appropriée
de ce patient en attendant une évaluation plus poussée?
A. Aspirine
B. Synthroid
C. Propranolol (Inderal)
D. stimulation cardiaque
E. Aucun traitement n'est indiqué
70. Which of the following physical examination findings is ‫ﺃﻱ ﻣﻦ ﻣﻈﺎﻫﺮ ﺍﻟﻔﺤﺺ ﺍﻟﺴﺮﻳﺮﻱ ﺍﻟﺘﺎﻟﻴﺔ ﺗﺸﺎﻫﺪ ﻓﻲ ﺍﻟﻮﺭﻡ ﺍﻟﻐﺪﻱ ﺍﻟﻠﻴﻔﻲ ﻓﻲ‬
typically seen in fibroadenoma of the breast ? :‫ﺍﻟﺜﺪﻱ؟‬
A- Rubbery mass .‫ ﻛﺘﻠﺔ ﻣﻄﺎﻁﻴﺔ‬-A
B- Retraction .‫ ﺍﻧﻜﻤﺎﺵ‬-B
C- Nipple flattening .‫ ﺣﻠﻤﺔ ﻣﺴﻄﺤﺔ‬-C
D- Nipple discharge .‫ ﺇﻓﺮﺍﺯ ﻣﻦ ﺍﻟﺤﻠﻤﺔ‬-D
A
Lequel des résultats de l'examen physique suivant est
généralement observé dans le fibroadénome du sein?
A- Consistance « caoutchouc »
B- Rétraction
C- aplatissement du mamelon
D- décharge dumamelon
71. Which of the following is the choice for Trichomonas :‫ﺃﻱ ﻣﻦ ﺍﻟﺘﺎﻟﻲ ﻫﻮ ﺍﻟﺨﻴﺎﺭ ﻟﻌﻼﺝ ﺍﻟﻤﺸﻌﺮﺓ ﺍﻟﻤﻬﺒﻠﻴﺔ؟‬
vginalis ? .‫ ﺑﻮﺩﻭﻓﻴﻠﻠﻴﻦ‬-A
A- Podophyllin .‫ ﺩﻭﻛﺴﻴﺴﻴﻜﻠﻴﻦ‬-B
B- Doxycycline .‫ ﻣﺘﺮﻭﻧﻴﺪﺍﺯﻭﻝ‬-C
C- Metronidazole .‫ ﺑﻨﻴﺴﻠﻠﻴﻦ‬-D
D- Penicillin .‫ ﻧﺴﺘﺎﺗﻴﻦ‬-E
E- Nystatin
C
Lequel des choix suivants est choisi pour Trichomonas
vginalis?
A- Podophylline
B-Doxycycline
C- Métronidazole
D- Pénicilline
E-Nystatine

33 A
72. Which of the following are the most common widespread ‫ﺃﻱ ﻣﻦ ﺍﻟﺘﺎﻟﻲ ﻫﻲ ﺍﻷﻋﺮﺍﺽ ﺍﻟﻤﻨﺘﺸﺮﺓ ﺍﻷﻛﺜﺮ ﺷﻴﻮﻋﺎ ﻟﻸﻟﻢ ﺍﻟﻠﻴﻔﻲ‬
symptoms of fibromyalgia ? :‫ﺍﻟﻌﻀﻠﻲ؟‬
A. Vision loss and jaw claudication .‫ ﻓﻘﺪﺍﻥ ﺍﻟﺮﺅﻳﺔ ﻭﻋﺮﺝ ﻓﻜﻲ‬-A
B. Headache and nuchal rigidity .‫ ﺻﺪﺍﻉ ﻭﺻﻼﺑﺔ ﻗﻔﻮﻳﺔ‬-B
C. Diarrhea and abdominal pain .‫ ﺇﺳﻬﺎﻝ ﻭﺃﻟﻢ ﺑﻄﻨﻲ‬-C
D. Depression and fatigue .‫ ﺍﻛﺘﺌﺎﺏ ﻭﺗﻌﺐ‬-D
E. Pain and stiffness .‫ ﺃﻟﻢ ﻭﺗﻴﺒﺲ‬-E
E
Parmi les symptômes suivants, lesquels sont les plus
répandus de la fibromyalgie?
A- Perte de vision et claudication de la mâchoire
B- Maux de tête et rigidité nucale
C- Diarrhée et douleurs abdominales
D- Dépression et fatigue
E- Douleur et raideur
73. Which of the following is the most likely cause of a ‫ﺃﻱ ﻣﻦ ﺍﻟﺘﺎﻟﻲ ﻫﻮ ﺍﻟﺴﺒﺐ ﺍﻷﻛﺜﺮ ﺗﺮﺟﻴﺤﺎ ﻟﻠﻔﺤﺺ ﺍﻹﻳﺠﺎﺑﻲ ﻟﻠﻜﺮﻳﺎﺕ‬
positive fecal leukocyte test ? :‫ﺍﻟﺒﻴﻀﺎء ﺍﻟﺒﺮﺍﺯﻳﺔ؟‬
A. External hemorrhoid .‫ ﺑﺎﺳﻮﺭ ﺧﺎﺭﺟﻲ‬-A
B. Shigella infection .‫ ﺍﻹﺻﺎﺑﺔ ﺑﺎﻟﺸﻴﻐﻠﻼ‬-B
C. Giardia infection .‫ ﺍﻹﺻﺎﺑﺔ ﺑﺎﻟﺠﻴﺎﺭﺩﻳﺎ‬-C
D. Viral infection .‫ ﺇﺻﺎﺑﺔ ﻓﻴﺮﻭﺳﻴﺔ‬-D
E. Rectal infection .‫ ﺇﺻﺎﺑﺔ ﺷﺮﺟﻴﺔ‬-E
B
Laquelle des causes suivantes est la cause la plus probable
d’un test positif aux leucocytes fécaux?
A- Hémorroïde externe
B- Infection à Shigella
C- Infection à Giardia
D- Infection virale
E- Infection rectale
74. Toxic megacolon is a common complication of which of ‫ﺗﻀﺨﻢ ﺍﻟﻘﻮﻟﻮﻥ ﺍﻟﺴﻤﻲ ﻫﻮ ﻣﻦ ﺍﻟﻤﻀﺎﻋﻔﺎﺕ ﺍﻟﺸﺎﺋﻌﺔ ﻷﻱ ﻣﻦ ﺍﻻﺿﻄﺮﺍﺑﺎﺕ‬
the following disorders ? :‫ﺍﻟﺘﺎﻟﻴﺔ؟‬
A. Ulcerative colitis .‫ ﺍﻟﺘﻬﺎﺏ ﺍﻟﻘﻮﻟﻮﻥ ﺍﻟﻤﺘﻘﺮﺡ‬-A
B. Cystic fibrosis .‫ ﺍﻟﺘﻠﻴﻒ ﺍﻟﻜﻴﺴﻲ‬-B
C. Diverticulosis .‫ ﺩﺍء ﺍﻟﺮﺗﻮﺝ‬-C
D. Crohn’s disease .‫ ﺩﺍء ﻛﺮﻭﻫﻦ‬-D
E. Colon cancer .‫ ﺳﺮﻁﺎﻥ ﺍﻟﻘﻮﻟﻮﻥ‬-E
A
Le mégacôlon toxique est une complication fréquente de
laquelle des troubles suivants?
A. colite ulcéreuse
B. Fibrose kystique
C. Diverticulose
D. maladie de Crohn
E. cancer du colon

34 A
75. A 25-year-old male presents with sudden onset of ‫ ﻋﺎﻣﺎ ﺣﻀﺮ ﻭﻫﻮ ﻳﺸﻜﻮ ﻣﻦ ﺑﺪء ﻣﻔﺎﺟﺊ ﻟﻘﺼﺮ ﺗﻨﻔﺲ ﻭﺃﻟﻢ‬25 ‫ﺭﺟﻞ ﻋﻤﺮﻩ‬
shortness of breath and chest pain. Physical examination ‫ ﺃﻅﻬﺮ ﺍﻟﻔﺤﺺ ﺍﻟﺴﺮﻳﺮﻱ ﻏﻴﺎﺏ ﺃﺻﻮﺍﺕ ﺍﻟﻨﻔﺲ ﻓﻲ ﻛﺎﻣﻞ‬.‫ﻓﻲ ﺍﻟﺼﺪﺭ‬
reveals absent breath sounds on the entire right side. :‫ ﺃﻱ ﻣﻦ ﺍﻟﺘﺎﻟﻲ ﻫﻮ ﺍﻟﺨﻴﺎﺭ ﺍﻟﻌﻼﺟﻲ؟‬.‫ﺍﻟﺠﺎﻧﺐ ﺍﻷﻳﻤﻦ‬
Which of the following is the treatment of choice ? .‫ ﻫﻴﺒﺎﺭﻳﻦ‬-A
A. Heparin . .‫ ﺗﻨﺒﻴﺐ ﺻﺪﺭﻱ‬-B
B. Chest tube .‫ ﺑﺮﻭﺳﺘﺎﻏﻼﻧﺪﻳﻦ‬-C
C. Prostaglandin .‫ ﻛﻮﺭﺗﻴﻜﻮﺳﺘﻴﺮﻭﺋﻴﺪ‬-D
D. Corticosteroids .‫ ﻗﻴﺎﺱ ﺍﻟﺘﻨﻔﺲ ﺍﻟﻤﺤﻔﺰ‬-E
E. Incentive spirometry
B
Un homme de 25 ans présente un essoufflement soudain
et des douleurs à la poitrine. L'examen physique révèle
des bruits de souffle absents sur tout le côté droit. Lequel
des traitements suivants est le traitement de choix?
A. Héparine
B. Tube thoracique
C. Prostaglandine
D. corticostéroïdes
E. Spirométrie incitative
76. High-risk groups, such as the elderly, patients with ‫ ﺍﻟﻤﺮﺿﻰ‬،‫ ﻣﺜﻞ ﻛﺒﺎﺭ ﺍﻟﺴﻦ‬،‫ﺍﻟﻤﺠﻤﻮﻋﺎﺕ ﺍﻷﻛﺜﺮ ﻋﺮﺿﺔ ﻟﻠﻤﺨﺎﻁﺮ‬
COPD, and children aged 6 to 24 months, should ،‫ ﺷﻬﺮﺍ‬24‫ ﻭ‬6 ‫ ﻭﺍﻷﻁﻔﺎﻝ ﻓﻲ ﺳﻦ ﻣﺎ ﺑﻴﻦ‬،‫ﺍﻟﻤﺼﺎﺑﻮﻥ ﺑﺎﻧﺴﺪﺍﺩ ﺭﺋﻮﻱ ﻣﺰﻣﻦ‬
routinely receive which of the following yearly vaccines? :‫ﻳﺠﺐ ﺗﻠﻘﻴﻬﻢ ﺑﺸﻜﻞ ﺭﻭﺗﻴﻨﻲ ﺃﻱ ﻣﻦ ﺍﻟﻠﻘﺎﺣﺎﺕ ﺍﻟﺴﻨﻮﻳﺔ ﺍﻟﺘﺎﻟﻴﺔ؟‬
A. Tuberculosis .‫ ﻟﻘﺎﺡ ﺍﻟﺴﻞ‬-A
B. Tetanus .‫ ﻟﻘﺎﺡ ﺍﻟﻜﺰﺍﺯ‬-B
C. Pertussis .‫ ﻟﻘﺎﺡ ﺍﻟﺸﺎﻫﻮﻕ‬-C
D. Influenza .‫ ﻟﻘﺎﺡ ﺍﻹﻧﻔﻠﻮﻧﺰﺍ‬-D
E. Pneumococcal .‫ ﻟﻘﺎﺡ ﺍﻟﻤﻜﻮﺭﺍﺕ ﺍﻟﺮﺋﻮﻳﺔ‬-E

D
Les groupes à haut risque, tels que les personnes âgées,
les patients atteints de BPCO et les enfants âgés de 6 à 24
mois, devraient-ils recevoir systématiquement l'un des
vaccins annuels suivants?
A. Tuberculose
B. Tétanos
C. La coqueluche
D. Influenza
E. Pneumocoque
77. Which of the following chemotherapy agents is most :‫ﻣﺎ ﻫﻮ ﺍﻟﻌﻼﺝ ﺍﻟﻜﻴﻤﻴﺎﺋﻲ ﺍﻷﻛﺜﺮ ﺗﺮﺟﻴﺤﺎ ﺃﻥ ﻳﺆﺩﻱ ﺇﻟﻰ ﺗﻠﻴﻒ ﺭﺋﻮﻱ؟‬
likely to lead to pulmonary fibrosis ? .‫ ﻓﻨﻜﺮﻳﺴﺘﻴﻦ‬-A
A. Vincristine .‫ ﺑﻠﻴﻮﻣﺎﻳﺴﻴﻦ‬-B
B. Bleomycin .‫ ﺳﻴﺴﺒﻼﺗﻴﻦ‬-C
C. Cisplatin .‫ ﺗﺎﻣﻮﻛﺴﻴﻔﻦ‬-D
D. Tamoxifen .‫ ﻣﻴﺘﻮﺗﺮﻛﺴﺎﺕ‬-E
E. Methotrexate
B
Lequel des agents de chimiothérapie suivants est le plus
susceptible d'entraîner une fibrose pulmonaire?
A. Vincristine
B. Bléomycine
C. cisplatine
D. Tamoxifène
E. Méthotrexate

35 A
78. Which of the following is a major side effect of the long- ‫ﺃﻱ ﻣﻦ ﺍﻟﺘﺎﻟﻲ ﻫﻮ ﺍﻟﺘﺄﺛﻴﺮ ﺍﻟﺠﺎﻧﺒﻲ ﺍﻟﺮﺋﻴﺴﻲ ﻟﻼﺳﺘﻌﻤﺎﻝ ﻁﻮﻳﻞ ﺍﻷﻣﺪ‬
term use of nitrofurantoin in the elderly ? :‫ﻟﻨﻴﺘﺮﻓﻮﺭﺍﻧﺘﻮﺋﻴﻦ ﻋﻨﺪ ﻛﺒﺎﺭ ﺍﻟﺴﻦ؟‬
A. Pulmonary fibrosis .‫ ﺗﻠﻴﻒ ﺭﺋﻮﻱ‬-A
B. Renal insufficiency .‫ ﻗﺼﻮﺭ ﻛﻠﻮﻱ‬-B
C. Macrocytic anemia .‫ ﻓﻘﺮ ﺩﻡ ﻛﺒﻴﺮ ﺍﻟﻜﺮﻳﺎﺕ‬-C
D. Cardiomyopathy .‫ ﺍﻋﺘﻼﻝ ﻋﻀﻠﺔ ﺍﻟﻘﻠﺐ‬-D
E. Neuropathy .‫ ﺍﻋﺘﻼﻝ ﻋﺼﺒﻲ‬-E

A
Lequel des effets suivants est l’un des effets secondaires
majeurs de l’utilisation à long terme de la nitrofurantoïne
chez les personnes âgées?
A. Fibrose pulmonaire
B. Insuffisance rénale
C. Anémie macrocytaire
D. Cardiomyopathie
E. neuropathie
79. Which of the following is the most effective treatment for :‫ﺃﻱ ﻣﻦ ﺍﻟﺘﺎﻟﻲ ﻫﻮ ﺍﻷﻛﺜﺮ ﻓﻌﺎﻟﻴﺔ ﻓﻲ ﻣﻌﺎﻟﺠﺔ ﺍﻟﻤﻔﻄﻮﺭﺓ ﺍﻟﺮﺋﻮﻳﺔ؟‬
M.pneumoniae ? .‫ ﺳﻴﻔﺎﻟﻜﺴﻴﻦ‬-A
A. Cephalexin .‫ ﺃﻣﻮﻛﺴﻴﺴﻴﻠﻠﻴﻦ‬-B
B. Amoxicillin .‫ ﻓﺎﻧﻜﻮﻣﺎﻳﺴﻴﻦ‬-C
C. Vancomycin .‫ ﺃﺯﻳﺘﺮﻭﻣﺎﻳﺴﻴﻦ‬-D
D. Azithromycin .‫ ﺳﺘﺮﺑﺘﻮﻣﺎﻳﺴﻴﻦ‬-E
E. Streptomycin
D
Lequel des traitements suivants est le traitement le plus
efficace contre M. pneumoniae?
A. Cephalexin
B. Amoxicilline
C. vancomycine
D. Azithromycine
E. Streptomycine
80. Which of the following is used in the treatment of ‫ﺃﻱ ﻣﻦ ﺍﻟﺘﺎﻟﻲ ﻳﺴﺘﺨﺪﻡ ﻓﻲ ﻣﻌﺎﻟﺠﺔ ﺍﻟﺸﻌﺮﺍﻧﻴﺔ ﺍﻟﺘﺎﻟﻲ ﻟﻤﺘﻼﺯﻣﺔ ﺍﻟﻤﺒﻴﺾ‬
hirsutism secondary of polycystic ovarian syndrome ? :‫ﻣﺘﻌﺪﺩ ﺍﻟﻜﻴﺴﺎﺕ؟‬
A. Acarbose (Precose) .(‫ ﺃﻛﺎﺭﺑﻮﺯ )ﺑﺮﻳﻜﻮﺯ‬-A
B. Pioglitazone (Actos) .(‫ ﺑﻴﻮﻏﻠﻴﺘﺎﺯﻭﻥ )ﺃﻛﺘﻮﺱ‬-B
C. Subcutaneous insulin .‫ ﺍﻧﺴﻮﻟﻴﻦ ﺗﺤﺖ ﺍﻟﺠﻠﺪ‬-C
D. Metformin (Glucophage) .(‫ ﻣﺘﻔﻮﺭﻣﻴﻦ )ﻏﻠﻮﻛﻮﻓﺎﺝ‬-D
E. Spironolactone (Aldactone) .(‫ ﺳﺒﻴﺮﺍﻧﻮﻻﻛﺘﻮﻥ )ﺃﻟﺪﺍﻛﺘﻮﻥ‬-E

E
Lequel des éléments suivants est utilisé dans le traitement
de l'hirsutisme secondaire au syndrome des ovaires
polykystiques?
A. Acarbose (Précose)
B. Pioglitazone (Actos)
C. insuline sous-cutanée
D. Metformine (Glucophage)
E. Spironolactone (Aldactone)

36 A
81. In which of the following conditions is the alkaline :‫ﻓﻲ ﺃﻱ ﻣﻦ ﺍﻟﺤﺎﻻﺕ ﺍﻟﺘﺎﻟﻴﺔ ﻳﻜﻮﻥ ﻣﺴﺘﻮﻯ ﺍﻟﻔﻮﺳﻔﺎﺗﺎﺯ ﺍﻟﻘﻠﻮﻳﺔ ﻁﺒﻴﻌﻲ؟‬
phosphatase level normal ? .‫ ﺩﺍء ﺍﻹﻧﺴﺪﺍﺩ ﺍﻟﻜﺒﺪﻱ‬-A
A. Obstructive liver disease .‫ ﻓﺮﻁ ﻧﺸﺎﻁ ﺍﻟﺪﺭﻕ‬-B
B. Hyperparathyroidism .‫ ﺩﺍء ﺑﺎﻏﻴﺖ‬-C
C. Paget’s disease .‫ ﻫﺸﺎﺷﺔ ﺍﻟﻌﻈﺎﻡ‬-D
D. Osteoporosis .‫ ﺍﻟﺤﻤﻞ‬-E
E. Pregnancy
D
Dans laquelle des conditions suivantes le niveau de
phosphatase alcaline est-il normal?
A. Maladie hépatique obstructive
B. Hyperparathyroïdie
C. maladie de Paget
D. l'ostéoporose
E. grossesse
82. Which of the following laboratory abnormalities is noted ‫ﺃﻱ ﻣﻦ ﺍﻟﻌﻴﻮﺏ ﺍﻟﻤﺨﺒﺮﻳﺔ ﺍﻟﺘﺎﻟﻴﺔ ﺗﻼﺣﻆ ﻋﻨﺪ ﺍﻟﻤﺮﺿﻰ ﺍﻟﻤﺴﺘﻬﻠﻜﻴﻦ ﺣﺪﻳﺜﺎ‬
in patients with recent alcohol use ? :‫ﻟﻠﻜﺤﻮﻝ؟‬
A. Elevated GGT .‫ ﻣﺮﺗﻔﻊ‬GGT -A
B. Elevated bilirubin .‫ ﺑﻠﻴﺮﻭﺑﻴﻦ ﻣﺮﺗﻔﻊ‬-B
C. Decreased RBC count .‫ ﻣﻨﺨﻔﺾ‬RBC ‫ ﺗﻌﺪﺍﺩ‬-C
D. Decreased MCV .‫ ﻣﻨﺨﻔﺾ‬MCV -D
E. Decreased AST .‫ ﻣﻨﺨﻔﺾ‬AST -E

A
Parmi les anomalies de laboratoire suivantes, laquelle est
notée chez les patients ayant récemment consommé de
l'alcool?
A. GGT élevé
B. Bilirubine élevée
C. Diminution du nombre de globules rouges
D. Diminution du MCV
E. Diminution de l'AST
83. Which of the following is a primary source of amylase? :‫ﻣﺎ ﻫﻮ ﺍﻟﻤﺼﺪﺭ ﺍﻟﺮﺋﻴﺴﻲ ﻟﻸﻣﻴﻼﺯ؟‬
A. Liver .‫ ﺍﻟﻜﺒﺪ‬-A
B. Spleen .‫ ﺍﻟﻄﺤﺎﻝ‬-B
C. Testicles .‫ ﺍﻟﺨﺼﻴﺘﺎﻥ‬-C
D. Small intestine .‫ ﺍﻹﻣﻌﺎء ﺍﻟﺪﻗﻴﻘﺔ‬-D
E. Salivary glands .‫ ﺍﻟﻐﺪﺩ ﺍﻟﻠﻌﺎﺑﻴﺔ‬-E

E
Laquelle des sources suivantes est une source primaire
d’amylase?
A. Foie
B. Spleen
C. Testicules
D. Intestin grêle
E. Glandes salivaires

37 A
84. Which of the following is the pathophysiologic :‫ﺃﻱ ﻣﻦ ﺍﻟﺘﺎﻟﻲ ﻫﻲ ﺁﻟﻴﺔ ﻓﻴﺰﻳﻮﻟﻮﺟﻴﺔ ﻣﺮﺿﻴﺔ ﻟﺪﺍء ﺍﻟﻐﺸﺎء ﺍﻟﻬﻴﺎﻟﻴﻨﻲ؟‬
mechanism of hyaline membrane disease ? .‫ ﻋﻮﺯ ﺍﻟﻌﺎﻣﻞ ﺍﻟﻔﻌﺎﻝ ﻋﻠﻰ ﺍﻟﺴﻄﺢ‬-A
A. Surfactant deficiency .‫ ﺷﻔﻂ ﺍﻟﻌﻘﻲ‬-B
B. Meconium aspiration .‫ ﻓﺘﻖ ﺣﺠﺎﺑﻲ ﺧﻠﻘﻲ‬-C
C. Congenital diaphragmatic hernia .‫ ﻣﻘﺎﻭﻣﺔ ﻭﻋﺎﺋﻴﺔ ﺭﺋﻮﻳﺔ ﻋﺎﻟﻴﺔ‬-D
D. High pulmonary vascular resistance .‫ ﺍﻣﺘﺼﺎﺹ ﺑﻄﻲء ﻟﻠﺴﺎﺋﻞ ﺍﻟﺮﺋﻮﻱ ﺍﻟﺠﻨﻴﻨﻲ‬-E
E. Slow absorption of fetal lung fluid

Lequel des mécanismes suivants est le mécanisme B


physiopathologique de la maladie de la membrane
hyaline?
A. Carence en tensioactif
B. aspiration de méconium
C. Hernie diaphragmatique congénitale
D. Résistance vasculaire pulmonaire élevée
E. Absorption lente du liquide pulmonaire fœtal

85. A 6 month-old male presents with a scrotal mass. The ‫ ﺇﻥ ﻛﻴﺲ ﺍﻟﺼﻔﻦ ﻣﺘﻀﺨﻢ‬.‫ ﺃﺷﻬﺮ ﻟﺪﻳﻪ ﻛﺘﻠﺔ ﺻﻔﻨﻴﺔ‬6 ‫ﻁﻔﻞ ﻋﻤﺮﻩ‬
scrotum is swollen and the testicules are nontender. The ‫ﺃﻱ ﻣﻦ ﺍﻟﺘﺎﻟﻲ ﻫﻮ‬.‫ ﻛﻴﺲ ﺍﻟﺼﻔﻦ ﻧﺎﻓﺬ ﻟﻠﻀﻮء‬.‫ﻭﺍﻟﺨﺼﻴﺘﻴﻦ ﺑﺪﻭﻥ ﺇﻳﻼﻡ‬
scrotum does transilluminate. Which of the following is :‫ﺍﻟﺨﻴﺎﺭ ﺍﻟﻌﻼﺟﻲ؟‬
the treatment of choice ? .‫ ﺗﺜﺒﻴﺖ ﺍﻟﺨﺼﻴﺔ‬-A
A. Orchidopexy .‫ ﻗﻄﻊ ﺍﻟﻘﻴﻠﺔ ﺍﻟﺪﻭﺍﻟﻴﺔ‬-B
B. Varicocelectomy .‫ ﺍﺳﺘﺌﺼﺎﻝ ﺟﺬﺭﻱ ﻟﻠﺨﺼﻴﺔ‬-C
C. Radical orchiectomy .‫ ﺑﻠﻤﺮﺓ ﻣﺸﺘﺮﻛﺔ ﻣﻦ ﺩﻛﺴﺘﺮﺍﻧﻮﻣﺮ ﻭﺣﻤﺾ ﺍﻟﻬﻴﺎﻟﻮﺭﻭﻧﻲ‬-D
D. Dextranomer/Hyaluronic acid copolymer .‫ ﻻ ﺣﺎﺟﺔ ﻟﻠﻌﻼﺝ ﻓﻲ ﻫﺬﺍ ﺍﻟﻮﻗﺖ‬-E
E. No treatment is needed at this time
E
Un enfant de 6 mois se présente avec une masse scrotale.
Le scrotum est gonflé et les testicules ne sont pas
douloureux. Le scrotum transilluminate. Lequel des
traitements suivants est le traitement de choix?
A. Orchidopexie
B. Varicocélectomie
C. Orchidectomie radicale
D. Copolymère de dextranomère / acide
hyaluronique
E. Aucun traitement n'est nécessaire pour le moment
86. An 8-year-old patient with nocturnal perianal itching. ‫ ﺃﻱ ﻣﻦ ﺍﻟﺘﺎﻟﻲ‬.‫ ﺳﻨﻮﺍﺕ ﻳﻌﺎﻧﻲ ﻣﻦ ﺣﻜﺔ ﺣﻮﻝ ﺍﻟﺸﺮﺝ ﻟﻴﻠﻴﺔ‬8 ‫ﻣﺮﻳﺾ ﻋﻤﺮﻩ‬
Which of the following is the best treatment option for :‫ﻫﻮ ﺃﻓﻀﻞ ﺧﻴﺎﺭ ﻋﻼﺟﻲ ﻟﻬﺬﺍ ﺍﻟﻤﺮﻳﺾ؟‬
this patient ? .‫ ﺃﺳﻴﻜﻠﻮﻓﻴﺮ‬-A
A. Acyclovir .‫ ﺑﻨﺴﻴﻠﻠﻴﻦ‬-B
B. Penicillin .‫ ﻣﺘﺮﻭﻧﻴﺪﺍﺯﻭﻝ‬-C
C. Metronidazole .‫ ﻣﺒﻨﺪﺍﺯﻭﻝ‬-D
D. Mebendazole .‫ ﻫﻴﺪﺭﻭﻛﺴﻲ ﻛﻠﻮﺭﻭﻛﻮﻳﻦ‬-E
E. Hydroxycloroquine
D
Patient de 8 ans présentant des démangeaisons nocturnes
périanales. Laquelle des options suivantes est la meilleure
option de traitement pour ce patient?
A. Acyclovir
B. Pénicilline
C. Métronidazole
D. Mebendazole
E. Hydroxycloroquine

38 A
87. A 65-year-old female presents with yellow, greasy- ‫ ﻋﺎﻣﺎ ﺗﻌﺎﻧﻲ ﻣﻦ ﻫﻴﺎﺝ ﺟﻠﺪﻱ ﺩﻫﻨﻲ ﺃﺻﻔﺮ ﻳﻈﻬﺮ ﻋﻠﻰ‬65 ‫ﺍﻣﺮﺃﺓ ﻋﻤﺮﻫﺎ‬
appearing eruptions on her face. Which of the following is :‫ ﺃﻱ ﻣﻦ ﺍﻟﺘﺎﻟﻲ ﻫﻮ ﺍﻟﺘﺸﺨﻴﺺ ﺍﻷﻛﺜﺮ ﺗﺮﺟﻴﺤﺎ؟‬.‫ﻭﺟﻬﻬﺎ‬
the most likely diagnosis ? .‫ ﺍﻟﻮﺭﺩﻳﺔ‬-A
A. Rosacea .‫ ﺍﻟﺼﺪﻓﻴﺔ‬-B
B. Psoriasis .‫ ﺣﺰﺍﺯ ﻣﺴﻄﺢ‬-C
C. Lichen planus .‫ ﺍﻟﻨﺨﺎﻟﻴﺔ ﺍﻟﻮﺭﺩﻳﺔ‬-D
D. Pityriasis rosea .‫ ﺍﻟﺘﻬﺎﺏ ﺍﻟﺠﻠﺪ ﺍﻟﻤﺜﻲ‬-E
E. Seborrheic dermatitis
E
Une femme de 65 ans présente des éruptions jaunes
d'apparence grasse sur le visage. Lequel des diagnostics
suivants est le plus probable?
A. Rosacée
B. Psoriasis
C. Lichen plan
D. Pityriasis rosea
E. dermatite séborrhéique
88. A 17-year-old male presents with right groin pain for the ‫ ﻋﺎﻣﺎ ﻳﺸﻜﻮ ﻣﻦ ﺃﻟﻢ ﻓﻲ ﺍﻟﻔﺨﺬ ﺍﻷﻳﺴﺮ ﺧﻼﻝ ﺍﻟﺴﺎﻋﺘﻴﻦ‬17 ‫ﺭﺟﻞ ﻋﻤﺮﻩ‬
past 2 hours. Elevation of the scrotum relieves the pain. ‫ ﺇﻥ ﺍﻟﻤﺮﻳﺾ ﻛﺎﻥ ﻣﻦ‬.‫ ﺭﻓﻊ ﻛﻴﺲ ﺍﻟﺼﻔﻦ ﻳﺨﻔﻒ ﻣﻦ ﺍﻷﻟﻢ‬.‫ﺍﻟﻤﺎﺿﻴﺘﻴﻦ‬
The patient has been sexually active with multiple ‫ ﻋﻨﺪ‬.‫ﺍﻟﻨﺎﺷﻄﻴﻦ ﺟﻨﺴﻴﺎ ﻣﻊ ﻋﺪﺓ ﺷﺮﻛﺎء ﻋﻠﻰ ﻣﺪﻯ ﺍﻟﺴﻨﺘﻴﻦ ﺍﻟﻤﺎﺿﻴﺘﻴﻦ‬
partners over the past 2 years. On physical examination, ‫ ﺗﻮﺭﻡ ﻭﺍﺣﻤﺮﺍﺭ ﻓﻲ ﻛﻴﺲ ﺍﻟﺼﻔﻦ‬،‫ ﻫﻨﺎﻙ ﺇﻳﻼﻡ‬،‫ﺍﻟﻔﺤﺺ ﺍﻟﺴﺮﻳﺮﻱ‬
the right scrotum is actuely tender, swollen, and :‫ﺃﻱ ﻣﻦ ﺍﻟﺘﺎﻟﻲ ﻫﻮ ﺍﻟﺘﺸﺨﻴﺺ ﺍﻷﻛﺜﺮ ﺗﺮﺟﻴﺤﺎ؟‬.‫ﺍﻷﻳﻤﻦ‬
erythematous. Which of the following is the most likely .‫ ﺍﻟﺘﻬﺎﺏ ﺑﺮﻭﺳﺘﺎﺕ‬-A
diagnosis ? .‫ ﺍﻟﺘﻬﺎﺏ ﺍﻟﺒﺮﺑﺦ‬-B
.‫ ﻓﺘﻖ ﻣﻨﺤﺒﺲ‬-C
A. Prostatis
.‫ ﺍﻟﺘﻮﺍء ﺍﻟﺨﺼﻴﺔ‬-D
B. Epididymitis
.‫ ﻏﻨﻐﺮﻳﻨﺔ ﻓﻮﺭﻧﻴﺮ‬-E
C. Incarcerated hernia
D. Torsion of the testicle
E. Fournier’s gangrene
B
Un homme de 17 ans souffre de douleurs à l'aine droite
depuis 2 heures. L'élévation du scrotum soulage la
douleur. La patiente a été sexuellement active avec
plusieurs partenaires au cours des 2 dernières années. À
l'examen physique, le scrotum droit est réellement tendre,
enflé et érythémateux. Lequel des diagnostics suivants est
le plus probable?
A. Prostatis
B. Épididymite
C. Hernie incarcérée
D. Torsion du testicule
E. La gangrène de Fournier

39 A
89. Which of the following statements is FALSE regarding :‫ﺃﻱ ﻣﻦ ﺍﻟﻌﺒﺎﺭﺍﺕ ﺍﻟﺘﺎﻟﻴﺔ ﺧﻄﺄ ﻓﻴﻤﺎ ﻳﺘﻌﻠﻖ ﺑﺎﻟﻠﻘﺎﺣﺎﺕ؟‬
vaccinations: ‫ ﺇﻥ ﺧﻄﺮ ﺍﻛﺘﺴﺎﺏ ﺟﺪﺭﻱ ﺍﻟﻤﺎء ﺑﻌﺪ ﺍﻟﺘﻌﺮﺽ ﻋﻨﺪ‬.A
A. The risk of acquiring chicken pox after exposure in the ‫ ﻫﻮ ﺃﻗﻞ‬،‫ ﻭﺗﻤﻨﻴﻊ ﺿﺪ ﺍﻟﺤﻤﺎﻕ‬،‫ﻁﻔﻞ ﺫﻭ ﺻﺤﺔ ﺟﻴﺪﺓ‬
healthy, varicella immunized child is less than 10%.
.%10 ‫ﻣﻦ‬
B. Vaccines have no adverse affects.
C. Many vaccines need to be administered more than .‫ ﺍﻟﻠﻘﺎﺣﺎﺕ ﻟﻴﺲ ﻟﺪﻳﻬﺎ ﺗﺄﺛﻴﺮ ﺟﺎﻧﺒﻲ‬.B
once. .‫ ﺍﻟﻌﺪﻳﺪ ﻣﻦ ﺍﻟﻠﻘﺎﺣﺎﺕ ﻳﺤﺘﺎﺝ ﺍﻋﻄﺎﺅﻫﺎ ﺃﻛﺜﺮ ﻣﻦ ﻣﺮﺓ‬.C
D. Rubella incidence has decreased 99% since 1969. %99 ‫ ﺍﻹﺻﺎﺑﺔ ﺑﺎﻟﺤﺼﺒﺔ ﺍﻷﻟﻤﺎﻧﻴﺔ ﺍﻧﺨﻔﻀﺖ ﺑﻨﺴﺒﺔ‬.D
E. Adverse effects of illnesses prevented by vaccines .1969 ‫ﻣﻨﺬ ﻋﺎﻡ‬
include death and damage to the central nervous system. ‫ ﺍﻟﺘﺄﺛﻴﺮﺍﺕ ﺍﻟﺠﺎﻧﺒﻴﺔ ﻟﻸﻣﺮﺍﺽ ﺍﻟﺘﻲ ﻟﻬﺎ ﻟﻘﺎﺣﺎﺕ‬.E
.‫ﺗﺘﻀﻤﻦ ﺍﻟﻤﻮﺕ ﻭﺇﺻﺎﺑﺔ ﺍﻟﺠﻬﺎﺯ ﺍﻟﻌﺼﺒﻲ ﺍﻟﻤﺮﻛﺰﻱ‬
Laquelle des affirmations suivantes est FAUSSE
B
concernant les vaccins:
A. Le risque de contracter la varicelle après une exposition
chez l’enfant en bonne santé et immunisé contre la
varicelle est inférieur à 10%.
B. Les vaccins n'ont pas d'effets indésirables.
C. De nombreux vaccins doivent être administrés plus
d'une fois.
D. L’incidence de la rubéole a diminué de 99% depuis
1969.
E. Parmi les effets indésirables des maladies évitées par
les vaccins figurent le déçès et des lésions du système
nerveux central.
90. All of the following are main areas affected in children ‫ﻛﻞ ﻣﺎ ﻳﻠﻲ ﻫﻲ ﻣﺴﺎﺣﺎﺕ ﺭﺋﻴﺴﻴﺔ ﺗﺘﻌﺮﺽ ﻟﻺﺻﺎﺑﺔ ﻋﻨﺪ‬
with Autistic Spectrum Disorder except: :‫ ﻣﺎ ﻋﺪﺍ‬،‫ﺍﻷﻁﻔﺎﻝ ﺍﻟﺬﻳﻦ ﻳﻌﺎﻧﻮﻥ ﻣﻦ ﺍﺿﻄﺮﺍﺏ ﺗﻮﺣﺪﻱ‬
A. Socialization .‫ ﺍﻻﺟﺘﻤﺎﻋﻴﺎﺕ‬.A
B. Language
.‫ ﺍﻟﻠﻐﺔ‬.B
C. Motor abilities
D. Repetitive and restricted interests and activities .‫ ﺍﻟﻘﺪﺭﺓ ﻋﻠﻰ ﺍﻟﺤﺮﻛﺔ‬.C
.‫ ﻧﺸﺎﻁﺎﺕ ﻭﺍﻫﺘﻤﺎﻣﺎﺕ ﻣﺤﺪﺩﺓ ﻭﻣﻜﺮﺭﺓ‬.D
C
Tous les domaines suivants sont principalement touchés
par les enfants atteints de trouble du spectre autistique, à
l'exception de:
A. La Sociabilité
B. Le Language
C. Les capacités motrices
D. Activités et intérêts répétitives et restreints
91. In G6PD deficiency, there is hyperbilirubinemia on the ‫ ﻫﻨﺎﻙ ﻓﺮﻁ ﺑﻴﻠﻴﺮﻭﺑﻴﻦ ﺍﻟﺪﻡ‬،G6PD ‫ﻓﻲ ﺣﺎﻟﺔ ﻋﻮﺯ ﺧﻤﻴﺮﺓ‬
basis of: :‫ﺑﺎﻻﺳﺘﻨﺎﺩ ﺇﻟﻰ‬
A. Hemolysis
.‫ ﺍﻧﺤﻼﻝ ﺍﻟﺪﻡ‬.A
B. Decreased conjugation
C. Both
.‫ ﺗﻨﺎﻗﺺ ﺍﻻﻗﺘﺮﺍﻥ‬.B
D. Neither .B + A .C
.‫ ﻛﻞ ﺍﻷﺟﻮﺑﺔ ﺃﻋﻼﻩ ﻏﻴﺮ ﺻﺤﻴﺤﺔ‬.D C
Dans le déficit en G6PD, il existe une hyperbilirubinémie
à cause de:
A. L’Hémolyse
B. Une Conjugaison diminuée
C. A+B
D. Les 2 réponses sont fausses

40 A
92. A true statement about the epidemiology of measles is :‫ﺇﻥ ﺍﻟﻌﺒﺎﺭﺓ ﺍﻟﺼﺤﻴﺤﺔﻋﻦ ﻭﺑﺎﺋﻴﺎﺕ ﺍﻟﺤﺼﺒﺔ ﻫﻲ ﺃﻥ‬
that: ‫ ﺃﻳﺎﻡ ﺍﺑﺘﺪﺍء ﻣﻦ ﺍﻟﺘﻌﺮﺽ‬5-4 ‫ ﻓﺘﺮﺓ ﺍﻟﺤﻀﺎﻧﺔ ﻫﻲ‬.A
A. Incubation period is 4-5 days from exposure to onset
.‫ﺣﺘﻰ ﺑﺪء ﺍﻷﻋﺮﺍﺽ‬
of symptoms
B. In Industrialized countries, immunization programs
‫ ﺑﺮﺍﻣﺞ ﺍﻟﺘﻤﻨﻴﻊ ﻗﺪ ﻗﻠﻠﺖ ﻣﻦ‬،‫ ﻓﻲ ﺍﻟﻤﺪﻥ ﺍﻟﺼﻨﺎﻋﻴﺔ‬.B
have reduced the incidence of infection by 60-70% .%70-60 ‫ﺣﻮﺍﺩﺙ ﺍﻹﺻﺎﺑﺔ ﺑﻨﺴﺒﺔ‬
C. Usually spread by direct contact with infectious ‫ ﺗﻨﺘﺸﺮ ﺍﻟﻌﺪﻭﻯ ﻋﺎﺩﺓ ﺑﺎﻟﺘﻤﺎﺱ ﺍﻟﻤﺒﺎﺷﺮ ﺧﺎﺻﺔ ﻣﻦ‬.C
salivary droplets .‫ﺭﺫﺍﺫ ﺍﻟﻠﻌﺎﺏ‬
D. Patients become contagious when the rash appears .‫ ﺗﺼﺒﺢ ﺍﻟﻤﺮﺿﻰ ﻣﻌﺪﻳﺔ ﻋﻨﺪﻡ ﻳﻈﻬﺮ ﺍﻟﻄﻔﺢ ﺍﻟﺠﻠﺪﻱ‬.D
Concernant la Rougeole, parmi les propositions qui C
suivent, citez celle qui est juste:
A. La phase d’incubation est de 4-5 jours allant de
l’exposition jusqu’à l’apparition des symptômes
B. Dans les pays industrialisés, les programmes
d’immunisation ont réduit l’incidence de cette maladie de
60-70%
C. La transmission est directe et se fait surtout par des
gouttelettes de salive
D. Les patients deviennet contagieux au moment de
l’apparition de l’éruption
93. You have just confirmed the diagnosis of cystic fibrosis in ‫ ﺍﻟﻮﺍﻟﺪﻳﻦ‬.‫ ﺳﻨﻮﺍﺕ‬3 ‫ﻟﻘﺪ ﺗﻢ ﺗﺸﺨﻴﺺ ﺗﻠﻴﻒ ﻛﻴﺴﻲ ﻟﻄﻔﻞ ﻋﻤﺮﻩ‬
a 3-year-old child. The parents are concerned about future ‫ ﻟﻘﺪ ﻓﺴﺮﺕ ﻟﻬﻤﺎ ﺑﺄﻥ‬.‫ﻳﺸﻌﺮﺍﻥ ﺑﺎﻟﻘﻠﻖ ﺣﻮﻝ ﺍﻟﺤﻤﻞ ﻓﻲ ﺍﻟﻤﺴﺘﻘﺒﻞ‬
pregnancies. You explain to them that the pattern of
:‫ﻧﻤﻂ ﺍﻻﻧﺘﻘﺎﻝ ﺍﻟﺠﻴﻨﻲ ﻟﻠﺘﻠﻴﻒ ﺍﻟﻜﻴﺴﻲ ﻫﻮ‬
genetic transmission of cystic fibrosis is:
A. Autosomal dominant
.‫ ﺻﺒﻐﻲ ﺟﺴﺪﻱ ﻣﺴﻴﻄﺮ‬.A
B. Autosomal recessive .‫ ﺻﺒﻐﻲ ﺟﺴﺪﻱ ﻣﺘﻨﺤﻲ‬.B
C. X-linked recessive .‫ ﻣﺘﻨﺤﻲ‬X ‫ ﻣﺮﺗﺒﻂ ﺏ‬.C
D. X-linked dominant .‫ ﻣﺴﻴﻄﺮ‬X‫ ﻣﺮﺗﺒﻂ ﺏ‬.D
B
Vous venez de confirmer le diagnostic d’une
Mucoviscidose chez un enfant de 3 ans. Les parents
s'inquiètent des futures grossesses. Vous leur expliquez
que le schéma de transmission génétique de la
Mucoviscidose est:
A. Autosomique dominante
B. Autosomique récessive
C. Récessif lié à l'X
D. Dominante liée à l'X
94. A 5-year-old girl diagnosed with pauciarticular juvenile ‫ ﺳﻨﻮﺍﺕ ﺗﻢ ﺗﺸﺨﻴﺼﻬﺎ ﺑﺎﻟﺘﻬﺎﺏ ﻣﻔﺎﺻﻞ ﻣﺠﻬﻮﻝ‬5 ‫ﻁﻔﻠﺔ ﻋﻤﺮﻫﺎ‬
idiopathic arthritis has a positive antinuclear antibody ‫ ﻭﻓﺤﺺ ﺍﻷﺿﺪﺍﺩ ﺍﻟﻤﻀﺎﺩﺓ ﻟﻠﻨﻮﻯ‬،‫ﺍﻟﺴﺒﺐ ﻳﻔﻌﻲ ﻗﻠﻴﻞ ﺍﻟﻤﻔﺎﺻﻞ‬
test. Which of the following would most likely be found in
:‫ ﺃﻱ ﻣﻦ ﺍﻟﺘﺎﻟﻲ ﻣﻦ ﺍﻷﺭﺟﺢ ﺃﻥ ﻳﺤﺪﺙ ﻟﻬﺬﻩ ﺍﻟﻤﺮﻳﻀﺔ؟‬.‫ﺇﻳﺠﺎﺑﻲ‬
this patient?
A. Pericarditis
.‫ ﺍﻟﺘﻬﺎﺏ ﺍﻟﺘﺎﻣﻮﺭ‬.A
B. Nephritis .‫ ﺍﻟﺘﻬﺎﺏ ﺍﻟﻜﻠﻰ‬.B
C. Uveitis .‫ ﺍﻟﺘﻬﺎﺏ ﺍﻟﻌﻨﺒﻴﺔ‬.C
D. Splenomegaly .‫ ﺿﺨﺎﻣﺔ ﻁﺤﺎﻝ‬.D
C
Une fille âgé de 5 ans et chez qui on a diagnostiqué une
arthrite idiopathique juvénile pauciarticulaire, a un test
positif aux anticorps antinucléaires.
Lequel des éléments suivants se trouverait le plus
probablement chez ce patient?
A. Péricardite
B. Néphrite
C. Uvéite
D. Splénomégalie

41 A
95. Which disease is characterized by the absence of T-cells, ‫ ﻭﻧﻘﺺ‬،‫ ﺗﻜﺰﺯ‬،T ‫ﻣﺎ ﻫﻮ ﺍﻟﻤﺮﺽ ﺍﻟﺬﻱ ﻳﺘﺼﻒ ﺑﻐﻴﺎﺏ ﺧﻼﻳﺎ‬
tetany, and hypocalcemia, with lowered cell-mediated ‫ ﻣﻊ ﺇﻧﺨﻔﺎﺽ ﻓﻲ ﺍﻟﻤﻨﺎﻋﺔ ﺍﻟﻤﺘﻮﺳﻄﺔ ﺑﻴﻦ‬،‫ﻛﺎﻟﺴﻴﻮﻡ ﺍﻟﺪﻡ‬
immunity?
:‫ﺍﻟﺨﻼﻳﺎ؟‬
A. Congenital thymic aplasia
B. Ulcerative colitis
.‫ ﻋﺪﻡ ﺗﻨﺴﺞ ﺍﻟﺘﻮﺗﺔ ﺍﻟﺨﻠﻘﻲ‬.A
C. Multiple sclerosis .‫ ﺍﻟﺘﻬﺎﺏ ﺍﻟﻘﻮﻟﻮﻥ ﺍﻟﻤﺘﻘﺮﺡ‬.B
D. Systemic lupus erythematosus .‫ ﺍﻟﺘﺼﻠﺐ ﺍﻟﻠﻮﻳﺤﻲ‬.C
.‫ ﺫﺋﺒﺔ ﺣﻤﺎﻣﻴﺔ ﺟﻬﺎﺯﻳﺔ‬.D A
Quelle maladie est caractérisée par l'absence de
lymphocytes T, de tétanie et d'hypocalcémie, avec une
diminution de l’immunité cellulaire?
A. Aplasie thymique congénitale
B. Colite ulcéreuse
C. Sclérose en plaques
D. Lupus érythémateux disséminé
96. Which of the following medications should be given for a ‫ﺃﻱ ﻣﻦ ﺍﻷﺩﻭﻳﺔ ﺍﻟﺘﺎﻟﻴﺔ ﻳﻨﺒﻐﻲ ﺃﻥ ﻳﻌﻄﻰ ﻟﻌﻼﺝ ﺁﻓﺔ ﻗﻠﺒﻴﺔ ﺧﻠﻘﻴﺔ‬
ductal-dependent cyanotic congenital heart lesion? :‫ﺯﺭﺍﻗﻴﺔ ﻣﻌﺘﻤﺪﺓ ﻋﻠﻰ ﻧﻔﻮﺫﻳﺔ ﺍﻟﻘﻨﺎﺓ ﺍﻟﺸﺮﻳﺎﻧﻴﺔ؟‬
A. Indomethacin infusion .‫ ﺗﺴﺮﻳﺐ ﺍﻹﻧﺪﻭﻣﻴﺘﺎﺳﻴﻦ‬.A
B. Prostaglandin E infusion
.E ‫ ﺗﺴﺮﻳﺐ ﺑﺮﻭﺳﺘﺎﻏﻼﻧﺪﻳﻦ‬.B
C. Adenosine infusion
D. Digoxin infusion .‫ ﺗﺴﺮﻳﺐ ﺍﻷﺩﻳﻨﻮﺯﻳﻦ‬.C
.‫ ﺗﺴﺮﻳﺐ ﺍﻟﺪﻳﺠﻮﻛﺴﻴﻦ‬.D
B
Lequel des médicaments suivants devrait être administré
en cas de cardiopathie congénitale cyanogènee et
dépendante de la perméabilité du canal artériel?
A. Administration d'Indométacine
B. Administration de Prostaglandine E
C. Administration d’Adénosine
D. Administration de Digoxine
97. Which of the following renal diseases is characterized by C3 ‫ﺃﻱ ﻣﻦ ﺍﻷﻣﺮﺍﺽ ﺍﻟﻜﻠﻮﻳﺔ ﺍﻟﺘﺎﻟﻴﺔ ﻳﺘﺼﻒ ﺑﻤﺴﺘﻮﻳﺎﺕ‬
low C3 levels? :‫ﻣﻨﺨﻔﻀﺔ؟‬
A. Focal segmental glomerulonephritis
.‫ ﺍﻟﺘﻬﺎﺏ ﻛﺒﻴﺒﺎﺕ ﺍﻟﻜﻠﻰ ﺍﻟﻘﻄﻌﻲ ﺍﻟﺒﺆﺭﻱ‬.A
B. IgA nephropathy
C. Acute poststreptococcal glomerulonephritis
.IgA ‫ ﺍﻋﺘﻼﻝ ﻛﻠﻮﻱ‬.B
D. Nephrotic syndrome ‫ ﺍﻟﺘﻬﺎﺏ ﻛﺒﻴﺒﺎﺕ ﺍﻟﻜﻠﻰ ﺍﻟﺤﺎﺩ ﺍﻟﺘﺎﻟﻲ ﻟﻺﺻﺎﺑﺔ‬.C
.‫ﺑﺎﻟﻌﻘﺪﻳﺎﺕ‬ C
Laquelle des maladies rénales suivantes est caractérisée .‫ ﻣﺘﻼﺯﻣﺔ ﻧﻔﺮﻭﻧﻴﺔ‬.D
par de faibles taux de C3?
A. Glomérulonéphrite segmentaire et focale
B. Néphropathie à IgA
C. Glomérulonéphrite aiguë post-streptococcique
D. Syndrome néphrotique
98. Which of the following is a reasonable diagnostic test in ‫ﺃﻱ ﻣﻦ ﺍﻟﺘﺎﻟﻲ ﻫﻮ ﺍﺧﺘﺒﺎﺭ ﺗﺸﺨﻴﺼﻲ ﻣﻨﻄﻘﻲ ﻋﻨﺪ ﻣﺮﻳﺾ‬
patient thought to potentially have Duchenne muscular :‫ﻳﺤﺘﻤﻞ ﺃﻥ ﻳﻜﻮﻥ ﻣﺼﺎﺑﺎ ﺑﻀﻤﻮﺭ ﺩﻭﺷﻦ ﺍﻟﻌﻀﻠﻲ؟‬
dystrophy? .‫ ﻣﺴﺘﻮﻯ ﻛﺮﻳﺎﺗﻴﻦ ﻓﻮﺳﻔﻮﻛﻴﻨﺎﺯ ﺍﻟﻤﺼﻠﻲ‬.A
A. Serum creatine phosphokinase (CPK) levels
.‫ ﺗﺨﻄﻴﻂ ﻋﻀﻠﺔ ﻛﻬﺮﺑﺎﺋﻲ‬.B
B. Electromyography
C. Nerve conduction tests .‫ ﻓﺤﻮﺹ ﺍﻟﺘﻮﺻﻴﻞ ﺍﻟﻌﺼﺒﻲ‬.C
D. Muscle biopsy .‫ ﺧﺰﻋﺔ ﻣﻦ ﺍﻟﻌﻀﻞ‬.D
A
Lequel des tests suivants constitue un test de diagnostic
raisonnable chez un patient susceptible de développer
une dystrophie musculaire de Duchenne?
A. Niveaux sériques de créatine phosphokinase (CPK)
B. Électromyographie
C. Tests de conduction nerveuse
D. Biopsie musculaire

42 A
99. A two-week-old baby has been constipated since birth, ‫ ﻳﻤﺮﺭ‬،‫ﻁﻔﻞ ﻋﻤﺮﻩ ﺃﺳﺒﻮﻋﻴﻦ ﻭﻫﻮ ﻓﻲ ﺣﺎﻟﺔ ﺇﻣﺴﺎﻙ ﻣﻨﺬ ﺍﻟﻮﻻﺩﺓ‬
passing only small stools. Vomiting, occasionally bile- ‫ ﻓﻲ‬،‫ ﻳﻼﺣﻆ ﺃﺣﻴﺎﻧﺎ ﻣﻊ ﺻﻔﺮﺍء‬،‫ ﺑﺪﺃ ﺍﻟﺘﻘﻴﺆ‬.‫ﻓﻘﻂ ﻗﻠﻴﻞ ﻣﻦ ﺍﻟﺒﺮﺍﺯ‬
stained, started in the first week of life and is becoming ‫ ﻋﻨﺪ‬.‫ﺍﻷﺳﺒﻮﻉ ﺍﻷﻭﻝ ﻣﻦ ﺍﻟﻮﻻﺩﺓ ﻭﻟﻘﺪ ﺃﺻﺒﺢ ﻳﺘﺰﺍﻳﺪ ﺑﺸﺪﺓ‬
increasingly severe. On examination, there is definite
.‫ ﻫﻨﺎﻙ ﺍﻧﺘﻔﺎﺥ ﺑﻄﻦ ﻣﺤﺪﺩ ﺣﻴﺚ ﺗﻔﻴﺪ ﻭﺍﻟﺪﺗﻪ ﺑﺄﻧﻪ ﻳﺘﺰﺍﻳﺪ‬،‫ﺍﻟﻔﺤﺺ‬
abdominal distension which the mother says is increasing.
The most likely diagnosis is:
:‫ﺇﻥ ﺍﻟﺘﺸﺨﻴﺺ ﺍﻷﻛﺜﺮ ﺗﺮﺟﻴﺤﺎ ﻫﻮ‬
A. Hypertrophic pyloric stenosis .‫ ﺗﻀﻴﻖ ﺍﻟﺒﻮﺍﺏ ﺍﻟﻀﺨﺎﻣﻲ‬.A
B. Congenital duodenal atresia .‫ ﺭﺗﻖ ﺍﻹﺛﻨﺎ ﻋﺸﺮﻱ ﺍﻟﺨﻠﻘﻲ‬.B
C. Hirschsprung's disease .‫ ﺩﺍء ﻫﺮﺷﺴﺒﺮﻭﻧﻎ‬.C
D. Gastro-esophageal reflux .‫ ﺍﺭﺗﺠﺎﻉ ﻣﻌﺪﻱ ﻣﺮﻳﺌﻲ‬.D
C
Un nouveau-né de 2 semaines de vie, est constipé depuis
la naissance et ne passant que de petites selles. Les
vomissements, parfois marqués par la bile, ont commencé
dans la première semaine de la vie et se sont aggravés. À
l'examen, il y a une distension abdominale qui augmente
selon la mère. Le diagnostic le plus probable est:
A. Sténose hypertrophique du pylore
B. Atrésie duodénale congénitale
C. Maladie de Hirschsprung
D. Reflux gastro-oesophagien
100. What organism is most likely present in a patient with ‫ﻣﺎ ﻫﻮ ﺍﻟﻌﺎﻣﻞ ﺍﻟﻤﺴﺒﺐ ﺍﻷﻛﺜﺮ ﺗﺮﺟﻴﺤﺎ ﻋﻨﺪ ﻣﺮﻳﺾ ﻳﻌﺎﻧﻲ ﻣﻦ‬
fever, conjunctivitis, rhinorrhea, and pharyngitis? :‫ ﺳﻴﻼﻥ ﺃﻧﻔﻲ ﻭﺍﻟﺘﻬﺎﺏ ﺑﻠﻌﻮﻡ؟‬،‫ ﺍﻟﺘﻬﺎﺏ ﻣﻠﺘﺤﻤﺔ‬،‫ﺣﺮﺍﺭﺓ‬
A. Adenovirus .‫ ﻓﻴﺮﻭﺱ ﺍﻟﻐﺪﻳﺔ‬.A
B. Rhinovirus
.‫ ﻓﻴﺮﻭﺱ ﺍﻷﻧﻔﻴﺔ‬.B
C. Group A β hemolytic Streptococcus
D. Haemophilus influenza type B . A‫ ﺍﻟﻌﻘﺪﻳﺎﺕ ﺍﻟﺤﺎﻟﺔ ﻟﻠﺪﻡ ﺑﻴﺘﺎ ﻣﻦ ﺍﻟﻤﺠﻤﻮﻋﺔ‬.C
.B ‫ ﺍﻟﻤﺴﺘﺪﻣﻴﺔ ﺍﻟﻨﺰﻟﻴﺔ ﻧﻤﻂ‬.D
A
Quel organisme est le plus probablement présent chez un
patient souffrant de fièvre, de conjonctivite, de rhinorrhée
et de pharyngite?
A. Adénovirus
B. Rhinovirus
C. Streptocoque β hémolytique du groupe A
D. Haemophilus influenza type B
101. Which of the following is not at an increased risk for the ‫ﺃﻱ ﻣﻦ ﺍﻟﺘﺎﻟﻲ ﻟﻴﺲ ﻓﻲ ﺧﻄﺮ ﻣﺘﺰﺍﻳﺪ ﻋﻨﺪ ﺗﻄﻮﺭ ﺍﻟﺘﻬﺎﺏ ﺷﻐﺎﻑ‬
development of infective endocarditis? :‫ﺍﻟﻘﻠﺐ ﺍﻹﻧﺘﺎﻧﻲ؟‬
A. Atrial septal defect .‫ ﻋﻴﺐ ﺍﻟﺤﺎﺟﺰ ﺍﻷﺫﻳﻨﻲ‬.A
B. Aortic insufficiency
.‫ ﻗﺼﻮﺭ ﺍﻷﺑﻬﺮ‬.B
C. Aortic stenosis
D. Mitral regurgitation .‫ ﺗﻀﻴﻖ ﺍﻷﺑﻬﺮ‬.C
.‫ ﻗﻠﺲ ﺗﺎﺟﻲ‬.D A
Lequel des pathologies qui suivent ne présente pas un
risque de développer une endocardite infectieuse?
A. Communication interauriculaire
B. Insuffisance aortique
C. Sténose aortique
D. Régurgitation mitrale

43 A
102. A one year old boy is brought to the Emergency ‫ﻁﻔﻞ ﻋﻤﺮﻩ ﺳﻨﺔ ﻭﺍﺣﺪﺓ ﺃﺣﻀﺮ ﺇﻟﻰ ﻗﺴﻢ ﺍﻟﻄﻮﺍﺭﺉ ﺑﺴﺒﺐ‬
Department with a 3 day history of rhinorrhoea, wheeze, ‫ ﺣﺮﺍﺭﺓ‬،‫ ﺃﺯﻳﺰ‬،‫ ﺃﻳﺎﻡ ﻋﻠﻰ ﺇﺻﺎﺑﺘﻪ ﺑﺴﻴﻼﻥ ﺃﻧﻔﻲ‬3 ‫ﻣﺮﻭﺭ‬
fever and cough. The most likely diagnosis is:
:‫ ﺇﻥ ﺍﻟﺘﺸﺨﻴﺺ ﺍﻷﻛﺜﺮ ﺗﺮﺟﻴﺤﺎ ﻫﻮ‬.‫ﻭﺳﻌﺎﻝ‬
A. Asthma
B. Bronchiolitis
.‫ ﺍﻟﺮﺑﻮ‬.A
C. Sinusitis .‫ ﺍﻟﺘﻬﺎﺏ ﺍﻟﻘﺼﻴﺒﺎﺕ‬.B
D. Pertussis infection .‫ ﺍﻟﺘﻬﺎﺏ ﺍﻟﺠﻴﻮﺏ ﺍﻷﻧﻔﻴﺔ‬.C
.‫ ﺍﻟﺸﺎﻫﻮﻕ‬.D
B
Un garçon, âgé de un an, est amené au service des
urgences avec une histoire de rhinorrhée, de respiration
sifflante, de fièvre et de toux depuis 3 jours. Le diagnostic
le plus probable est:
A. Asthme
B. Bronchiolite
C. Sinusite
D. Coqueluche
103. An 18 month old is brought to the Emergency Department ‫ ﺃﺣﻀﺮ ﺇﻟﻰ ﻗﺴﻢ ﺍﻟﻄﻮﺍﺭﺉ ﺑﺘﺸﺨﻴﺺ‬،‫ ﺷﻬﺮﺍ ﻣﻦ ﺍﻟﻌﻤﺮ‬18
with a diagnosis of CROUP. Which of the following is :‫ ﺃﻱ ﻣﻦ ﺍﻟﺘﺎﻟﻲ ﺻﺤﻴﺢ؟‬.‫ﻟﻠﺨﺎﻧﻮﻕ‬
correct?
.‫ ﻻ ﻳﺤﺪﺙ ﺍﻟﺼﺮﻳﺮ ﻓﻲ ﺍﻟﺤﺎﻻﺕ ﺍﻟﺨﻔﻴﻔﺔ‬.A
A. Stridor does not occur in mild cases
B. Inspiratory stridor is present .‫ ﺍﻟﺼﺮﻳﺮ ﺍﻟﺸﻬﻴﻘﻲ ﻣﻮﺟﻮﺩ‬.B
C. Salbutamol decreases hospital length of stay .‫ ﺍﻟﺴﺎﻟﺒﻮﺗﺎﻣﻮﻝ ﻳﻘﻠﻞ ﻣﻦ ﻓﺘﺮﺓ ﺍﻟﺒﻘﺎء ﻓﻲ ﺍﻟﻤﺴﺘﺸﻔﻰ‬.C
D. Children receiving adrenaline nebuliser must be ‫ ﻳﺠﺐ ﺇﺩﺧﺎﻝ ﺍﻷﻁﻔﺎﻝ ﺍﻟﺬﻳﻦ ﻳﺴﺘﺨﺪﻣﻮﻥ ﺃﺩﺭﻳﻨﺎﻟﻴﻦ‬.D
admitted .‫ﺭﺫﺍﺫﻱ ﺇﻟﻰ ﺍﻟﻤﺴﺘﺸﻔﻰ‬
B
Un enfant de 18 mois est amené au service des urgences
avec un diagnostic de CROUP. Parmi les propositions qui
suivent, laquelle est correcte?
A. Le Stridor ne s’oberve pas dans les cas minimes
B. Le stridor inspiratoire est présent
C. La Salbutamol diminue la durée du séjour à l'hôpital
D. Les enfants recevant l'Adrénaline en nébuliseur
doivent être admis
104. A 2½ year old boy appears with intermittent loose stools ‫ﻁﻔﻞ ﻋﻤﺮﻩ ﺳﻨﺘﺎﻥ ﻭﻧﺼﻒ ﻳﻌﺎﻧﻲ ﻣﻦ ﺑﺮﺍﺯ ﻓﻀﻔﺎﺽ ﻣﺘﻘﻄﻊ‬
for the past 1 month. Stools typically occur during the day ‫ ﻳﺤﺪﺙ ﺍﻟﺘﺒﺮﺯ ﻋﺎﺩﺓ ﺃﺛﻨﺎء ﺍﻟﻨﻬﺎﺭ ﻭﻟﻴﺲ ﺧﻼﻝ‬.‫ﻣﻨﺬ ﺷﻬﺮ ﻣﻀﻰ‬
and not overnight. The boy is otherwise healthy. The ‫ ﺍﻟﻨﻤﻮ‬.‫ ﻋﺪﺍ ﻋﻦ ﺫﻟﻚ ﻓﺈﻥ ﺍﻟﻄﻔﻞ ﻳﺘﻤﺘﻊ ﺑﺼﺤﺔ ﺟﻴﺪﺓ‬.‫ﺍﻟﻠﻴﻞ‬
growth and development are normal. The most likely
:‫ ﺇﻥ ﺍﻟﺘﺸﺨﻴﺺ ﺍﻷﻛﺜﺮ ﺗﺮﺟﻴﺤﺎ ﻫﻮ‬.‫ﻭﺍﻟﺘﻄﻮﺭ ﻁﺒﻴﻌﻲ‬
diagnosis is:
A. Chronic enteritis
.‫ ﺍﻟﺘﻬﺎﺏ ﻣﻌﻮﻱ ﻣﺰﻣﻦ‬.A
B. Rotavirus enteritis .‫ ﺍﻟﺘﻬﺎﺏ ﻣﻌﻮﻱ ﺑﺴﺒﺐ ﻓﻴﺮﻭﺱ ﺍﻟﺮﻭﺗﺎ‬.B
C. Salmonella enteritis .‫ ﺍﻟﺘﻬﺎﺏ ﻣﻌﻮﻱ ﺑﺴﺒﺐ ﺍﻟﺴﻠﻤﻮﻧﻴﻠﻼ‬.C
D. Food allergy. .‫ ﺗﺤﺴﺲ ﻏﺬﺍﺋﻲ‬.D
E. None of the above .‫ ﻛﻞ ﺍﻷﺟﻮﺑﺔ ﺃﻋﻼﻩ ﻏﻴﺮ ﺻﺤﻴﺤﺔ‬.E
E
Un garçon de 2 ans et demi présente des selles molles
intermittentes depuis un mois. Les selles sont
généralement présentes pendant la journée et absentes
durant la nuit. Le garçon est par ailleurs en bonne santé.
La croissance et le développement sont normaux. Le
diagnostic le plus probable est:
A. Gastro-Entérite chronique
B. Gastro-Entérite à rotavirus
C. Gastro-Entérite à Salmonella
D. Allergie alimentaire.
E. Toutes les propositions sont fausses

44 A
105. A full term male infant appears jaundiced during the first ‫ ﺳﺎﻋﺔ ﻣﻦ‬24 ‫ﺭﺿﻴﻊ ﺫﻛﺮ ﻧﺎﺿﺞ ﻳﻈﻬﺮ ﻋﻠﻴﻪ ﻳﺮﻗﺎﻥ ﺃﺛﻨﺎء ﺃﻭﻝ‬
24 hours of life due to AO incompatibility and Coombs ‫ ﻛﻤﺎ ﺃﻥ ﻧﺘﺎﺋﺞ ﻛﻮﻣﺒﺲ ﻛﺎﻧﺖ‬AO ‫ﺍﻟﻮﻻﺩﺓ ﺑﺴﺒﺐ ﻋﺪﻡ ﺗﻮﺍﻓﻖ‬
positive results. His serum bilirubin level is 35 mg/dL.
Two exchange transfusions are performed. The most
‫ ﻟﻘﺪ ﺗﻢ‬.‫ ﺩﺳﻞ‬/‫ ﻣﻠﻎ‬35 ‫ ﻣﺴﺘﻮﻯ ﺑﻴﻠﻴﺮﻭﺑﻴﻦ ﺍﻟﻤﺼﻞ‬.‫ﺇﻳﺠﺎﺑﻴﺔ‬
likely morbidity is: :‫ ﺍﻟﺨﻄﺮ ﺍﻷﻛﺜﺮ ﺗﺮﺟﻴﺤﺎ ﻫﻮ‬.‫ﺇﺟﺮﺍء ﻧﻘﻞ ﺩﻡ ﻣﺮﺗﻴﻦ‬
A. Choreoathetotic cerebral palsy .‫ ﺷﻠﻞ ﺩﻣﺎﻏﻲ ﻛﻨﻌﻲ ﺭﻗﺼﻲ‬.A
B. Spastic diplegia .‫ ﺷﻠﻞ ﻣﺰﺩﻭﺝ ﺗﺸﻨﺠﻲ‬.B
C. Hydrocephalus .‫ ﻣﻮﻩ ﺍﻟﺮﺃﺱ‬.C
D. Spastic quadriplegia .‫ ﺷﻠﻞ ﺭﺑﺎﻋﻲ ﺗﺸﻨﺠﻲ‬.D
A
Un enfant né à terme et de sexe masculin présente un
ictère au cours des 24 premières heures de vie en raison
d'une incompatibilité AO et test de Coombs positif. Son
taux de bilirubine sérique est de 35 mg/dL. Deux
exsanguino-transfusions sont réalisées. Le risque de
séquelles le plus probable est:
A. Paralysie cérébrale choréoathétotique
B. Diplégie spastique
C. Hydrocéphalie
D. Tétraplégie spastique
106. Earliest sign of pathological gastroesophageal reflux in ‫ﺍﻹﺷﺎﺭﺓ ﺍﻟﻤﺒﻜﺮﺓ ﻟﻼﺭﺗﺠﺎﻉ ﺍﻟﻤﻌﺪﻱ ﺍﻟﻤﺮﻳﺌﻲ ﺍﻟﻤﺮﺿﻲ ﻋﻨﺪ‬
infants is? :‫ﺍﻷﻁﻔﺎﻝ ﺍﻟﺮﺿﻊ ﻫﻲ‬
A. Upper Gastro intestinal bleeding .‫ ﻧﺰﻳﻒ ﻓﻲ ﺍﻟﺠﻬﺎﺯ ﺍﻟﻬﻀﻤﻲ ﺍﻟﻌﻠﻮﻱ‬.A
B. Respiratory symptoms
.‫ ﺃﻋﺮﺍﺽ ﺗﻨﻔﺴﻴﺔ‬.B
C. Oesophageal stricture
D. Postprandial regurgitation .‫ ﺗﻀﻴﻖ ﺍﻟﻤﺮﻳﺊ‬.C
.‫ ﻗﻠﺲ ﺑﻌﺪ ﺍﻷﻛﻞ‬.D B
Les premiers signes de reflux gastro-oesophagien
pathologique chez les nourrissons sont?
A. Saignements Gastro Intestianles supérieurs
B. Symptômes respiratoires
C. Sténose de l'œsophage
D. Régurgitation postprandiale
107. The coagulation profile in a 13-year old girl with ‫ ﻋﺎﻣﺎ ﺗﺸﻜﻮ ﻣﻦ ﻏﺰﺍﺭﺓ ﻁﻤﺚ‬13 ‫ﺇﻥ ﻣﻠﻒ ﺍﻟﺘﺨﺜﺮ ﻟﻔﺘﺎﺓ ﻋﻤﺮﻫﺎ‬
Menorrhagia having von Willebrand disease is: :‫ﻭﺗﻌﺎﻧﻲ ﻣﻦ ﺩﺍء ﻓﻮﻥ ﻭﻳﻠﺒﺮﺍﻧﺪ ﻫﻮ‬
A. Isolated prolonged PTT with a normal PT
.‫ﻁﺒﻴﻌﻲ‬PT ‫ ﻣﻌﺰﻭﻝ ﻁﻮﻳﻞ ﻣﻊ‬PTT .A
B. Isolated prolonged PT with a normal PTT
C. Prolongation of both PT and PTT .‫ ﻁﺒﻴﻌﻲ‬PTT ‫ ﻣﻌﺰﻭﻝ ﻁﻮﻳﻞ ﻣﻊ‬PT .B
D. Prolongation of thrombin time .PTT‫ ﻭ‬PT ‫ ﺇﻁﺎﻟﺔ ﻓﻲ ﺯﻣﻦ‬.C
.‫ ﺇﻁﺎﻟﺔ ﻓﻲ ﺯﻣﻦ ﺍﻟﺜﺮﻭﻣﺒﻴﻦ‬.D A
Le profil de coagulation chez une jeune fille de 13 ans
atteinte de ménorragie et atteinte de la maladie de von
Willebrand est:
A. PTT prolongé isolé avec un PT normal
B. PT prolongé isolé avec un PTT normal
C. Prolongation des deux PT et PTT
D. Prolongation du temps de thrombine

45 A
108. An affected male infant born to normal parents could be ‫ﺍﻟﻤﻮﻟﻮﺩ ﺍﻟﺬﻛﺮ ﺍﻟﻤﺼﺎﺏ ﻷﺑﻮﻳﻦ ﻁﺒﻴﻌﻴﻴﻦ ﻳﻤﻜﻦ ﺃﻥ ﻳﻜﻮﻥ ﻣﺜﺎﻻ‬
an example of all of the following hereditary :‫ ﻣﺎ ﻋﺪﺍ‬،‫ﻟﺠﻤﻴﻊ ﺍﻷﻣﺮﺍﺽ ﺍﻟﺘﺎﻟﻴﺔ ﺍﻟﻤﻨﺘﻘﻠﺔ ﻭﺭﺍﺛﻴﺎ‬
transmissions, except: .‫ ﺍﺿﻄﺮﺍﺏ ﺻﺒﻐﻲ ﺟﺴﺪﻱ ﻣﺴﻴﻄﺮ‬.A
A. An Autosomal dominant disorder
.‫ ﺍﺿﻄﻄﺮﺍﺏ ﺻﺒﻐﻲ ﺟﺴﺪﻱ ﻣﺘﻨﺤﻲ‬.B
B. An Autosomal recessive disorder
C. A polygenic disorder .‫ ﺍﺿﻄﺮﺍﺏ ﻣﺘﻌﺪﺩ ﺍﻟﺠﻴﻨﺎﺕ‬.C
D. B+C .C + B .D
A
Un nourrisson de sexe masculin atteint de parents
normaux pourrait être un exemple de toutes les
transmissions héréditaires qui suivent, sauf
A. Un trouble autosomique dominant
B. Un trouble autosomique récessif
C. Un trouble polygénique
D. B+C
109. A 1 month old boy is referred for failure to thrive. On ‫ ﻋﻨﺪ‬.‫ﻁﻔﻞ ﻋﻤﺮﻩ ﺷﻬﺮ ﺗﻤﺖ ﺇﺣﺎﻟﺘﻪ ﺑﺴﺒﺐ ﺗﺄﺧﺮ ﻓﻲ ﺍﻟﻨﻤﻮ‬
examination, he shows feature of congestive heart failure. ‫ ﺍﻟﻨﺒﺾ ﺍﻟﻔﺨﺬﻱ‬.‫ ﺗﺒﻴﻦ ﺃﻥ ﻟﺪﻳﻪ ﻗﺼﻮﺭ ﻗﻠﺐ ﺍﺣﺘﻘﺎﻧﻲ‬،‫ﺍﻟﻔﺤﺺ‬
The femoral pulses are weak as compared to radial pulses ‫ ﺇﻥ ﺍﻟﺘﺸﺨﻴﺺ‬.‫ﺿﻌﻴﻒ ﺑﺎﻟﻤﻘﺎﺭﻧﺔ ﻣﻊ ﺍﻟﻨﺒﺾ ﺍﻟﻜﻌﺒﺮﻱ ﺍﻟﻮﺍﺿﺢ‬
well palpable. The most likely clinical diagnosis is:
:‫ﺍﻟﺴﺮﻳﺮﻱ ﺍﻷﻛﺜﺮ ﺗﺮﺟﻴﺤﺎ ﻫﻮ‬
A. Congenital aortic stenosis
B. Coarctation of aorta
.‫ ﺗﻀﻴﻖ ﺃﺑﻬﺮ ﺧﻠﻘﻲ‬.A
C. Patent ductus arteriosus .‫ ﺗﻀﻴﻖ ﺍﻷﺑﻬﺮ‬.B
D. Large Atrial Septal Defect .‫ ﺍﻟﻘﻨﺎﺓ ﺍﻟﺸﺮﻳﺎﻧﻴﺔ ﺍﻟﺴﺎﻟﻜﺔ‬.C
.‫ ﻋﻴﺐ ﻛﺒﻴﺮ ﻓﻲ ﺍﻟﺤﺎﺟﺰ ﺍﻷﺫﻳﻨﻲ‬.D
B
Un garçon âgé de un mois est référé pour retard de
croissance. À l'examen, il présente une insuffisance
cardiaque congestive. Les pouls fémoraux sont faibles par
rapport aux pouls radiaux bien perçus. Le diagnostic
clinique le plus probable est:
A. Sténose aortique congénitale
B. Coarctation de l'aorte
C. Persistance du canal artériel
D. Large Communication Inter Auriculaire
110. The earliest indicator of response after starting iron in a 6- ‫ﺇﻥ ﺍﻟﻤﺆﺷﺮﺍﻟﻤﺨﺒﺮﻱ ﺍﻟﻤﺒﻜﺮ ﺑﻌﺪ ﺍﻟﺒﺪء ﺑﺎﻟﻤﻌﺎﻟﺠﺔ ﺑﺎﻟﺤﺪﻳﺪ ﻋﻨﺪ‬
year-old girl with iron :‫ ﺳﻨﻮﺍﺕ ﻭﺗﻌﺎﻧﻲ ﻣﻦ ﻋﻮﺯ ﺍﻟﺤﺪﻳﺪ ﻫﻮ‬6 ‫ﻓﺘﺎﺓ ﻋﻤﺮﻫﺎ‬
deficiency is:
.‫ ﺯﻳﺎﺩﺓ ﻓﻲ ﺗﻌﺪﺍﺩ ﺍﻟﺸﺒﻜﻴﺎﺕ‬.A
A. Increased reticulocyte count
B. Increased hemoglobin
.‫ ﺯﻳﺎﺩﺓ ﺍﻟﻬﻴﻤﻮﻏﻠﻮﺑﻴﻦ‬.B
C. Increased ferritin .‫ ﺯﻳﺎﺩﺓ ﺍﻟﻔﻴﺮﺭﺗﻴﻦ‬.C
D. Increased serum iron .‫ ﺯﻳﺎﺩﺓ ﻓﻲ ﺣﺪﻳﺪ ﺍﻟﻤﺼﻞ‬.D
A
L’indicateur de laboratoire le plus précoce après le début
du traitement au fer chez une fillette de 6 ans présentant
une carence en fer est:
A. Augmentation du nombre de réticulocytes
B. Augmentation de l'hémoglobine
C. Augmentation de la ferritine
D. Augmentation du fer sérique

46 A
111. Which of the following pathologies is a pediatric :‫ﺃﻱ ﻣﻦ ﺍﻷﻣﺮﺍﺽ ﺍﻟﺘﺎﻟﻴﺔ ﻫﻲ ﺣﺎﻟﺔ ﻁﺐ ﺃﻁﻔﺎﻝ ﻁﺎﺭﺋﺔ؟‬
emergency? .‫ ﺍﻟﺘﻬﺎﺏ ﻟﻮﺯﺗﻴﻦ ﺟﺮﺛﻮﻣﻲ‬.A
A. Bacterial tonsillitis .‫ ﺍﻟﺘﻬﺎﺏ ﺍﻟﺠﻴﻮﺏ ﺍﻷﻧﻔﻴﺔ‬.B
B. Sinusitis
C. Acute Epiglottitis
.‫ ﺍﻟﺘﻬﺎﺏ ﻟﺴﺎﻥ ﺍﻟﻤﺰﻣﺎﺭ ﺍﻟﺤﺎﺩ‬.C
D. None of the above .‫ ﻛﻞ ﺍﻷﺟﻮﺑﺔ ﺃﻋﻼﻩ ﻏﻴﺮ ﺻﺤﻴﺤﺔ‬.D
C
Laquelle des pathologies suivantes est une urgence
pédiatrique?
A. Amygdalite bactérienne
B. Sinusite
C. Épiglottite aiguë
D. Toutes les propositions sont fausses
112. The small intestinal biopsy is indicated in: :‫ﻳﻄﻠﺐ ﺃﺧﺬ ﺧﺰﻋﺔ ﻣﻦ ﺍﻷﻣﻌﺎء ﺍﻟﺪﻗﻴﻘﺔ ﻓﻲ ﺣﺎﻟﺔ‬
A. Intolerance to cow's milk proteins .‫ ﻋﺪﻡ ﺗﺤﻤﻞ ﺑﺮﻭﺗﻴﻨﺎﺕ ﺣﻠﻴﺐ ﺍﻟﺒﻘﺮ‬.A
B. Celiac disease .‫ ﻣﺮﺽ ﺑﻄﻨﻲ‬.B
C. Lactose intolerance
D. Cystic fibrosis
.‫ ﻋﺪﻡ ﺗﺤﻤﻞ ﺍﻟﻼﻛﺘﻮﺯ‬.C
.‫ ﺍﻟﺘﻠﻴﻒ ﺍﻟﻜﻴﺴﻲ‬.D B
La biopsie de l'intestin grêle est indiquée dans:
A. Intolérance aux protéines de lait de vache
B. Maladie coeliaque
C. Intolérance au lactose
D. Mucoviscidose
113. During the first 8 years of life, all the following vaccines ‫ ﻛﻞ ﺍﻟﻠﻘﺎﺣﺎﺕ ﺍﻟﺘﺎﻟﻴﺔ‬،‫ﺧﻼﻝ ﺃﻭﻝ ﺛﻤﺎﻧﻲ ﺳﻨﻮﺍﺕ ﻣﻦ ﺍﻟﺤﻴﺎﺓ‬
should be given at least 3 times, except: :‫ ﻣﺎﻋﺪﺍ‬،‫ ﻣﺮﺍﺕ‬3 ‫ﻳﻨﺒﻐﻲ ﺇﻋﻄﺎءﻫﺎ ﻋﻠﻰ ﺍﻷﻗﻞ‬
A. Measles, Mumps and Rubella vaccine
.‫ ﺍﻟﺤﺼﺒﺔ ﺍﻷﻟﻤﺎﻧﻴﺔ‬،‫ ﺍﻟﻨﻜﺎﻑ‬،‫ ﻟﻘﺎﺡ ﺍﻟﺤﺼﺒﺔ‬.A
B. Diphtheria, Tetanus toxoids and Pertussis vaccine
C. Hepatitis B .‫ ﺫﻳﻔﺎﻥ ﺍﻟﻜﺰﺍﺯ ﻭﺍﻟﺸﺎﻫﻮﻕ‬،‫ ﻟﻘﺎﺡ ﺍﻟﺨﺎﻧﻮﻕ‬.B
D. L’Haemophilus influenzae type b .B ‫ ﻟﻘﺎﺡ ﺍﻟﺘﻬﺎﺏ ﺍﻟﻜﺒﺪ ﺍﻟﻔﻴﺮﻭﺳﻲ‬.C
. b‫ ﻟﻘﺎﺡ ﺍﻟﻤﺴﺘﺪﻣﻴﺔ ﺍﻟﻨﺰﻟﻴﺔ ﻧﻤﻂ‬.D
A
Au cours des 8 premières années de vie, tous les vaccins
suivants devraient être administrés au moins trois fois, à
l'exception de:
A. La rougeole, les oreillons et la rubéole
B. La diphtérie, le tétanos et le vaccin contre la coqueluche
C. L'hépatite B
D. Haemophilus influenzae de type b
114. The most common cause of death between 3 and 6 months :‫ ﺃﺷﻬﺮ ﻣﻦ ﺍﻟﻌﻤﺮ ﻫﻮ‬6‫ ﻭ‬3 ‫ﺇﻥ ﺍﻟﺴﺒﺐ ﺍﻟﺮﺋﻴﺴﻲ ﻟﻠﻮﻓﺎﺓ ﻣﺎ ﺑﻴﻦ‬
of age is: .‫ ﺍﺑﻴﻀﺎﺽ ﺍﻟﺪﻡ‬.A
A. Leukemia
.‫ ﻣﺮﺽ ﻗﻠﺐ ﺧﻠﻘﻲ‬.B
B. Congenital Heart Disease
C. Renal disease .‫ ﺩﺍء ﻛﻠﻮﻱ‬.C
D. Bacterial meningitis .‫ ﺍﻟﺘﻬﺎﺏ ﺳﺤﺎﻳﺎ ﺟﺮﺛﻮﻣﻲ‬.D
E. Sudden Infant Death Syndrome .‫ ﻣﺘﻼﺯﻣﺔ ﻣﻮﺕ ﺍﻟﺮﺿﻴﻊ ﺍﻟﻤﻔﺎﺟﺊ‬.E
E
La cause la plus fréquente de décès entre 3 et 6 mois de
vie est:
A. La leucémie
B. Les malformations cardiaques
C. Maladies rénales
D. La méningite bactérienne
E. Mort subite inexpliquée du nourrisson

47 A
115. Treatment of Acetaminophen Overdose is: :‫ﻋﻼﺝ ﺍﻟﺠﺮﻋﺔ ﺍﻟﺰﺍﺋﺪﺓ ﻣﻦ ﺍﻷﺳﻴﺘﺎﻣﻴﻨﻮﻓﻴﻦ ﻫﻮ‬
A. Naloxone. .‫ ﻧﺎﻟﻮﻛﺴﻮﻥ‬.A
B. N-acetylcysteine. .‫ﺍﺳﺘﻴﻞ ﺳﻴﺴﺘﺌﻴﻦ‬-‫ ﻥ‬.B
C. Atropine.
D. All of the above.
.‫ ﺃﺗﺮﻭﺑﻴﻦ‬.C
E. None of the above. .‫ ﻛﻞ ﺍﻷﺟﻮﺑﺔ ﺃﻋﻼﻩ ﺻﺤﻴﺤﺔ‬.D
.‫ ﻛﻞ ﺍﻷﺟﻮﺑﺔ ﺃﻋﻼﻩ ﻏﻴﺮ ﺻﺤﻴﺤﺔ‬.E B
Le traitement du surdosage en acétaminophène est:
A. Naloxone.
B. N-acétylcystéine.
C. Atropine.
D. Toutes les propositions sont justes.
E. Toutes les propositions sont fausses
116. In infants, Dehydration is considered as moderate if the ‫ ﻳﻌﺘﺒﺮ ﺍﻟﺘﺠﻔﺎﻑ ﻣﺘﻮﺳﻂ ﺇﺫﺍ ﻛﺎﻥ ﻓﻘﺪﺍﻥ ﺍﻟﻮﺯﻥ‬،‫ﻋﻨﺪ ﺍﻟﺮﺿﻊ‬
acute weight loss is: :‫ﺍﻟﺤﺎﺩ ﻫﻮ‬
A. Between 3 and 5%. .%5‫ ﻭ‬3 ‫ ﺑﻴﻦ‬.A
B. Between 5 and 10%.
C. Between 11 and 15%.
.% 10‫ ﻭ‬5 ‫ ﺑﻴﻦ‬.B
D. > 15%. .%15‫ ﻭ‬11 ‫ ﺑﻴﻦ‬.C
.%15 ‫ ﺃﻛﺒﺮ ﻣﻦ‬.D B
Chez le nourrisson, la déshydratation est considérée
comme modérée si la perte de poids aiguë est:
A. Entre 3 et 5%.
B. Entre 5 et 10%.
C. Entre 11 et 15%.
D. > 15%.
117. Which of the following propositions is (are) ‫ﺃﻱ ﻣﻦ ﺍﻟﻌﺒﺎﺭﺍﺕ ﺍﻟﺘﺎﻟﻴﺔ ﻫﻲ )ﻫﻢ( ﻣﻦ ﻣﻀﺎﻋﻔﺎﺕ ﺍﻟﺘﻬﺎﺏ‬
complication(s) of otitis media? :‫ﺍﻷﺫﻥ ﺍﻟﻮﺳﻄﻰ؟‬
A. Meningitis. .‫ ﺍﻟﺘﻬﺎﺏ ﺍﻟﺴﺤﺎﻳﺎ‬.A
B. Lateral sinus thrombophlebitis.
.‫ ﺍﻟﺘﻬﺎﺏ ﻭﺭﻳﺪﻱ ﺧﺜﺎﺭﻱ ﺟﻴﺒﻲ ﺟﺎﻧﺒﻲ‬.B
C. Hearing disorders
D. All of the above. .‫ ﺍﺿﻄﺮﺍﺑﺎﺕ ﻓﻲ ﺍﻟﺴﻤﻊ‬.C
E. None of the above. .‫ ﻛﻞ ﺍﻷﺟﻮﺑﺔ ﺃﻋﻼﻩ ﺻﺤﻴﺤﺔ‬.D
.‫ ﻛﻞ ﺍﻷﺟﻮﺑﺔ ﺃﻋﻼﻩ ﻏﻴﺮ ﺻﺤﻴﺤﺔ‬.E D
Laquelle des propositions suivantes est (sont) une (des)
complication (s) de l’otite moyenne?
A. Méningite.
B. Thrombophlébite des sinus latéraux.
C. Toubles de l’audition
D. Toutes les propositions sont justes
E. Toutes les propositions sont fausses
118. Concerning indications for prophylactic treatment of ،‫ﻓﻴﻤﺎ ﻳﺘﻌﻠﻖ ﺑﺈﺭﺷﺎﺩﺍﺕ ﺍﻟﻌﻼﺝ ﺍﻟﻮﻗﺎﺋﻲ ﻟﻨﻮﺑﺎﺕ ﺇﺧﺘﻼﺝ ﺍﻟﺤﻤﻰ‬
febrile seizures, all of the following are true, except: :‫ﻛﻞ ﻣﺎ ﻳﻠﻲ ﺻﺤﻴﺢ ﻣﺎ ﻋﺪﺍ‬
A. If age less than five years .‫ ﺇﺫﺍ ﻛﺎﻥ ﺍﻟﺴﻦ ﺃﻗﻞ ﻣﻦ ﺧﻤﺲ ﺳﻨﻮﺍﺕ‬.A
B. If frequent recurrences
.‫ ﺇﺫﺍ ﻛﺎﻧﺖ ﺗﺘﻜﺮﺭ ﺑﻜﺜﺮﺓ‬.B
C. If neurologic abnormalities
D. If complex seizures .‫ ﺇﺫﺍ ﻛﺎﻥ ﻫﻨﺎﻙ ﺷﺬﻭﺫ ﻋﺼﺒﻲ‬.C
.‫ ﺇﺫﺍ ﻛﺎﻧﺖ ﺍﻟﻨﻮﺑﺎﺕ ﻣﻌﻘﺪﺓ‬.D
A
En ce qui concerne les indications du traitement
prophylactique des convulsions fébriles, toutes les
propositions qui suivent sont vraies, sauf:
A. Si âge inférieur à cinq ans
B. Si récidives fréquentes
C. Si présence d’anomalies neurologiques
D. En cas de crises complexes

48 A
119. A young woman with a history of seizures has a series of ‫ ﺣﺪﺙ ﻟﻬﺎ ﺳﻠﺴﻠﺔ ﻣﻦ‬،‫ﺍﻣﺮﺃﺓ ﺷﺎﺑﺔ ﺗﻌﺎﻧﻲ ﻣﻦ ﻧﻮﺑﺎﺕ ﺻﺮﻋﻴﺔ‬
grand mal seizures in the emergency room. She is ‫ ﻫﻲ ﺑﺤﺎﻟﺔ ﻧﻮﺍﻡ‬.‫ﻧﻮﺑﺎﺕ ﺍﻟﺼﺮﻉ ﺍﻟﻜﺒﻴﺮ ﻓﻲ ﻏﺮﻓﺔ ﺍﻟﻄﻮﺍﺭﺉ‬
lethargic and has a nonfocal neurologic examination. Her ،7.2=PH ‫ ﺃﻅﻬﺮ ﻏﺎﺯ ﺍﻟﺪﻡ‬.‫ﻭﻓﺤﺼﻬﺎ ﺍﻟﻌﺼﺒﻲ ﻏﻴﺮ ﺑﺆﺭﻱ‬
blood gas reveals a pH of 7.12, carbon dioxide of 48, PO2
of 86, and calculated bicarbonate of 16. How would you
‫ ﻭﺛﺎﻧﻲ ﻓﺤﻤﺎﺕ‬،86 PO2 ،48 ‫ﺛﺎﻧﻲ ﺃﻭﻛﺴﻴﺪ ﺍﻟﻜﺮﺑﻮﻥ‬
best characterize her underlying acid-base problem? :‫ﺍﻟﻘﻠﻮﻱ ﻋﻨﺪﻫﺎ؟‬-‫ﻛﻴﻒ ﻳﻤﻜﻦ ﺃﻥ ﺗﺼﻒ ﻣﺸﻜﻠﺔ ﺍﻟﺤﻤﺾ‬.16
A. respiratory acidosis .‫ ﺗﺤﻤﻀﻦ ﺗﻨﻔﺴﻲ‬.A
B. metabolic and respiratory acidosis .‫ ﺗﺤﻤﻀﻦ ﺗﻨﻔﺴﻲ ﻭﺍﺳﺘﻘﻼﺑﻲ‬.B
C. metabolic acidosis and respiratory alkalosis .‫ ﺗﺤﻤﻀﻦ ﺍﺳﺘﻘﻼﺑﻲ ﻭﻗﻠﻮﻧﺔ ﺗﻨﻔﺴﻴﺔ‬.C
D. metabolic alkalosis and respiratory acidosis .‫ ﻗﻠﻮﻧﺔ ﺍﺳﺘﻘﻼﺑﻴﺔ ﻭﺗﺤﻤﻀﻦ ﺗﻨﻔﺴﻲ‬.D
E. metabolic acidosis
.‫ ﺗﺤﻤﻀﻦ ﺍﺳﺘﻘﻼﺑﻲ‬.E
Une jeune femme ayant des antécédents de crises B
convulsives est admise pour une série de crises d'épilepsie
aux urgences. Elle est léthargique et subit un examen
neurologique sans signe de deficit. Ses gaz sanguins
révèlent un pH de 7,12, un dioxyde de carbone de 48, une
PO2 de 86 et une valeur calculée de bicarbonate de 16.
Comment caractériseriez-vous le mieux son problème
acido-basique sous-jacent?
A. Acidose respiratoire
B. acidose métabolique et respiratoire
C. acidose métabolique et alcalose respiratoire
D. alcalose métabolique et acidose respiratoire
E. acidose métabolique
120. Infection with herpes simplex virus, a common human ‫ﺃﻱ ﻣﻦ ﺍﻟﻌﺒﺎﺭﺍﺕ ﺍﻟﺘﺎﻟﻴﺔ ﻫﻲ ﺃﻓﻀﻞ ﻭﺻﻒ ﻟﻺﺻﺎﺑﺔ ﺑﻔﻴﺮﻭﺱ‬
pathogen, is best described by which of the following :‫ ﻭﻫﻮ ﻋﺎﻣﻞ ﻣﻤﺮﺽ ﺑﺸﺮﻱ ﺷﺎﺋﻊ؟‬،‫ﺍﻟﺤﻸ ﺍﻟﺒﺴﻴﻂ‬
statements? ‫ ﺍﻟﺠﻬﺎﺯ ﺍﻟﻌﺼﺒﻲ ﺍﻟﻤﺮﻛﺰﻱ ﻭﺍﻷﻋﻀﺎء ﺍﻟﺤﺸﻮﻳﺔ ﻫﻲ‬.A
a. The CNS and visceral organs are usually involved
.‫ﻋﺎﺩﺓ ﻋﺮﺿﺔ ﻟﻺﺻﺎﺑﺔ‬
b. It rarely recurs in a host who has a high antibody titer
c. It can be reactivated by emotional disturbances or
‫ ﻫﻲ ﻧﺎﺩﺭﺍ ﻣﺎ ﺗﺘﻜﺮﺭ ﻋﻨﺪ ﻣﻀﻴﻒ ﻟﺪﻳﻪ ﻋﻴﺎﺭ ﺃﺿﺪﺍﺩ‬.B
prolonged exposure to sunlight .‫ﻋﺎﻟﻲ‬
d. Initial infection usually occurs by intestinal absorption ‫ ﻳﻤﻜﻦ ﺃﻥ ﺗﺠﺪﺩ ﻧﺸﺎﻁﻬﺎ ﺑﺤﺎﻟﺔ ﺍﻻﺿﻄﺮﺍﺑﺎﺕ‬.C
of the virus .‫ﺍﻟﻌﺎﻁﻔﻴﺔ ﺃﻭ ﺍﻟﺘﻌﺮﺽ ﺍﻟﻤﻄﻮﻝ ﻷﺷﻌﺔ ﺍﻟﺸﻤﺲ‬
e. Infection with type 1 virus is most common ‫ ﺗﺤﺪﺙ ﺍﻹﺻﺎﺑﺔ ﺍﻟﺒﺪﺋﻴﺔ ﻋﺎﺩﺓ ﺑﺎﻻﻣﺘﺼﺎﺹ ﺍﻟﻤﻌﻮﻱ‬.D
Lequel des énoncés suivants décrit le mieux l’infection par .‫ﻟﻠﻔﻴﺮﻭﺱ‬ C
le virus de l’herpès simplex, un agent pathogène humain .‫ ﻫﻲ ﺍﻷﻛﺜﺮ ﺷﻴﻮﻋﺎ‬I ‫ ﺍﻹﺻﺎﺑﺔ ﺑﺎﻟﻔﻴﺮﻭﺱ ﻧﻤﻂ‬.E
courant,
a. Le système nerveux central et les organes viscéraux
sont généralement impliqués
b. Il se reproduit rarement chez un hôte qui a un titre
élevé d'anticorps
c. Il peut être réactivé par des perturbations émotionnelles
ou une exposition prolongée au soleil
d. L'infection initiale se produit généralement par
absorption intestinale du virus
e. L'infection par le virus de type 1 est la plus courante

49 A
121. A 42-year-old obese woman complains of hirsutism, ‫ ﺍﻧﻘﻄﺎﻉ‬،‫ ﺗﺸﻜﻮ ﻣﻦ ﺷﻌﺮﺍﻧﻴﺔ‬،‫ ﻋﺎﻣﺎ‬42 ‫ﺍﻣﺮﺃﺓ ﺑﺪﻳﻨﺔ ﻋﻤﺮﻫﺎ‬
amenorrhea, and difficulty becoming pregnant. What is :‫ ﻣﺎ ﻫﻮ ﺍﻟﺘﺸﺨﻴﺺ ﺍﻷﻭﻝ؟‬.‫ ﻭﺻﻌﻮﺑﺔ ﺃﻥ ﺗﺼﺒﺢ ﺣﺎﻣﻼ‬،‫ﻁﻤﺚ‬
your first diagnosis? .‫ ﻣﺘﻼﺯﻣﺔ ﻛﻮﺷﻴﻨﻎ‬.A
A. Cushing's syndrome
.‫ ﺩﺍء ﺃﺩﻳﺴﻮﻥ‬.B
B. Addison's disease
C. polycystic ovarian disease
.‫ ﺩﺍء ﺍﻟﻤﺒﻴﺾ ﺍﻟﻤﺘﻌﺪﺩ ﺍﻟﻜﻴﺴﺎﺕ‬.C
D. hyperparathyroidism .‫ ﻓﺮﻁ ﻧﺸﺎﻁ ﺍﻟﺪﺭﻳﻖ‬.D
E. hypothyroidism .‫ ﻧﻘﺺ ﻧﺸﺎﻁ ﺍﻟﺪﺭﻕ‬.E
C
Une femme obèse de 42 ans se plaint d'hirsutisme,
d'aménorrhée et de difficultés à tomber enceinte. Quel est
votre premier diagnostic?
A.Syndrome de Cushing
B. maladie d'Addison
C. maladie des ovaires polykystiques
D. hyperparathyroïdie
E. hypothyroïdie
122. A 33 y old lady has idiopathic thrombocytopenia . Which ‫ ﻋﺎﻣﺎ ﺗﻌﺎﻧﻲ ﻣﻦ ﻧﻘﺺ ﺻﻔﻴﺤﺎﺕ ﻣﺠﻬﻮﻝ‬33 ‫ﺳﻴﺪﺓ ﻋﻤﺮﻫﺎ‬
of the following would not be an indication of :‫ ﺃﻱ ﻣﻦ ﺍﻟﺘﺎﻟﻲ ﻟﻦ ﻳﻜﻮﻥ ﻣﺆﺷﺮﺍ ﻻﺳﺘﺌﺼﺎﻝ ﺍﻟﻄﺤﺎﻝ؟‬.‫ﺍﻟﺴﺒﺐ‬
splenectomy ‫ ﻣﺮﺿﻰ ﺑﺪﻭﻥ ﺃﻋﺮﺍﺽ ﻭﺗﻌﺪﺍﺩ ﺻﻔﻴﺤﺎﺕ ﺑﻴﻦ‬.A
A. asymptomatic patients with platelet count between
.³‫ ﻣﻠﻢ‬50000-30000
30000-50000 mm3
.‫ ﻧﻘﺺ ﺻﻔﻴﺤﺎﺕ ﺣﺮﻭﻥ‬.B
B. refractory thrombocytopenia .‫ ﺍﻧﺘﻜﺎﺳﺔ ﺑﻌﺪ ﺍﻟﻌﻼﺝ ﺑﺎﻟﻜﻮﺭﺗﻴﻜﻮﺋﻴﺪﺍﺕ ﺍﻟﺴﻜﺮﻳﺔ‬.C
C. relapse after glucocorticoid therapy 6 ‫ ﺑﺎﻟﺮﻏﻢ ﻣﻦ ﻣﻌﺎﻟﺠﺔ ﻝ‬10000 ‫ ﺗﻌﺪﺍﺩ ﺍﻟﺼﻔﻴﺤﺎﺕ‬.D
D. platelet count of 10000 despite management for 6 .‫ﺃﺳﺎﺑﻴﻊ ﻭﻟﻜﻦ ﺑﺪﻭﻥ ﻧﺰﻳﻒ‬
weeks but no bleeding. .‫ ﻋﺪﺓ ﻧﻮﺑﺎﺕ ﻣﻦ ﺍﻟﻨﺰﻳﻒ‬.E
E. Many episodes of bleeding
A
A 33 ans une jeune dame a une thrombocytopénie
idiopathique. Lequel des éléments suivants ne serait pas
une indication à la splénectomie
A. Asymptomatiques avec numération plaquettaire entre
30000-50000 mm3
B. thrombocytopénie réfractaire
C. Rechute après corticothérapie
D. le nombre de plaquettes à 10000 malgré un traitement
pendant 6 semaines, mais pas de saignement.
E. De nombreux épisodes de saignement
123. Infectious mononucleosis, a viral disorder that can be ‫ ﻫﻮ ﺍﺿﻄﺮﺍﺏ ﻓﻴﺮﻭﺳﻲ ﻳﻤﻜﻦ ﺃﻥ‬،‫ﻛﺜﺮﺓ ﺍﻟﻮﺣﻴﺪﺍﺕ ﺍﻟﺨﻤﺠﻴﺔ‬
debilitating, is characterized by which of the following :‫ ﺃﻱ ﻣﻦ ﺍﻟﻌﺒﺎﺭﺍﺕ ﺍﻟﺘﺎﻟﻴﺔ ﺗﺼﻒ ﻫﺬﺍ ﺍﻟﻤﺮﺽ؟‬،‫ﻳﻜﻮﻥ ﻣﺘﻌﺒﺎ‬
statements? .‫ ﺳﻨﺔ‬14 ‫ ﻫﻮ ﺃﻛﺜﺮ ﺍﻧﺘﺸﺎﺭﺍ ﻋﻨﺪ ﺍﻷﻁﻔﺎﻝ ﺃﻗﻞ ﻣﻦ‬.A
a. It is most prevalent in children less than 14 years old
.‫ ﺍﻟﻌﺎﻣﻞ ﺍﻟﻤﺴﺒﺐ ﻫﻮ ﻓﻴﺮﻭﺱ ﺍﻟﻌﻀﻠﻲ‬.B
b. It is caused by a rhabdovirus
c. The causative pathogen is an Epstein-Barr virus
.‫ﺑﺎﺭ‬-‫ ﺍﻟﻌﺎﻣﻞ ﺍﻟﻤﻤﺮﺽ ﻫﻮ ﻓﻴﺮﻭﺱ ﺇﻱ ﺑﺸﺘﺎﻳﻦ‬.C
d. Affected persons respond to treatment with the ‫ ﺍﻷﺷﺨﺎﺹ ﺍﻟﻤﺼﺎﺑﻮﻥ ﻳﺴﺘﺠﻴﺒﻮﻥ ﺇﻟﻰ ﺍﻟﻌﻼﺝ ﻣﻊ‬.D
production of antibodies .‫ﺍﻧﺘﺎﺝ ﺃﺿﺪﺍﺩ‬
e. Ribavirin is the treatment of choice .‫ ﺭﻳﺒﺎﻓﻴﺮﻳﻦ ﻫﻮ ﺍﻟﺪﻭﺍء ﺍﻟﻤﺨﺘﺎﺭ‬.E
C
La mononucléose infectieuse, une affection virale pouvant
être débilitante, est caractérisée par laquelle des
affirmations suivantes?
a. Il est plus fréquent chez les enfants de moins de 14 ans
b. Il est causé par un rhabdovirus
c. L'agent pathogène en cause est le virus d'Epstein-Barr
d. Les personnes affectées répondent au traitement en
produisant des anticorps
e. La ribavirine est le traitement de choix

50 A
124. A 42-year-old patient suffering from alcoholism has ‫ ﻋﺎﻣﺎ ﻳﻌﺎﻧﻲ ﻣﻦ ﺍﻹﺩﻣﺎﻥ ﻋﻠﻰ ﺍﻟﻜﺤﻮﻝ ﻭﻫﻮ‬42 ‫ﻣﺮﻳﺾ ﻋﻤﺮﻩ‬
advanced liver disease with ascites. He is hospitalized for ‫ ﻫﻮ ﻳﻌﺎﻟﺞ ﻓﻲ‬.‫ﻣﺼﺎﺏ ﺑﻤﺮﺽ ﻛﺒﺪﻱ ﻣﺘﻘﺪﻡ ﻣﻊ ﺍﺳﺘﺴﻘﺎء‬
agitation and bizarre behavior. Which of the following ‫ ﺃﻱ ﻣﻦ ﺍﻟﻤﻈﺎﻫﺮ‬.‫ﺍﻟﻤﺴﺘﺸﻔﻰ ﺑﺴﺒﺐ ﺍﻟﻬﻴﺎﺝ ﻭﺍﻟﺴﻠﻮﻙ ﺍﻟﻐﺮﻳﺐ‬
findings is most helpful in making the diagnosis of
‫ﺍﻟﺘﺎﻟﻴﺔ ﻫﻲ ﺍﻷﻛﺜﺮ ﻣﺴﺎﻋﺪﺓ ﻓﻲ ﺗﺸﺨﻴﺺ ﺍﻻﻋﺘﻼﻝ ﺍﻟﺪﻣﺎﻏﻲ‬
hepatic encephalopathy?
A. jaundice
:‫ﺍﻟﻜﺒﺪﻱ؟‬
B. asterixis of the hands .‫ ﺍﻟﻴﺮﻗﺎﻥ‬.A
C. spider angiomas on the face and chest .‫ ﺍﻻﺭﺗﻌﺎﺵ ﺍﻟﺨﺎﻓﻖ ﻟﻠﻴﺪﻳﻦ‬.B
D. heme-positive stool .‫ ﻭﺭﻡ ﻭﻋﺎﺋﻲ ﻋﻨﻜﺒﻲ ﻋﻠﻰ ﺍﻟﻮﺟﻪ ﻭﺍﻟﺼﺪﺭ‬.C
E. positive fluid wave on abdominal examination .‫ ﺩﻡ ﺍﻟﺒﺮﺍﺯ ﺇﻳﺠﺎﺑﻲ‬.D
.‫ ﻣﻮﺟﺔ ﺳﺎﺋﻞ ﺇﻳﺠﺎﺑﻴﺔ ﻋﻨﺪ ﻓﺤﺺ ﺍﻟﺒﻄﻦ‬.E B
Un patient de 42 ans souffrant d’alcoolisme a une atteinte
avancée du foie associée à une ascite. Il est hospitalisé
pour agitation et comportement bizarre. Laquelle des
constatations suivantes est la plus utile pour poser le
diagnostic d’encéphalopathie hépatique?
A. jaunisse
B. astérixis des mains
C. angiomes d'araignées sur le visage et la poitrine
D. selles séropositives
E. vague de fluide à l'examen abdominal clinique
125. A 78-year-old woman comes to your primary care office ‫ ﻋﺎﻣﺎ ﺣﻀﺮﺕ ﺇﻟﻰ ﻣﻜﺘﺐ ﺍﻟﺮﻋﺎﻳﺔ ﺍﻟﺼﺤﻴﺔ‬78 ‫ﺍﻣﺮﺃﺓ ﻋﻤﺮﻫﺎ‬
practice with her son who is concerned about changes in ‫ ﻟﻘﺪ ﺃﻓﺎﺩﺕ‬.‫ﻣﻊ ﺍﺑﻨﻬﺎ ﺍﻟﺬﻱ ﻳﺸﻌﺮ ﺑﺎﻟﻘﻠﻖ ﺇﺯﺍء ﺗﻐﻴﺮﺍﺕ ﻣﺰﺍﺟﻬﺎ‬
her mood. She is less interested in going out to dinner and ‫ﺃﻧﻬﺎ ﺃﺻﺒﺤﺖ ﺃﻗﻞ ﺍﻫﺘﻤﺎﻣﺎ ﺑﺎﻟﺨﺮﻭﺝ ﻟﻠﻌﺸﺎء ﻭﻻ ﺗﺮﻳﺪ ﺃﻥ ﺗﺰﻭﺭ‬
does not want to visit family or friends. Her language
‫ ﻳﺒﺪﻭ ﺃﻥ ﻣﻬﺎﺭﺍﺗﻬﺎ ﺍﻟﻠﻐﻮﻳﺔ ﺗﺪﻫﻮﺭﺕ ﻋﻠﻰ‬.‫ﺍﻟﻌﺎﺋﻠﺔ ﻭﺍﻷﺻﺤﺎﺏ‬
skills seem to have deteriorated over the last few years
and her memory is not as sharp. Her gait and motor
‫ ﺇﻥ‬.‫ﻣﺪﻯ ﺍﻟﺴﻨﻮﺍﺕ ﺍﻟﻘﻠﻴﻠﺔ ﺍﻷﺧﻴﺮﺓ ﻭﻟﻢ ﺗﻌﺪ ﺫﺍﻛﺮﺗﻬﺎ ﻗﻮﻳﺔ‬
strength are normal. .‫ﻣﺸﻴﺘﻬﺎ ﻭﻗﻮﺓ ﺣﺮﻛﺘﻬﺎ ﻁﺒﻴﻌﻴﺔ‬
Which of the following is the most likely diagnosis? :‫ﺃﻱ ﻣﻦ ﺍﻟﺘﺎﻟﻲ ﻫﻮ ﺍﻟﺘﺸﺨﻴﺺ ﺍﻷﻛﺜﺮ ﺗﺮﺟﻴﺤﺎ؟‬
A. Parkinson's disease .‫ ﺩﺍء ﺑﺎﺭﻛﻨﺴﻮﻥ‬.A
B. anxiety disorder .‫ ﻗﻠﻖ‬.B
C. meningioma .‫ ﻭﺭﻡ ﺳﺤﺎﺋﻲ‬.C
D. Alzheimer's disease .‫ ﺩﺍء ﺍﻟﺰﻫﺎﻳﻤﺮ‬.D
E. dysthymia .‫ ﺍﻛﺘﺌﺎﺏ ﺟﺰﺋﻲ‬.E
D
Une femme de 78 ans se présente à votre cabinet de soins
primaires aidee de son fils qui se préoccupe de
changements dans son humeur. Elle est moins intéressée à
sortir dîner et ne veut pas rendre visite à sa famille ou à
ses. Amis. Ses compétences linguistiques semblent s'être
détériorées au cours des dernières années et sa mémoire
est moins claire. Sa démarche et sa force motrice sont
normales.
Lequel des diagnostics suivants est le plus probable?
A. maladie de Parkinson
B. trouble d'anxiété
C. méningiome
D. maladie d'Alzheimer
E. dysthymie

51 A
126. One of the most common sexually transmitted diseases ‫ﺃﺣﺪ ﺍﻷﻣﺮﺍﺽ ﺍﻟﻤﻨﻘﻮﻟﺔ ﺟﻨﺴﻴﺎ ﺍﻷﻛﺜﺮ ﺷﻴﻮﻋﺎ ﻭﺍﻟﺘﻲ ﻳﻤﻜﻦ ﺃﻥ‬
that may lead to cervical carcinoma is caused by which of ‫ ﺳﺒﺒﻬﺎ ﺃﻱ ﻣﻦ ﺍﻟﻔﻴﺮﻭﺳﺎﺕ‬،‫ﺗﺆﺩﻱ ﺇﻟﻰ ﺳﺮﻁﺎﻥ ﻋﻨﻖ ﺍﻟﺮﺣﻢ‬
the following viruses? :‫ﺍﻟﺘﺎﻟﻴﺔ؟‬
a. Cytomegalovirus
.‫ ﺍﻟﻔﻴﺮﻭﺱ ﺍﻟﻤﻀﺨﻢ ﻟﻠﺨﻼﻳﺎ‬.A
b. Papillomavirus
c. Epstein-Barr virus
.‫ ﻓﻴﺮﻭﺱ ﺍﻟﻮﺭﻡ ﺍﻟﺤﻠﻴﻤﻲ‬.B
d. Herpes simplex virus .‫ ﻓﻴﺮﻭﺱ ﺇﻳﺐ ﺷﺘﺎﻳﻦ ﺑﺎﺭ‬.C
e. Adenovirus .‫ ﻓﻴﺮﻭﺱ ﺍﻟﺤﻸ ﺍﻟﺒﺴﻴﻂ‬.D
.‫ ﻓﻴﺮﻭﺱ ﺍﻟﻐﺪﻳﺔ‬.E B
Parmi les virus suivants, l’une des maladies sexuellement
transmissibles les plus courantes peut entraîner un
carcinome du col utérin ; quel virus?
a. Cytomégalovirus
b. Papillomavirus
c. Virus d'Epstein-Barr
d. Virus de l'herpès simplex
e. Adénovirus
127. A hospital nurse is found to have hepatitis B surface ‫ﺗﺒﻴﻦ ﺃﻥ ﻣﻤﺮﺿﺔ ﻣﺴﺘﺸﻔﻰ ﻟﺪﻳﻬﺎ ﻣﻮﻟﺪ ﺿﺪ ﺳﻄﺤﻲ ﻻﻟﺘﻬﺎﺏ‬
antigen. Subsequent tests reveal the presence of e antigen ‫ ﺗﻜﺸﻒ ﺍﻟﻔﺤﻮﺹ ﺍﻟﻼﺣﻘﺔ ﻋﻦ ﻭﺟﻮﺩ ﻣﻮﻟﺪ‬.B ‫ﺍﻟﻜﺒﺪ ﺍﻟﻔﻴﺮﻭﺳﻲ‬
as well. She most likely :‫ ﺇﻧﻬﺎ ﻋﻠﻰ ﺍﻷﺭﺟﺢ‬.‫ ﻛﺬﻟﻚ‬e ‫ﺿﺪ‬
a. Is infective and has active hepatitis
.‫ ﻣﺼﺎﺑﺔ ﻭﺗﻌﺎﻧﻲ ﻣﻦ ﺍﻟﺘﻬﺎﺏ ﻛﺒﺪ ﻧﺸﻴﻂ‬.A
b. Is infective but does not have active hepatitis
c. Is not infective
.‫ ﻣﺼﺎﺑﺔ ﻭﻻ ﺗﻌﺎﻧﻲ ﺍﻟﺘﻬﺎﺏ ﻛﺒﺪ ﻧﺸﻴﻂ‬.B
d. Is evincing a biologic false-positive test for hepatitis .‫ ﻏﻴﺮ ﻣﺼﺎﺑﺔ‬.C
e. Has both hepatitis b and c ‫ ﺗﺜﺒﺖ ﺃﻥ ﺍﺧﺘﺒﺎﺭ ﺍﻟﺘﻬﺎﺏ ﺍﻟﻜﺒﺪ ﺍﻟﺤﻴﻮﻱ ﻫﻮ ﺇﻳﺠﺎﺑﻲ‬.D
.‫ﻛﺎﺫﺏ‬
A
Une infirmière d’hôpital présente un antigène de surface .C‫ ﻭ‬B‫ ﺗﻌﺎﻧﻲ ﻣﻦ ﺍﻟﺘﻬﺎﺏ ﺍﻟﻜﺒﺪ ﺍﻟﻔﻴﺮﻭﺳﻲ‬.E
de l’hépatite B. Des tests ultérieurs révèlent également la
présence de l'antigène e. Elle
a. Est contagieuse et elle a l'hépatite active
b. Est contagieuse mais ne présente pas d'hépatite active
c. N'est pas contagieuse
ré. Fait preuve d'un test biologique faussement positif
pour l'hépatite
e. A les deux hépatites B et C
128. A 23-year-old man presents complaining of severe ‫ ﻋﺎﻣﺎ ﻳﺸﻜﻮ ﻣﻦ ﺃﻟﻢ ﺑﻄﻨﻲ ﺗﺸﻨﺠﻲ ﺷﺪﻳﺪ ﻭﺩﻡ‬23 ‫ﺭﺟﻞ ﻋﻤﺮﻩ‬
crampy abdominal pain and blood in his stool over the ‫ﻟﻘﺪ ﺣﺪﺛﺖ ﻟﻪ ﻧﻮﺑﺔ‬.‫ﻓﻲ ﺍﻟﺒﺮﺍﺯ ﻋﻠﻰ ﻣﺪﻯ ﺍﻟﻴﻮﻣﻴﻦ ﺍﻟﻤﺎﺿﻴﻴﻦ‬
past 2 days. A similar episode occurred a few months ago .‫ ﻟﻴﺲ ﻟﺪﻳﻪ ﺗﺎﺭﻳﺦ ﺳﻔﺮ‬.‫ﻣﺸﺎﺑﻬﺔ ﻣﻨﺬ ﻋﺪﺓ ﺃﺷﻬﺮ ﻭﺍﻧﺘﻬﺖ ﺗﻠﻘﺎﺋﻴﺎ‬
and spontaneously resolved. No history of travel.
‫ ﺃﻱ ﻣﻦ ﺍﻟﺘﺎﻟﻲ ﻫﻮ‬.‫ﺗﻈﻬﺮ ﺍﻷﺷﻌﺔ ﺍﻟﺴﻴﻨﻴﺔ ﺗﻤﺪﺩ ﻗﻮﻟﻮﻧﻲ ﺧﻔﻴﻒ‬
Abdominal x-ray shows mild colonic dilatation. Which of
the following is the most likely diagnosis?
:‫ﺍﻟﺘﺸﺨﻴﺺ ﺍﻷﻛﺜﺮ ﺗﺮﺟﻴﺤﺎ؟‬
A. ulcerative colitis .‫ ﺍﻟﺘﻬﺎﺏ ﻗﻮﻟﻮﻧﻲ ﻣﺘﻘﺮﺡ‬.A
B. viral gastroenteritis .‫ ﺍﻟﺘﻬﺎﺏ ﻫﻀﻤﻲ ﻓﻴﺮﻭﺳﻲ‬.B
C. irritable bowel syndrome .‫ ﻣﺘﻼﺯﻣﺔ ﺍﻟﻘﻮﻟﻮﻥ ﺍﻟﻤﺘﻬﻴﺞ‬.C
D. celiac sprue .‫ ﺫﺭﺏ ﺑﻄﻨﻲ‬.D
E. Whipple disease .‫ ﺩﺍء ﻫﻮﻳﺒﻞ‬.E
A
Un homme de 23 ans se plaint de douleurs abdominales
intenses et de sang dans les selles au cours des 2 derniers
jours. Un épisode similaire a eu lieu il y a quelques mois
et s'est spontanément résolu. Pas d'histoire de voyage . La
radiographie abdominale montre une légère dilatation du
côlon. Lequel des diagnostics suivants est le plus
probable?
A. colite ulcéreuse
B. gastro-entérite virale
C. syndrome du côlon irritable
D. sprue coeliaque
E. maladie de Whipple
52 A
129. A26-year-old man presents with a hard, painless testicular ‫ ﻋﺎﻣﺎ ﻳﻌﺎﻧﻲ ﻣﻦ ﻛﺘﻠﺔ ﺧﺼﻴﻮﻳﺔ ﺻﻠﺒﺔ ﻏﻴﺮ‬26 ‫ﺭﺟﻞ ﻋﻤﺮﻩ‬
mass. At operation, frozen section reveals testicular ‫ ﻛﺸﻒ ﺍﻟﻘﺴﻢ ﺍﻟﻤﺠﻤﺪ ﻋﻦ ﻭﺟﻮﺩ ﺳﺮﻁﺎﻥ‬،‫ ﺃﺛﻨﺎء ﺍﻟﻌﻤﻠﻴﺔ‬.‫ﻣﺆﻟﻤﺔ‬
cancer. Which of the following is a risk factor? :‫ ﺃﻱ ﻣﻦ ﺍﻟﺘﺎﻟﻲ ﻫﻮ ﻋﺎﻣﻞ ﻣﻬﺪﺩ؟‬.‫ﺧﺼﻴﺔ‬
A. family history of testicular cancer
.‫ ﺳﻴﺮﺓ ﻋﺎﺋﻠﻴﺔ ﻣﻦ ﺳﺮﻁﺎﻥ ﺍﻟﺨﺼﻴﺔ‬.A
B. masturbation
C. prior history of radiation exposure
.‫ ﺍﻹﺳﺘﻤﻨﺎء‬.B
D. cryptorchidism .‫ ﺗﺎﺭﻳﺦ ﻣﺴﺒﻖ ﻣﻦ ﺍﻟﺘﻌﺮﺽ ﻟﻺﺷﻌﺎﻉ‬.C
E. maternal diethylstilbestrol (DES) during pregnancy .‫ ﺍﺧﺘﻔﺎء ﺍﻟﺨﺼﻴﺔ‬.D
.‫ ﺩﻱ ﺇﺗﻴﻞ ﺳﺘﻴﻠﺒﺴﺘﺮﻭﻝ ﺃﻣﻮﻣﻲ ﺃﺛﻨﺎء ﺍﻟﺤﻤﻞ‬.E
D
Un homme de 26 ans présente une masse testiculaire dure
et indolore. À l'opération, un extemporane révèle un
cancer du testicule. Lequel des facteurs suivants est un
facteur de risque?
A. antécédents familiaux de cancer du testicule
B. masturbation
C. antécédents d'exposition au rayonnement
D. cryptorchidie
E. Diéthylstilbestrol (DES) maternel pendant la grossesse
130. An immunocompromised person with history of seizures ،‫ﺷﺨﺺ ﻣﺜﺒﻂ ﻣﻨﺎﻋﻴﺎ ﻭﻟﺪﻳﻪ ﺳﻴﺮﺓ ﻣﻦ ﺍﻟﻨﻮﺑﺎﺕ ﺍﻟﺼﺮﻋﻴﺔ‬
had an MRI that revealed a temporal lobe lesion. Brain ‫ﺃﺟﺮﻯ ﺗﺼﻮﻳﺮ ﺑﺎﻟﺮﻧﻴﻦ ﺍﻟﻤﻐﻨﺎﻁﻴﺴﻲ ﺣﻴﺚ ﺗﺒﻴﻦ ﻭﺟﻮﺩ ﺁﻓﺔ ﻓﻲ‬
biopsy results showed multinucleated giant cells with ‫ ﺃﻅﻬﺮﺕ ﻧﺘﺎﺋﺞ ﺧﺰﻋﺔ ﺍﻟﺪﻣﺎﻍ ﻭﺟﻮﺩ ﺧﻼﻳﺎ‬.‫ﺍﻟﻔﺺ ﺍﻟﺼﺪﻏﻲ‬
intranuclear inclusions. The most probable cause of the
‫ ﺇﻥ ﺍﻟﺴﺒﺐ‬.‫ﻋﻤﻼﻗﺔ ﻣﺘﻌﺪﺩﺓ ﺍﻟﻨﻮﻯ ﻣﻊ ﺗﻀﻤﻴﻨﺎﺕ ﺩﺍﺧﻞ ﺍﻟﻨﻮﻯ‬
lesion is
a. Hepatitis C virus
:‫ﺍﻷﻛﺜﺮ ﺍﺣﺘﻤﺎﻻ ﻟﻶﻓﺔ ﻫﻮ‬
b. Herpes simplex virus .C ‫ ﻓﻴﺮﻭﺱ ﺍﻟﺘﻬﺎﺏ ﺍﻟﻜﺒﺪ ﺍﻟﻔﻴﺮﻭﺳﻲ‬.A
c. Listeria monocytogenes .‫ ﻓﻴﺮﻭﺱ ﺍﻟﺤﻸ ﺍﻟﺒﺴﻴﻂ‬.B
d. Coxsackievirus .‫ ﺍﻟﻠﻴﺴﺘﺮﻳﺔ ﺍﻟﻤﺴﺘﻮﺣﺪﺓ‬.C
e. Parvovirus .‫ ﻓﻴﺮﻭﺱ ﻛﻮﻛﺴﺎﻛﻲ‬.D
.‫ ﺍﻟﻔﻴﺮﻭﺳﺔ ﺍﻟﺼﻐﻴﺮﺓ‬.E B
Une personne immunodéprimée ayant déjà eu des crises
convulsives a une IRM révélant une lésion du lobe
temporal. Les résultats de la biopsie cérébrale ont montré
des cellules géantes multinucléées avec des inclusions
intranucléaires. La cause la plus probable de la lésion est :
a.Virus de l'hépatite C
b. Virus de l'herpès simplex
c. Listeria monocytogenes
d. Coxsackievirus
e. Parvovirus

53 A
131. Which of the following procedures or clinical signs is ‫ﺃﻱ ﻣﻦ ﺍﻹﺟﺮﺍءﺍﺕ ﺃﻭ ﺍﻟﻌﻼﻣﺎﺕ ﺍﻟﺴﺮﻳﺮﻳﺔ ﺍﻟﺘﺎﻟﻴﺔ ﻫﻲ ﺍﻷﻛﺜﺮ‬
most specific for the diagnosis of infectious ‫ﻧﻮﻋﻴﺔ ﻓﻲ ﺗﺸﺨﻴﺺ ﺍﻧﺘﺎﻥ ﻭﺣﻴﺪﺍﺕ ﺍﻟﻨﻮﻯ ﺍﻟﻨﺎﺟﻢ ﻋﻦ ﻓﻴﺮﻭﺱ‬
mononucleosis caused by the Epstein-Barr virus? :‫ﺇﻱ ﺑﺸﺘﺎﻳﻦ ﺑﺎﺭ؟‬
a. Laboratory diagnosis is based on the presence of
‫ ﻳﺴﺘﻨﺪ ﺍﻟﺘﺸﺨﻴﺺ ﺍﻟﻤﺨﺒﺮﻱ ﻋﻠﻰ ﻭﺟﻮﺩ ﺍﻟﻠﻴﻤﻔﺎﻭﻳﺎﺕ‬.A
“atypical lymphocytes” and EBV-specific antibody
b. Growth in tissue culture cells
. EBV‫ﺍﻟﻼﻧﻤﻄﻴﺔ ﻭﺍﻟﻀﺪ ﺍﻟﻨﻮﻋﻲ ﻝ‬
c. Heterophile antibodies in serum .‫ ﺍﻟﻨﻤﻮ ﻓﻲ ﺧﻼﻳﺎ ﺍﻟﺰﺭﻉ ﺍﻟﻨﺴﻴﺠﻴﺔ‬.B
d. Lymphadenopathy and splenomegaly on physical .‫ ﺃﺿﺪﺍﺩ ﻣﺨﺘﻠﻔﺔ ﻓﻲ ﺍﻟﻤﺼﻞ‬.C
examination ‫ ﺗﻀﺨﻢ ﺍﻟﻌﻘﺪ ﺍﻟﻠﻤﻔﻴﺔ ﻭﺿﺨﺎﻣﺔ ﻁﺤﺎﻝ ﻋﻨﺪ ﺍﻟﻔﺤﺺ‬.D
e. B-cell lymphocyte proliferation .‫ﺍﻟﺴﺮﻳﺮﻱ‬
.B ‫ ﺍﻧﺘﺸﺎﺭ ﺧﻼﻳﺎ ﺍﻟﻠﻤﻔﺎﻭﻳﺎﺕ‬.E B
Laquelle des procédures ou des signes cliniques suivants
est la plus spécifique pour le diagnostic de la
mononucléose infectieuse causée par le virus d’Epstein-
Barr
a. Le diagnostic de laboratoire repose sur la présence de
«lymphocytes atypiques» et d’anticorps spécifique de
l'EBV
b. Croissance dans des cellules de culture tissulaire
c. Anticorps hétérophiles dans le sérum
d. Adénopathie et splénomégalie à l'examen physique
e. Prolifération des lymphocytes B
132. A 62 years old patient shows you a mammogram ‫ ﺃﻅﻬﺮﺕ ﺻﻮﺭﺓ ﺍﻟﺜﺪﻱ ﺍﻟﺸﻌﺎﻋﻴﺔ‬،‫ ﻋﺎﻣﺎ‬62 ‫ﻣﺮﻳﻀﺔ ﻋﻤﺮﻫﺎ‬
containing all the elements below. What is the one that is ‫ ﺃﻱ ﻣﻦ ﺍﻟﺘﺎﻟﻲ ﻫﻮ‬.‫ﺍﺣﺘﻮﺍﺋﻬﺎ ﻋﻠﻰ ﻛﻞ ﺍﻟﻌﻨﺎﺻﺮ ﺍﻟﻮﺍﺭﺩﺓ ﺃﺩﻧﺎﻩ‬
the least suggestive of breast cancer :‫ﺍﻷﻗﻞ ﺍﺣﺘﻤﺎﻻ ﻟﺴﺮﻁﺎﻥ ﺍﻟﺜﺪﻱ؟‬
A. A stellar opacity of 2cm
.‫ ﺳﻢ‬2 ‫ ﻋﺘﺎﻣﺔ ﻧﺠﻤﻴﺔ‬.A
B. Not well delineated opacity of 1 cm
C. macrocalcification measuring 7 mm
.‫ ﺳﻢ‬1 ‫ ﻋﺘﺎﻣﺔ ﺫﺍﺕ ﺍﺧﺘﻄﺎﻁ ﻏﻴﺮ ﺟﻴﺪ‬.B
D. fine cluster scattered microcalcification .‫ ﻣﻠﻢ‬7 ‫ ﺗﻜﻠﺲ ﻛﺒﻴﺮ ﺑﻘﻴﺎﺱ‬.C
E. V-shaped or Y microcalcifications .‫ ﺗﻜﻠﺲ ﺻﻐﻴﺮ ﻣﺘﺒﻌﺜﺮ ﻋﻨﻘﻮﺩﻱ ﺩﻗﻴﻖ‬.D
.Y ‫ ﺃﻭ‬V ‫ ﺗﻜﻠﺲ ﺻﻐﻴﺮ ﺑﺸﻜﻞ‬.E C
Une patiente de 62 ans vous montre une mammographie
avec tous les elements ci-dessous. Quel est celui le moins
suggestif d’un cancer du sein
A. Une opacité stellaire de 2 cm
B. Une opacité à contour flou de 1 cm
C. Une macrocalcification mesurant 7 mm
D. Des microcalcification fines eparpillés en amas
E. Des microcalcifications en forme de V ou Y

54 A
133. Cytomegalovirus (CMV) infection is common. Which one ‫ ﺃﻱ ﻣﻦ ﺍﻟﻌﺒﺎﺭﺍﺕ‬.‫ﺍﻟﻔﻴﺮﻭﺱ ﺍﻟﻤﻀﺨﻢ ﻟﻠﺨﻼﻳﺎ ﻫﻮ ﺇﺻﺎﺑﺔ ﺷﺎﺋﻌﺔ‬
of the following statements best characterizes CMV? :‫؟‬CMV ‫ﺍﻟﺘﺎﻟﻴﺔ ﺗﺼﻒ ﺑﺸﻜﻞ ﺃﻓﻀﻞ‬
a. It can be transmitted across the placental barrier .‫ ﻳﻤﻜﻦ ﺃﻥ ﻳﻨﺘﻘﻞ ﻋﺒﺮ ﺍﻟﺤﺎﺟﺰ ﺍﻟﻤﺸﻴﻤﻲ‬.A
b. While a common infection, CMV is almost always
‫ ﻫﻮ ﺗﻘﺮﻳﺒﺎ ﺩﺍﺋﻤﺎ‬CMV ،‫ ﺑﻤﺎ ﺃﻧﻪ ﺇﺻﺎﺑﺔ ﺷﺎﺋﻌﺔ‬.B
symptomatic
c. The CMV can be cultured from red blood cells of
.‫ﻣﺘﺮﺍﻓﻖ ﻣﻊ ﺃﻋﺮﺍﺽ‬
infected patients ‫ ﻣﻦ ﻛﺮﻳﺎﺕ ﺍﻟﺪﻡ ﺍﻟﺤﻤﺮﺍء‬CMV ‫ ﻳﻤﻜﻦ ﺯﺭﻉ‬.C
d. Unlike other viral infections, CMV is not activated by .‫ﻟﻠﻤﺮﺿﻰ ﺍﻟﻤﺼﺎﺑﻴﻦ‬
immunosuppressive therapy ‫ ﻻ‬CMV ،‫ ﺧﻼﻓﺎ ﻟﻐﻴﺮﻩ ﻣﻦ ﺍﻹﺻﺎﺑﺎﺕ ﺍﻟﻔﻴﺮﻭﺳﻴﺔ‬.D
e. There is no specific therapy for CMV .‫ﻳﻨﺸﻂ ﺑﺎﻟﻌﻼﺝ ﺍﻟﻤﺜﺒﻂ ﻟﻠﻤﻨﺎﻋﺔ‬
. CMV‫ ﻻ ﻳﻮﺟﺪ ﻋﻼﺝ ﻧﻮﻋﻲ ﻝ‬.E A
L’infection à cytomégalovirus (CMV) est courante.
Laquelle des affirmations suivantes caractérise le mieux le
CMV?
a. Il peut être transmis à travers la barrière placentaire
b. Bien qu’étant une infection commune, le CMV est
presque toujours symptomatique
c. Le CMV peut être cultivé à partir de globules rouges de
patients infectés
d. Contrairement aux autres infections virales, le CMV
n'est pas activé par la thérapie immunosuppressive.
e. Il n'y a pas de traitement spécifique pour le CMV
134. Human papillomavirus is most commonly associated :‫ﻓﻴﺮﻭﺱ ﺍﻟﻮﺭﻡ ﺍﻟﺤﻠﻴﻤﻲ ﺍﻟﺒﺸﺮﻱ ﻫﻮ ﺃﻛﺜﺮ ﺷﻴﻮﻋﺎ ﻣﻊ‬
with .‫ ﺳﻠﻴﻠﺔ ﻣﺴﺘﻘﻴﻤﺔ‬.A
a. Rectal polyps .‫ ﺳﺮﻁﺎﻥ ﺍﻟﺒﺮﻭﺳﺘﺎﺕ‬.B
b. Prostate cancer
.‫ ﻭﺭﻡ ﻟﻘﻤﻲ ﻣﺆﻧﻒ‬.C
c. Condyloma acuminatum
d. Hepatic carcinoma
.‫ ﺳﺮﻁﺎﻥ ﻛﺒﺪﻱ‬.D
e. Carcinoma of the lung .‫ ﺳﺮﻁﺎﻥ ﺭﺋﺔ‬.E
C
Le virus du papillome humain est le plus souvent associé
à:
a. Polypes rectaux
b. Cancer de la prostate
c. Condyloma acuminatum
d. Carcinome hépatique
e. Carcinome du poumon
135. Shigellosis is common in travelers to developing ‫ﺩﺍء ﺍﻟﺸﻴﻐﻠﻼ ﻫﻮ ﻣﺮﺽ ﺷﺎﺋﻊ ﻋﻨﺪ ﺍﻟﻤﺴﺎﻓﺮﻳﻦ ﺇﻟﻰ ﺍﻟﺪﻭﻝ‬
countries. Infection is commonly acquired through the :‫ ﻫﺬﻩ ﺍﻹﺻﺎﺑﺔ ﻫﻲ ﻋﺎﺩﺓ ﺗﻜﺘﺴﺐ ﻋﺒﺮ‬.‫ﺍﻟﻨﺎﻣﻴﺔ‬
a. Skin .‫ ﺍﻟﺠﻠﺪ‬.A
b. Gastrointestinal tract
.‫ ﺍﻟﺠﻬﺎﺯ ﺍﻟﻬﻀﻤﻲ‬.B
c. Respiratory tract
d. Genital tract
.‫ ﺍﻟﺠﻬﺎﺯ ﺍﻟﺘﻨﻔﺴﻲ‬.C
e. Nasal tract .‫ ﺍﻟﺠﻬﺎﺯ ﺍﻟﺘﻨﺎﺳﻠﻲ‬.D
.‫ ﺍﻟﺠﻬﺎﺯ ﺍﻷﻧﻔﻲ‬.E
B
La shigellose est fréquente chez les voyageurs se rendant
dans les pays en développement. L’infection est
généralement contractée par le biais de :
a. Peau
b. Tube digestif
c. Voies respiratoires
d. Tractus génital
e. Voie nasale

55 A
136. A 17-year-old girl presents with cervical ‫ ﻋﺎﻣﺎ ﺗﻌﺎﻧﻲ ﻣﻦ ﺗﻀﺨﻢ ﻋﻘﺪ ﻟﻤﻔﺎﻭﻳﺔ ﻓﻲ‬17 ‫ﻓﺘﺎﺓ ﻋﻤﺮﻫﺎ‬
lymphadenopathy, fever, and pharyngitis. Infectious ‫ ﻳﺸﺘﺒﻪ ﺑﺈﻧﺘﺎﻥ ﻭﺣﻴﺪﺍﺕ‬.‫ ﻭﺍﻟﺘﻬﺎﺏ ﺍﻟﺒﻠﻌﻮﻡ‬،‫ ﺣﺮﺍﺭﺓ‬،‫ﺍﻟﺮﻗﺒﺔ‬
mononucleosis is suspected. The most rapid and clinically ‫ ﺇﻥ ﺍﻟﻔﺤﺺ ﺍﻷﻛﺜﺮ ﺳﺮﻋﺔ ﻭﺍﻟﻤﻔﻴﺪ ﺳﺮﻳﺮﻳﺎ ﻹﺟﺮﺍء ﻫﺬﺍ‬.‫ﺍﻟﻨﻮﻯ‬
useful test to make this diagnosis is
:‫ﺍﻟﺘﺸﺨﻴﺺ ﻫﻮ‬
a. IgM antibody to viral core antigen (VCA)
b. IgG antibody to VCA
‫ ﻟﻤﺴﺘﻀﺪ ﺍﻟﻐﻼﻑ ﺍﻟﻔﻴﺮﻭﺳﻲ‬IgM ‫ ﺃﺿﺪﺍﺩ‬.A
c. Antibody to Epstein-Barr nuclear antigen (EBNA) .(VCA)
d. Culture .(VCA) ‫ ﻝ‬IgG ‫ ﺃﺿﺪﺍﺩ‬.B
e. C reactive protein (CRP) ‫ ﺃﺿﺪﺍﺩ ﻟﻠﻤﺴﺘﻀﺪ ﺍﻟﻨﻮﻭﻱ ﻹﻳﺐ ﺷﺘﺎﻳﻦ ﺑﺎﺭ‬.C
.(EBNA) A
Une fille de 17 ans présente des adenopathies cervicales, .‫ ﺍﻟﺰﺭﻉ‬.D
une fièvre et pharyngite. Une mononucléose infectieuse et .(CRP) C ‫ ﺍﻟﺒﺮﻭﺗﻴﻦ ﺍﻟﺘﻔﺎﻋﻠﻲ‬.E
suspecte. Le test le plus rapide et le plus utile cliniquement
pour le diagnostic est :
A. Anticorps IgM pour le viral core antigene (VCA)
B. Anticorps IgG pour le VCA
C. Anticorps pour l’antigeène Epstein Barr (EBNA)
D. Culture
E. CRP
137. Hepatitis C (HCV) is usually a clinically mild disease, ،‫ ﻫﻮ ﻋﺎﺩﺓ ﻣﺮﺽ ﺧﻔﻴﻒ ﺳﺮﻳﺮﻳﺎ‬C ‫ﺍﻟﺘﻬﺎﺏ ﺍﻟﻜﺒﺪ ﺍﻟﻔﻴﺮﻭﺳﻲ‬
with only minimal elevation of liver enzymes. ‫ ﺃﻱ ﻣﻦ‬.‫ﻳﺘﺮﺍﻓﻖ ﻓﻘﻂ ﻣﻊ ﺍﺭﺗﻔﺎﻉ ﻗﻠﻴﻞ ﺑﺎﻟﺨﻤﺎﺋﺮ ﺍﻟﻜﺒﺪﻳﺔ‬
Hospitalization is unusual. Which one of the following :‫ ﺑﺸﻜﻞ ﺃﻓﻀﻞ؟‬HCV ‫ﺍﻟﻌﺒﺎﺭﺍﺕ ﺍﻟﺘﺎﻟﻴﺔ ﺗﺼﻒ‬
statements best characterizes HCV?
.‫ ﺑﻌﺾ ﺍﻟﺤﺎﻻﺕ ﺗﺘﻔﺎﻗﻢ ﻟﺘﺘﺤﻮﻝ ﻟﻤﺮﺽ ﻛﺒﺪﻱ ﻣﺰﻣﻦ‬.A
a. Few cases progress to chronic liver disease
b. It often occurs in post transfusion patients
.‫ ﻳﺤﺪﺙ ﻋﺎﺩﺓ ﺑﻌﺪ ﻧﻘﻞ ﺍﻟﺪﻡ ﻟﻠﻤﺮﺿﻰ‬.B
c. HBV but not HCV infections occur in IV drug abusers ‫ ﺗﺤﺪﺙ ﻋﻨﺪ ﻣﺘﻌﺎﻁﻲ‬HCV ‫ ﻭﻟﻴﺲ‬HBV ‫ ﻋﺪﻭﻯ‬.C
d. It is a DNA virus .‫ﺍﻟﻤﺨﺪﺭﺍﺕ ﻭﺭﻳﺪﻳﺎ‬
e. Blood products are not tested for antibody to HCV .DNA ‫ ﻫﻮ ﻓﻴﺮﻭﺱ‬.D
‫ ﻧﻮﺍﺗﺞ ﺍﻟﺪﻡ ﻻ ﺗﻔﺤﺺ ﻣﻦ ﺃﺟﻞ ﺍﻟﺘﺤﺮﻱ ﻋﻦ ﺃﺿﺪﺍﺩ‬.E
L'hépatite C (VHC) est généralement une maladie .HCV
cliniquement bénigne, caractérisée par une élévation
B
minime des enzymes hépatiques. L'hospitalisation est
inhabituelle. Lequel des énoncés suivants caractérise le
mieux le VHC?
a. Peu de cas évoluent vers une maladie hépatique
chronique
b. Il survient souvent chez les patients post-
transfusionnels
c. Les infections à VHB mais non à VHC se produisent
chez les toxicomanes par voie intraveineuse
d. C'est un virus à ADN
e. Les produits sanguins ne sont pas testés pour les
anticorps anti-VHC

56 A
138. A 55-year-old man with a 50 pack-year history of smoking ‫ ﺣﻀﺮ ﻭﻫﻮ‬،‫ ﻋﻠﺒﺔ ﺳﻨﻮﻳﺎ‬50 ‫ ﻋﺎﻣﺎ ﻳﺪﺧﻦ‬55 ‫ﺭﺟﻞ ﻋﻤﺮﻩ‬
presents with hemoptysis. CXR shows a left upper lobe ‫ ﺃﻅﻬﺮﺕ ﺍﻷﺷﻌﺔ ﺍﻟﺴﻴﻨﻴﺔ ﻟﻠﺼﺪﺭ ﻛﺘﻠﺔ ﻓﻲ‬.‫ﻳﺸﻜﻮ ﻣﻦ ﻧﻔﺚ ﺍﻟﺪﻡ‬
mass and laboratory evaluation reveals hypercalcemia. ‫ﺍﻟﻔﺺ ﺍﻟﻌﻠﻮﻱ ﺍﻷﻳﺴﺮ ﻛﻤﺎ ﻛﺸﻒ ﺍﻟﺘﻘﻴﻴﻢ ﺍﻟﻤﺨﺒﺮﻱ ﻋﻦ ﻓﺮﻁ‬
Which of the following is the most likely diagnosis?
.‫ﻛﺎﻟﺴﻴﻮﻡ ﺍﻟﺪﻡ‬
A. small cell lung cancer
B. tuberculosis
:‫ﺃﻱ ﻣﻦ ﺍﻟﺘﺎﻟﻲ ﻫﻮ ﺍﻟﺘﺸﺨﻴﺺ ﺍﻷﻛﺜﺮ ﺗﺮﺟﻴﺤﺎ؟‬
C. squamous cell lung cancer .‫ ﺳﺮﻁﺎﻥ ﺭﺋﺔ ﺻﻐﻴﺮ ﺍﻟﺨﻼﻳﺎ‬.A
D. adenocarcinoma of the lung .‫ ﺩﺍء ﺍﻟﺴﻞ‬.B
E. metastatic testicular cancer .‫ ﺳﺮﻁﺎﻥ ﺭﺋﺔ ﺫﺍﺕ ﺍﻟﺨﻼﻳﺎ ﺍﻟﺤﺮﺷﻔﻴﺔ‬.C
.‫ ﺳﺮﻁﺎﻧﻪ ﻏﺪﻳﺔ ﻓﻲ ﺍﻟﺮﺋﺔ‬.D
C
Un homme de 55 ans fumeur (50 paquets-annees) se .‫ ﺳﺮﻁﺎﻥ ﺍﻟﺨﺼﻴﺔ ﺍﻟﻨﻘﻴﻠﻲ‬.E
présente avec une hémoptysie. La radio thorax montre
une masse du lobe supèrieur gauche et une évaluation en
laboratoire révèle une hypercalcémie. Lequel des
diagnostics suivants est le plus probable?
A. cancer du poumon à petites cellules
B. tuberculose
C. cancer du poumon epidermoide
D. adénocarcinome du poumon
E. cancer du testicule métastatique
139. Lethargy, malaise, and fatigue are observed in a patient 2 ‫ ﺍﻟﺨﻤﻮﻝ ﻭﺍﻟﺘﻌﺐ ﻋﻨﺪ ﻣﺮﻳﺾ ﺑﻌﺪ ﺃﺳﺒﻮﻋﻴﻦ‬،‫ﻟﻘﺪ ﻟﻮﺣﻆ ﺍﻟﻨﻮﺍﻡ‬
weeks after eating raw hamburger at a restaurant. The ‫ ﺇﻥ ﺍﻟﻌﺎﻣﻞ ﺍﻟﻤﺴﺒﺐ‬.‫ﻣﻦ ﺗﻨﺎﻭﻟﻪ ﻟﻮﺟﺒﺔ ﻫﺎﻣﺒﺮﻏﺮ ﻓﻲ ﻣﻄﻌﻢ‬
most likely infectious cause is :‫ﺍﻷﻛﺜﺮ ﺗﺮﺟﻴﺤﺎ ﻫﻮ‬
a. Toxoplasma
.‫ ﺍﻟﻤﻘﻮﺳﺎﺕ‬.A
b. Cytomegalovirus
c. E. coli
.‫ ﺍﻟﻔﻴﺮﻭﺱ ﺍﻟﻤﻀﺨﻢ ﻟﻠﺨﻼﻳﺎ‬.B
d. Salmonella .‫ ﺍﺷﺮﻳﻜﻴﺎﺕ ﻗﻮﻟﻮﻧﻴﺔ‬.C
e. Clostridium .‫ ﺳﺎﻟﻤﻮﻧﻴﻼ‬.D
.‫ ﺍﻟﻤﻄﺜﻴﺎﺕ‬.E
A
On observe une léthargie, un malaise et de la fatigue chez
un patient deux semaines après avoir mangé un
hamburger cru au restaurant. La cause infectieuse la plus
probable est
a. Toxoplasma
b. Cytomégalovirus
c. E. coli
d. Salmonella
e. Clostridium

57 A
140. A 65-year-old woman with a long history of uncontrolled ‫ ﻋﺎﻣﺎ ﺗﻌﺎﻧﻲ ﻣﻨﺬ ﺯﻣﻦ ﻣﻦ ﺍﺭﺗﻔﺎﻉ ﺿﻐﻂ‬65 ‫ﺍﻣﺮﺃﺓ ﻋﻤﺮﻫﺎ‬
hypertension and valvular heart disease presents for ‫ ﺣﻀﺮﺕ ﻣﻦ ﺃﺟﻞ‬،‫ﻏﻴﺮ ﻣﻨﻀﺒﻂ ﻭﻣﺮﺽ ﻗﻠﺐ ﺻﻤﺎﻣﻲ‬
evaluation. She is fatigued and complains of swelling in ‫ ﻫﻲ ﻣﺘﻌﺒﺔ ﻭﺗﺸﻜﻮ ﻣﻦ ﺗﻮﺭﻡ ﻓﻲ ﺳﺎﻗﻴﻬﺎ ﻭﺿﻴﻖ ﻓﻲ‬.‫ﺍﻟﺘﻘﻴﻴﻢ‬
her legs and shortness of breath. Which of the following is
‫ ﺃﻱ ﻣﻦ ﺍﻟﺘﺎﻟﻲ ﻫﻮ ﺍﻟﺴﻤﺔ ﺍﻟﻤﻤﻴﺰﺓ ﻟﻘﺼﻮﺭ ﺑﻄﻴﻨﻲ‬.‫ﺍﻟﺘﻨﻔﺲ‬
the distinguishing feature of left ventricular failure?
A. elevated liver enzymes
:‫ﺃﻳﺴﺮ؟‬
B. pulmonary edema .‫ ﺍﺭﺗﻔﺎﻉ ﺧﻤﺎﺋﺮ ﺍﻟﻜﺒﺪ‬.A
C. ascites .‫ ﻭﺫﻣﺔ ﺭﺋﻮﻳﺔ‬.B
D. peripheral edema .‫ ﺍﺳﺘﺴﻘﺎء‬.C
E. jugular venous distention .‫ ﻭﺫﻣﺔ ﻣﺤﻴﻄﻴﺔ‬.D
.‫ ﺍﻧﺘﻔﺎﺥ ﺍﻟﻮﺭﻳﺪ ﺍﻟﻮﺩﺍﺟﻲ‬.E
Une femme de 65 ans ayant de longs antécédents B
d’hypertension non contrôlée et de cardiopathie
valvulaire se présente pour évaluation. Elle est fatiguée et
se plaint de gonflement des jambes et d’essoufflement.
Lequel de
Ce qui suit est une caractéristique distinctive de
l'insuffisance ventriculaire gauche?
A. enzymes hépatiques élevées
B. œdème pulmonaire
C. ascite
D. œdème périphérique
E. Distension veineuse jugulaire
141. Burkitt’s lymphoma is characterized by elevated “early ‫ﺗﺘﺼﻒ ﻟﻴﻤﻔﻮﻣﺎ ﺑﻴﺮﻛﻴﺖ ﺑﺎﺭﺗﻔﺎﻉ " ﺍﻟﻤﺴﺘﻀﺪ ﺍﻟﻤﺒﻜﺮ" ﻣﻊ ﻧﻤﻂ‬
antigen” tests with a restricted pattern of fluorescence. .‫ﻣﺤﺪﺩ ﻣﻦ ﺍﻟﺘﺄﻟﻖ‬
This disease is caused by :‫ﻫﺬﺍ ﺍﻟﻤﺮﺽ ﺳﺒﺒﻪ‬
a. Cytomegalovirus
.‫ ﺍﻟﻔﻴﺮﻭﺱ ﺍﻟﻤﻀﺨﻢ ﻟﻠﺨﻼﻳﺎ‬.A
b. B. burgdorferi
c. Epstein-Barr virus
.‫ ﺑﻮﺭﻟﻴﻪ ﺑﻮﺭﻏﺪﺭﻓﻴﺮﻱ‬.B
d. Lymphogranuloma venereum .‫ ﻓﻴﺮﻭﺱ ﺇﻳﺐ ﺷﺘﺎﻳﻦ ﺑﺎﺭ‬.C
e. Herpes simplex virus .‫ ﻭﺭﻡ ﺣﺒﻴﺒﻲ ﻟﻤﻔﻲ ﻣﻨﻘﻮﻝ ﺟﻨﺴﻴﺎ‬.D
.‫ ﻓﻴﺮﻭﺱ ﺍﻟﺤﻸ ﺍﻟﺒﺴﻴﻂ‬.E C
Le lymphome de Burkitt se caractérise par des tests
«d'antigène précoce» élevés avec un type de fluorescence
restreint. Cette maladie est causée par
a. Cytomégalovirus
b. B. burgdorferi
c. Virus d'Epstein-Barr
d. Lymphogranuloma venereum
e. Virus de l'herpès simplex

58 A
142. A 42-year-old man admitted with a high fever and ‫ ﻋﺎﻣﺎ ﺃﺩﺧﻞ ﺇﻟﻰ ﺍﻟﻤﺴﺘﺸﻔﻰ ﺑﺴﺒﺐ ﺍﺭﺗﻔﺎﻉ‬42 ‫ﺭﺟﻞ ﻋﻤﺮﻩ‬
leukocytosis is transferred to the intensive care unit in ‫ ﻭﻟﻘﺪ ﺗﻢ ﻧﻘﻠﻪ ﺇﻟﻰ ﻭﺣﺪﺓ ﺍﻟﻌﻨﺎﻳﺔ‬،‫ﺣﺮﺍﺭﺓ ﻭﻛﺜﺮﺓ ﺍﻟﻜﺮﻳﺎﺕ ﺍﻟﺒﻴﺾ‬
shock. Which of the following is a common finding in the ‫ ﺃﻱ ﻣﻦ ﺍﻟﺘﺎﻟﻲ ﻫﻮ ﺍﻻﺳﺘﻨﺘﺎﺝ‬.‫ﺍﻟﻤﺮﻛﺰﺓ ﻭﻫﻮ ﺑﺤﺎﻟﺔ ﺻﺪﻣﺔ‬
early stages of septic shock?
:‫ﺍﻟﺸﺎﺋﻊ ﻓﻲ ﺍﻟﻤﺮﺍﺣﻞ ﺍﻟﻤﺒﻜﺮﺓ ﻟﺼﺪﻣﺔ ﺧﻤﺞ ﺍﻟﺪﻡ؟‬
A. reduced cardiac output
B. bradycardia
.‫ ﻧﻘﺺ ﻧﺘﺎﺝ ﺍﻟﻘﻠﺐ‬.A
C. decreased systemic vascular resistance (SVR) .‫ ﺗﺒﺎﻁﺆ ﻗﻠﺐ‬.B
D. hypertension .(SVR) ‫ ﻧﻘﺺ ﺍﻟﻤﻘﺎﻭﻣﺔ ﺍﻟﻮﻋﺎﺋﻴﺔ ﺍﻟﺠﻬﺎﺯﻳﺔ‬.C
E. metabolic alkalosis .‫ ﺍﺭﺗﻔﺎﻉ ﺿﻐﻂ‬.D
.‫ ﻗﻠﻮﻧﻪ ﺍﺳﺘﻘﻼﺑﻴﺔ‬.E
C
Un homme de 42 ans admis avec une forte fièvre et une
leucocytose est transféré à l'unité de soins intensifs en état
de choc. Laquelle des constatations suivantes est
commune dans les premiers stades du choc septique?
A. débit cardiaque réduit
B. bradycardie
C. diminution de la résistance vasculaire systémique
(RVS)
D. hypertension
E. alcalose métabolique
143. A 30-year-old male patient was seen by the emergency ‫ ﻋﺎﻣﺎ ﺗﻤﺖ ﻣﻌﺎﻳﻨﺘﻪ ﻓﻲ ﻗﺴﻢ ﺍﻟﻄﻮﺍﺭﺉ ﻭﻗﺪ‬30 ‫ﻣﺮﻳﺾ ﻋﻤﺮﻩ‬
service and reported a 2-week history of a penile ulcer. He ‫ ﻟﻘﺪ‬.‫ﺃﻓﺎﺩ ﺍﻟﺘﻘﺮﻳﺮ ﻋﻦ ﺇﺻﺎﺑﺘﻪ ﺑﺘﻘﺮﺡ ﺍﻟﻘﻀﻴﺐ ﻣﻨﺬ ﺃﺳﺒﻮﻋﻴﻦ‬
noted that this ulcer did not hurt. Which one of the ‫ ﺃﻱ ﻣﻦ ﺍﻻﺳﺘﻨﺘﺎﺟﺎﺕ‬.‫ﺃﻓﺎﺩ ﺍﻟﻤﺮﻳﺾ ﺑﺄﻥ ﻫﺬﺍ ﺍﻟﺘﻘﺮﺡ ﻏﻴﺮ ﻣﺆﻟﻢ‬
following conclusions/actions is most valid?
:‫ ﺍﻹﺟﺮﺍءﺍﺕ ﻫﻲ ﺍﻷﻛﺜﺮ ﻓﻌﺎﻟﻴﺔ؟‬/ ‫ﺍﻟﺘﺎﻟﻴﺔ‬
a. Draw blood for a herpes antibody test
b. Perform a dark-field examination of the lesion
.‫ ﺳﺤﺐ ﺩﻡ ﻣﻦ ﺃﺟﻞ ﻓﺤﺺ ﺃﺿﺪﺍﺩ ﺍﻟﻬﺮﺑﺲ‬.A
c. Prescribe acyclovir for primary genital herpes .‫ ﺇﺟﺮﺍء ﺩﺭﺍﺳﺔ ﺣﻘﻞ ﺍﻟﻈﻼﻡ ﻟﻶﻓﺔ‬.B
d. Even if treated, the lesion will remain for months .‫ ﻭﺻﻒ ﺃﺳﻴﻜﻠﻮﻓﻴﺮ ﻟﻌﻼﺝ ﺍﻟﻬﺮﺑﺲ ﺍﻟﺘﻨﺎﺳﻠﻲ ﺍﻟﺒﺪﺋﻲ‬.C
e. Failure to treat the patient will have no untoward effect, .‫ ﺳﺘﺒﻘﻰ ﺍﻵﻓﺔ ﻷﺷﻬﺮ ﺭﻏﻢ ﺍﻟﻌﻼﺝ‬.D
as this is a self-limiting ‫ ﺍﻟﻔﺸﻞ ﺑﻌﻼﺝ ﺍﻟﻤﺮﻳﺾ ﻟﻦ ﻳﻜﻮﻥ ﻟﻪ ﺃﺛﺮ ﻏﻴﺮ‬.E
infection .‫ ﻷﻥ ﻫﺬﻩ ﺍﻹﺻﺎﺑﺔ ﻫﻲ ﻣﺤﺪﻭﺩﺓ ﺫﺍﺗﻴﺎ‬،‫ﻣﺮﻏﻮﺏ ﺑﻪ‬
Un patient âgé de 30 ans admis au service d'urgence a B
signalé un ulcère du pénis au cours des deux dernières
semaines. Il a noté que cet ulcère ne faisait pas mal.
Laquelle des conclusions / actions suivantes est la plus
valable?
a. Prélevez du sang pour un test d'anticorps anti-herpès
b. Effectuer un examen microscopique de la lésion sur
fond noir
c. Prescrire l'acyclovir pour l'herpès génital primaire
d. Même si elle est traitée, la lésion restera pendant des
mois
e. L'absence de traitement du patient n'aura pas d'effet
indésirable, car il s'agit d'une infection limitée

59 A
144. A 35-year-old man presents with acute low back pain ‫ ﻋﺎﻣﺎ ﻳﺸﻜﻮ ﻣﻦ ﺃﻟﻢ ﺣﺎﺩ ﻓﻲ ﺃﺳﻔﻞ ﺍﻟﻈﻬﺮ ﺑﻌﺪ‬35 ‫ﺭﺟﻞ ﻋﻤﺮﻩ‬
after lifting a couch in his home. Pain is in the ‫ ﻳﺘﻮﺿﻊ ﺍﻷﻟﻢ ﻓﻲ ﺍﻟﻤﺴﺎﺣﺔ ﺍﻟﻘﻄﻨﻴﺔ‬.‫ﺭﻓﻌﻪ ﻷﺭﻳﻜﺔ ﻓﻲ ﻣﻨﺰﻟﻪ‬
lumbosacral area and increases with walking and ‫ ﺃﻅﻬﺮﺕ ﺍﻟﻔﺤﻮﺹ‬.‫ﺍﻟﻌﺠﺰﻳﺔ ﻭﻳﺰﺩﺍﺩ ﺃﺛﻨﺎء ﺍﻟﺴﻴﺮ ﻭﺍﻻﻧﺤﻨﺎء‬
bending. Examination reveals paraspinal muscle spasm
‫ﺇﻳﻼﻡ ﻭﺗﺸﻨﺞ ﺍﻟﻌﻀﻠﺔ ﺟﺎﻧﺐ ﺍﻟﻔﻘﺮﺍﺕ ﻭﺍﻟﺮﻓﻊ ﺍﻟﻤﺴﺘﻘﻴﻤﻲ‬
and tenderness and negative straight leg raise bilaterally.
Lower extremity strength is intact. Which of the following
‫ ﺃﻱ ﻣﻦ ﺍﻟﺘﺎﻟﻲ ﻫﻮ‬.‫ ﻗﻮﺓ ﺍﻷﻁﺮﺍﻑ ﺍﻟﺴﻔﻠﻴﺔ ﺳﻠﻴﻤﺔ‬.‫ﻟﻠﺴﺎﻗﻴﻦ ﺳﻠﺒﻲ‬
is the best next step in managing this patient? :‫ﺍﻟﺨﻄﻮﺓ ﺍﻟﺘﺎﻟﻴﺔ ﺍﻷﻓﻀﻞ ﻓﻲ ﻋﻼﺝ ﻫﺬﺍ ﺍﻟﻤﺮﻳﺾ؟‬
A. bed rest for 1 week .‫ ﺍﻟﺮﺍﺣﺔ ﻓﻲ ﺍﻟﺴﺮﻳﺮ ﻟﻤﺪﺓ ﺃﺳﺒﻮﻉ‬.A
B. referral to an orthopedic specialist .‫ ﺍﻹﺣﺎﻟﺔ ﺇﻟﻰ ﺃﺧﺼﺎﺋﻲ ﺟﺮﺍﺣﺔ ﻋﻈﺎﻡ‬.B
C. x-ray of lumbosacral spine .‫ ﺃﺷﻌﺔ ﺳﻴﻨﻴﺔ ﻟﻠﻔﻘﺮﺍﺕ ﺍﻟﻘﻄﻨﻴﺔ ﺍﻟﻌﺠﺰﻳﺔ‬.C
D. treatment with anti-inflammatory medication and ‫ ﺍﻟﻤﻌﺎﻟﺠﺔ ﺑﻤﻀﺎﺩ ﺍﻟﺘﻬﺎﺏ ﻭﺍﻟﻌﻮﺩﺓ ﺍﻟﺘﺪﺭﻳﺠﻴﺔ ﻟﻠﻨﺸﺎﻁ‬.D
gradual return to normal activity .‫ﺍﻟﻄﺒﻴﻌﻲ‬
E. referral to a pain clinic .‫ ﺍﻹﺣﺎﻟﺔ ﺇﻟﻰ ﻋﻴﺎﺩﺓ ﻟﻌﻼﺝ ﺍﻷﻟﻢ‬.E
D
Un homme de 35 ans se présente avec une douleur aiguë
au bas du dos après avoir soulevé un canapé chez lui. La
douleur est dans la région lombo-sacrée et augmente avec
la marche et la flexion. L'examen révèle des spasmes et de
la sensibilité des muscles paraspinaux et un test d’
élévation négative de la jambe droite au niveau bilatéral.
La force des membres inférieurs est intacte. Quelle est la
meilleure proposition pour de ce patient?
A. repos au lit pendant 1 semaine
B. référence à un orthopédiste
C. Radiographie de la colonne lombo-sacrée
D. traitement par anti-inflammatoire et reprise
progressive de l'activité
E. référer à une clinique de la douleur
145. In a 33 year old hypertensive pregnant patient , G1P0 , ‫ ﻋﺎﻣﺎ ﻭﺗﻌﺎﻧﻲ ﻣﻦ ﺍﺭﺗﻔﺎﻉ‬33 ‫ ﻋﻤﺮﻫﺎ‬،G1P0 ،‫ﻣﺮﻳﻀﺔ ﺣﺎﻣﻞ‬
what are the signs that would indicate a risk of an :‫ ﻣﺎ ﻫﻲ ﺍﻟﻌﻼﻣﺎﺕ ﺍﻟﺘﻲ ﺗﺸﻴﺮ ﺇﻟﻰ ﺧﻄﺮ ﺍﻻﺭﺗﻌﺎﺝ؟‬،‫ﺿﻐﻂ‬
eclampsia: .‫ﺍﻟﻮﺗﺮﻳﺔ‬-‫ ﺍﻟﻐﺎء ﺍﻟﻤﻨﻌﻜﺴﺎﺕ ﺍﻟﻌﻈﻤﻴﺔ‬.A
A. Abolition of osteo-tendinous reflexes
.‫ ﻏﺜﻴﺎﻥ‬.B
B. Nausea
C. Ear-ringing
.‫ ﻁﻨﻴﻦ ﺃﺫﻧﻴﻦ‬.C
D. Weight gain of 4 kg in three days .‫ ﻛﻎ ﻓﻲ ﺛﻼﺛﺔ ﺃﻳﺎﻡ‬4 ‫ ﺯﻳﺎﺩﺓ ﻭﺯﻥ‬.D
E. Calf cramps .‫ ﺗﺸﻨﺠﺎﺕ ﺍﻟﺴﺎﻕ‬.E
C
Quels sont les signes faisant craindre l'imminence d'une
crise d'éclampsie chez une femme enceinte G1P0,
hypertendue :
A. Abolition des réflexes ostéo-tendineux
B. Nausées
C. Bourdonnements d'oreille
D. Prise de poids de 4 kg en trois jours
E. Crampes aux mollets

60 A
146. Among the following, and according to the ‫ ﻭﻭﻓﻘﺎ ﻟﻠﻔﻴﺰﻳﻮﻟﻮﺟﻴﺎ ﺍﻟﻤﺮﺿﻴﺔ ﻟﻤﻘﺪﻣﺎﺕ‬،‫ﻣﻦ ﺑﻴﻦ ﻣﺎ ﻳﻠﻲ‬
pathophysiology of preeclampsia, what would be the ‫ ﻣﺎ ﻫﻮ ﺍﻟﺪﻭﺍء ﺍﻟﺬﻱ ﻳﺠﺐ ﻋﺪﻡ ﺇﻋﻄﺎﺅﻩ ﻟﺤﺎﻣﻞ ﻓﻲ‬،‫ﺍﻻﺭﺗﻌﺎﺝ‬
drug not to give in a preeclamptic at 26 weeks: :‫ ﻭﺗﻌﺎﻧﻲ ﻣﻦ ﻣﻘﺪﻣﺎﺕ ﺍﻻﺭﺗﻌﺎﺝ؟‬26 ‫ﺍﻷﺳﺒﻮﻉ‬
a. calcium channel
.‫ ﺣﺎﺟﺒﺎﺕ ﻗﻨﻮﺍﺕ ﺍﻟﻜﺎﻟﺴﻴﻮﻡ‬.A
b. diuretics
c. Alpha methyl dopa
.‫ ﻣﺪﺭﺍﺕ‬.B
d. labetolol .‫ ﺃﻟﻔﺎ ﻣﺘﻴﻞ ﺩﻭﺑﺎ‬.C
e. Central antihypertensives .‫ ﻻﺑﻴﺘﺎﻟﻮﻝ‬.D
.‫ ﺧﺎﻓﻀﺎﺕ ﺿﻐﻂ ﻣﺮﻛﺰﻳﺔ‬.E B
Parmi les suivants , et selon la physiopathologie de la
preeclampsie, quel est le medicament à ne pas donner
chez une preeclamptique à 26 SA
a. Anticalciques
b. Diuretiques
c. Alpha methyl dopa
d. Labetolol
e. Antihypertenseurs centraux
147. Which of the following is the most common cause of ‫ﺃﻱ ﻣﻦ ﺍﻟﺘﺎﻟﻲ ﻫﻮ ﺍﻟﺴﺒﺐ ﺍﻷﻛﺜﺮ ﺷﻴﻮﻋﺎ ﻓﻲ ﺍﻟﻤﺘﻼﺯﻣﺔ‬
nephrotic syndrome? :‫ﺍﻟﻨﻔﺮﻭﻧﻴﺔ؟‬
A. diabetes mellitus .‫ ﺩﺍء ﺍﻟﺴﻜﺮﻱ‬.A
B. Hodgkin's lymphoma
.‫ ﺍﻟﻮﺭﻡ ﺍﻟﻠﻤﻔﺎﻭﻱ ﻫﻮﺩﺟﻜﻦ‬.B
C. heroin abuse
D. malignant hypertension
.‫ ﺗﻌﺎﻁﻲ ﺍﻟﻬﻴﺮﻭﻳﻦ‬.C
E. renal failure .‫ ﺍﺭﺗﻔﺎﻉ ﺿﻐﻂ ﺧﺒﻴﺚ‬.D
.‫ ﻗﺼﻮﺭ ﻛﻠﻮﻱ‬.E A
Laquelle des causes suivantes est la cause la plus
fréquente de syndrome néphrotique?
A. diabète sucré
B Lymphome de. Hodgkin
C. abus d'héroïne
D. hypertension maligne
E. insuffisance rénale
148. A 24-year-old man is found to be seropositive for HIV on ‫ ﺑﻌﺪ‬HIV ‫ ﻋﺎﻣﺎ ﺗﺒﻴﻦ ﺃﻧﻪ ﻣﺼﺎﺏ ﺑﻔﻴﺮﻭﺱ‬24 ‫ﺭﺟﻞ ﻋﻤﺮﻩ‬
a screening test. Which of the following opportunistic ‫ ﺃﻱ ﻣﻦ ﺍﻹﺻﺎﺑﺎﺕ ﺍﻻﻧﺘﻬﺎﺯﻳﺔ ﺍﻟﺘﺎﻟﻴﺔ ﻫﻲ‬.‫ﺧﻀﻮﻋﻪ ﻟﻠﻔﺤﺺ‬
infections is most likely to develop in this patient? :‫ﺍﻷﻛﺜﺮ ﺗﺮﺟﻴﺤﺎ ﺃﻥ ﺗﺤﺪﺙ ﻋﻨﺪ ﻫﺬﺍ ﺍﻟﻤﺮﻳﺾ؟‬
A. Pneumocystis carinii pneumonia
.‫ ﺍﻟﻤﺘﻜﻴﺴﺔ ﺍﻟﻜﺎﺭﻳﻨﻴﺔ ﺍﻟﺮﺋﻮﻳﺔ‬.A
B. Candida albicans fungemia
C. disseminated Mycobacterium avium intracellulare
.‫ ﻓﻄﺮ ﻣﺒﻴﻀﺎﺕ ﺍﻟﺒﻴﺾ‬.B
infection .‫ ﺇﺻﺎﺑﺔ ﺑﺎﻟﻤﺘﻔﻄﺮﺍﺕ ﺍﻟﻄﻴﺮﻳﺔ ﻣﻨﺘﺸﺮﺓ ﺩﺍﺧﻞ ﺍﻟﺨﻠﻮﻱ‬.C
D. cryptococcal meningitis .‫ ﺍﻟﺘﻬﺎﺏ ﺍﻟﺴﺤﺎﻳﺎ ﺑﺎﻟﻤﺴﺘﺨﻔﻴﺎﺕ‬.D
E. cytomegalovirus retinitis .‫ ﺍﻟﺘﻬﺎﺏ ﺍﻟﺸﺒﻜﻴﺔ ﺑﺎﻟﻔﻴﺮﻭﺱ ﺍﻟﻤﻀﺨﻢ ﻟﻠﺨﻼﻳﺎ‬.E
A
Un homme de 24 ans s'avère séropositif pour le VIH lors
d'un test de dépistage. Laquelle des infections
opportunistes suivantes est la plus susceptible de se
développer chez ce patient?
A.Pneumonie à pneumocystis carinii
B. Fongémie à Candida albicans
C. infection disséminée à Mycobacterium avium
intracellulare
D. méningite à cryptocoque
E. Rétinite à cytomégalovirus

61 A
149. A 17 year old man comes to an emergency room because ‫ ﻋﺎﻣﺎ ﺣﻀﺮ ﺇﻟﻰ ﻏﺮﻓﺔ ﺍﻟﻄﻮﺍﺭﺉ ﺑﺴﺒﺐ‬17 ‫ﺭﺟﻞ ﻋﻤﺮﻩ‬
of an infected dog bite. The wound is found to contain ‫ ﻟﻘﺪ ﺗﺒﻴﻦ ﺃﻥ ﺍﻟﺠﺮﺡ ﻳﺤﺘﻮﻱ ﻋﻠﻰ ﻋﺼﻴﺎﺕ ﺳﻠﺒﻴﺔ‬.‫ﻋﻀﺔ ﻛﻠﺐ‬
small Gram-negative rods. The most likely cause of :‫ ﺍﻹﺻﺎﺑﺔ ﺍﻟﺠﺮﺛﻮﻣﻴﺔ ﺍﻷﻛﺜﺮ ﺗﺮﺟﻴﺤﺎ ﻫﻲ‬.‫ﺍﻟﻐﺮﺍﻡ‬
infection is
.‫ ﺍﺷﺮﻳﻜﻴﺎﺕ ﻗﻮﻟﻮﻧﻴﺔ‬.A
a. E. coli
b. H. influenzae
.‫ ﺍﻟﻤﺴﺘﺪﻣﻴﺎﺕ ﺍﻟﻨﺰﻟﻴﺔ‬.B
c. Pasteurella multocida .‫ ﺍﻟﺒﺎﺳﺘﻮﺭﻳﻠﻼ ﺍﻟﻘﺘﺎﻟﺔ‬.C
d. Brucella canis .‫ ﺍﻟﺒﺮﻭﺳﻴﻠﺔ ﺍﻟﻜﻠﺒﻴﺔ‬.D
e. Klebsiella pneumoniae .‫ ﺍﻟﻜﻠﺒﺴﻴﻠﻼ ﺍﻟﺮﺋﻮﻳﺔ‬.E
C
Un homme de 17 ans se présente à une salle d'urgence à
cause d'une morsure de chien infectée. La plaie contient
de bacilles à Gram négatif. La cause d'infection la plus
probable est
a. E. coli
b. H. influenzae
c. Pasteurella multocida
d. Brucella canis
e. Klebsiella pneumonia
150. A 62-year-old man presents with weakness and aching in ‫ ﻋﺎﻣﺎ ﻳﺸﻜﻮ ﻣﻦ ﺿﻌﻒ ﻭﺃﻟﻢ ﻓﻲ ﺍﻟﻮﺭﻛﻴﻦ‬62 ‫ﺭﺟﻞ ﻋﻤﺮﻩ‬
his hips and shoulders which has progressed ‫ ﻟﻘﺪ‬.‫ﻭﺍﻟﻜﺘﻔﻴﻦ ﻭﺍﻟﺬﻱ ﺗﻔﺎﻗﻢ ﻋﻠﻰ ﻣﺪﻯ ﺍﻷﺷﻬﺮ ﺍﻟﻘﻠﻴﻠﺔ ﺍﻟﻤﺎﺿﻴﺔ‬
over the last few months. He reports generalized fatigue ،‫ ﺗﺸﻤﻞ ﺍﻹﺟﺮﺍءﺍﺕ ﺗﻌﺪﺍﺩ ﺩﻡ ﺷﺎﻣﻞ‬.‫ﺃﻓﺎﺩ ﻋﻦ ﻭﻫﻦ ﻭﺗﻌﺐ ﻣﻌﻤﻢ‬
and malaise. Workup includes a normal complete blood
‫ ﺃﻱ ﻣﻦ‬.102 ‫ ﻭﺳﺮﻋﺔ ﺗﺮﺳﺐ‬،‫ﻓﺤﻮﺹ ﻟﻠﻜﺒﺪ ﻭﺍﻟﻜﻠﻴﺔ ﺍﻟﻄﺒﻴﻌﻴﺔ‬
count, kidney and liver tests, and a sedimentation rate of
102. Which of the following is
:‫ﺍﻟﺘﺎﻟﻲ ﻫﻮ ﺍﻟﺘﺸﺨﻴﺺ ﺍﻷﻛﺜﺮ ﺗﺮﺟﻴﺤﺎ؟‬
the most likely diagnosis? .‫ ﺫﺋﺒﺔ ﺣﻤﺎﻣﻴﺔ ﺟﻬﺎﺯﻳﺔ‬.A
A. systemic lupus erythematous .‫ ﺩﺍء ﺳﻜﺮﻱ‬.B
B. diabetes .‫ ﻭﺭﻡ ﺣﺒﻴﺒﻲ ﻭﻏﻨﺮﻱ‬.C
C. Wegener's granulomatosis .‫ ﺃﻟﻢ ﻣﺘﻌﺪﺩ ﺭﺛﻴﻮﻱ‬.D
D. polymyalgia rheumatica .‫ ﺩﺍء ﻏﺮﻳﻔﺰ‬.E
E. Graves' disease
D
Un homme de 62 ans présente une faiblesse et des
douleurs dans les hanches et les épaules qui ont progressé
au cours des derniers mois. Il fait état d’une fatigue et
d’un malaise généralisés. Le bilan comprend une FNS ,
tests rénaux et hépatiques normaux , et une vitesse de
sédimentation de 102. Lequel des éléments suivants est le
diagnostic le plus probable?
A. lupus erythemateux systémique
B. diabète
C Granulomatose de. Wegener
D. polymyalgia rhumatica
E. maladie de Graves

62 A
151. Group B streptococcus sepsis in an infant is preventable. ‫ ﻳﻤﻜﻦ ﺍﻟﻮﻗﺎﻳﺔ ﻣﻨﻪ ﻋﻨﺪ‬B ‫ﺇﻥ ﺗﺠﺮﺛﻢ ﺍﻟﺪﻡ ﺑﻤﺠﻤﻮﻋﺔ ﺍﻟﻌﻘﺪﻳﺎﺕ‬
Which one of the following procedures is most likely to ‫ ﺃﻱ ﻣﻦ ﺍﻟﺘﺎﻟﻲ ﻫﻮ ﺍﻹﺟﺮﺍء ﺍﻷﻓﻀﻞ ﺍﻟﺬﻱ ﻳﺠﺐ‬.‫ﺍﻷﻁﻔﺎﻝ‬
reduce the incidence of group B streptococcal disease? ‫ﺍﺗﺨﺎﺫﻩ ﻟﻠﺤﺪ ﻣﻦ ﺣﺎﻻﺕ ﺍﻹﺻﺎﺑﺔ ﺑﺎﻷﻣﺮﺍﺽ ﺍﻟﻌﻘﺪﻳﺔ ﻣﺠﻤﻮﻋﺔ‬
a. Intrapartum antibiotic treatment
:‫ ؟‬B
b. Use of a polysaccharide vaccine
c. Screening of pregnant females in the second trimester
.‫ ﺍﻟﻤﻌﺎﻟﺠﺔ ﺑﻤﻀﺎﺩ ﺣﻴﻮﻱ ﺃﺛﻨﺎء ﺍﻟﻮﺿﻊ‬.A
d. Identification of possible high-risk births .‫ ﺍﺳﺘﺨﺪﺍﻡ ﻟﻘﺎﺡ ﻋﺪﻳﺪ ﺍﻟﺴﻜﺎﺭﻳﺪ‬.B
e. Screening of pregnant females at the first office visit, ‫ﺼﻲ ﺍﻟﻨﺴﺎء ﺍﻟﺤﻮﺍﻣﻞ ﻓﻲ ﺍﻟﻔﺼﻞ ﺍﻟﺜﺎﻧﻲ ﻣﻦ‬ ً ‫ ﺗﻘ‬.C
usually during the first trimester .‫ﺍﻟﺤﻤﻞ‬
.‫ ﺗﺤﺪﻳﺪ ﺍﻟﻮﻻﺩﺍﺕ ﻋﺎﻟﻴﺔ ﺍﻟﺨﻄﻮﺭﺓ ﺍﻟﻤﺤﺘﻤﻠﺔ‬.D
A
La septicémie à streptocoque du groupe B chez un ،‫ﺼﻲ ﺍﻟﻨﺴﺎء ﺍﻟﺤﻮﺍﻣﻞ ﻋﻨﺪ ﺍﻟﺰﻳﺎﺭﺓ ﺍﻷﻭﻟﻰ ﻟﻠﻌﻴﺎﺩﺓ‬ً ‫ ﺗﻘ‬.E
nourrisson est évitable. Laquelle des procédures suivantes .‫ﻋﺎﺩﺓ ﻓﻲ ﺍﻟﻔﺼﻞ ﺍﻷﻭﻝ ﻣﻦ ﺍﻟﺤﻤﻞ‬
est la plus susceptible de réduire l’incidence de la maladie
à streptocoque du groupe B?
a. Traitement antibiotique intrapartum
b. Utilisation d'un vaccin polysaccharidique
c. Dépistage des femmes enceintes au deuxième trimestre
d. Identification d'éventuelles naissances à haut risque
e. Dépistage des femmes enceintes lors de la première
visite, généralement au cours du premier trimestre
152. A 54-year-old woman is brought to the ER with ‫ ﻋﺎﻣﺎ ﺃﺣﻀﺮﺕ ﺇﻟﻰ ﻗﺴﻢ ﺍﻟﻄﻮﺍﺭﺉ ﻭﻫﻲ‬54 ‫ﺍﻣﺮﺃﺓ ﻋﻤﺮﻫﺎ‬
palpitations and dizziness. She has a history of ‫ ﻟﻘﺪ‬.‫ ﻫﻲ ﺗﻌﺎﻧﻲ ﻣﺴﺒﻘﺎ ﻣﻦ ﻻ ﻧﻈﻤﻴﺔ‬.‫ﺗﺸﻜﻮ ﻣﻦ ﺧﻔﻘﺎﻥ ﻭﺩﻭﺍﺭ‬
arrhythmia. Adenosine is given and the patient converts ‫ ﻣﺎ ﻫﻮ‬.‫ﺃﻋﻄﻴﺖ ﺃﺩﻳﻨﻮﺯﻳﻦ ﺣﻴﺚ ﺗﻢ ﺍﻟﺘﺤﻮﻳﻞ ﺇﻟﻰ ﺍﻟﻨﻈﻢ ﺍﻟﺠﻴﺒﻲ‬
to a sinus rhythm. With which of the following rhythms
:‫ﻧﻮﻉ ﺍﻟﻨﻈﻢ ﺍﻟﺘﻲ ﻛﺎﻧﺖ ﻣﺼﺎﺑﺔ ﺑﻪ ﺍﻟﻤﺮﻳﻀﺔ ﻗﺒﻞ ﻫﺬﺍ ﺍﻟﻌﻼﺝ؟‬
did this patient most likely present to the ER?
A. ventricular tachycardia
.‫ ﺗﺴﺎﺭﻉ ﻗﻠﺐ ﺑﻄﻴﻨﻲ‬.A
B. atrial fibrillation .‫ ﺭﺟﻔﺎﻥ ﺃﺫﻳﻨﻲ‬.B
C. atrial flutter .‫ ﺭﻓﺮﻓﺔ ﺃﺫﻳﻨﻴﻪ‬.C
D. paroxysmal supraventricular tachycardia .‫ ﺗﺴﺎﺭﻉ ﻗﻠﺐ ﻓﻮﻕ ﺑﻄﻴﻨﻲ ﺍﻧﺘﻴﺎﺑﻲ‬.D
E. ventricular fibrillation .‫ ﺭﺟﻔﺎﻥ ﺑﻄﻴﻨﻲ‬.E
D
Une femme de 54 ans est amenée aux urgences avec des
palpitations et des vertiges. Elle a des antécédents
d'arythmie. L'adénosine est administrée et le patient
adopte alors un rythme sinusal. Quel serait pour vous le
rythme de présentation aux urgences avant ce traitement?
A. tachycardie ventriculaire
B. fibrillation auriculaire
C. flutter auriculaire
D. Tachycardie supraventriculaire paroxystique
E. fibrillation ventriculaire
153. The most common portal of entry for C. tetani, the cause of ‫ ﺍﻟﺘﻲ‬،‫ﺇﻥ ﻣﻤﺮ ﺍﻟﺪﺧﻮﻝ ﺍﻷﻛﺜﺮ ﺷﻴﻮﻋﺎ ﻟﻠﻤﻄﺜﻴﺎﺕ ﺍﻟﻜﺰﺍﺯﻳﺔ‬
tetanus, is the :‫ ﻫﻮ‬،‫ﺗﺴﺒﺐ ﺩﺍء ﺍﻟﻜﺰﺍﺯ‬
a. Skin .‫ ﺍﻟﺠﻠﺪ‬.A
b. Gastrointestinal tract
.‫ ﺍﻟﺠﻬﺎﺯ ﺍﻟﻬﻀﻤﻲ‬.B
c. Respiratory tract
d. Genital tract
.‫ ﺍﻟﺠﻬﺎﺯ ﺍﻟﺘﻨﻔﺴﻲ‬.C
e. Nasal tract .‫ ﺍﻟﺠﻬﺎﺯ ﺍﻟﺘﻨﺎﺳﻠﻲ‬.D
.‫ ﺍﻟﺠﻬﺎﺯ ﺍﻷﻧﻔﻲ‬.E A
Le point d’entrée le plus courant pour C. tetani, , cause du
tétanos, est le
a. Peau
b. Tube digestif
c. Voies respiratoires
d. Tractus génital
e. Voie nasale

63 A
154. Amebae that are parasitic in humans are found in the oral ‫ ﻓﻲ ﺍﻟﺠﻮﻑ‬،‫ ﺍﻟﺘﻲ ﺗﺘﻄﻔﻞ ﻋﻠﻰ ﺍﻟﺒﺸﺮ‬،‫ﺗﻢ ﺍﻟﻌﺜﻮﺭ ﻋﻠﻰ ﺍﻷﻣﻴﺒﺎ‬
cavity and the intestinal tract. Which one of the following ‫ ﺃﻱ ﻣﻦ ﺍﻟﻌﺒﺎﺭﺍﺕ ﺍﻟﺘﺎﻟﻴﺔ ﺗﺼﻒ‬.‫ﺍﻟﻔﻤﻮﻱ ﻭﻓﻲ ﺍﻟﺠﻬﺎﺯ ﺍﻟﻤﻌﻮﻱ‬
statements best describes these intestinal amebae? :‫ﺑﺸﻜﻞ ﺃﻓﻀﻞ ﺍﻷﻣﻴﺒﺎ ﺍﻟﻤﻌﻮﻳﺔ؟‬
a. They are usually nonpathogenic
.‫ ﻫﻲ ﻋﺎﺩﺓ ﻏﻴﺮ ﻣﻤﺮﺿﺔ‬.A
b. They can cause peritonitis and liver abscesses
c. They are usually transmitted as trophozoites
‫ ﻫﻲ ﻳﻤﻜﻦ ﺃﻥ ﺗﺴﺒﺐ ﺍﻟﺘﻬﺎﺏ ﺍﻟﺼﻔﺎﻕ ﻭﺧﺮﺍﺟﺎﺕ‬.B
d. They occur most abundantly in the duodenum .‫ﻛﺒﺪﻳﺔ‬
e. Infection with Entamoeba histolytica is limited to the .‫ ﻫﻲ ﻋﺎﺩﺓ ﺗﻨﺘﻘﻞ ﻋﻠﻰ ﺷﻜﻞ ﻧﻮﺍﺷﻂ‬.C
intestinal tract .‫ ﻫﻲ ﺗﺘﻮﺍﺟﺪ ﺑﺸﻜﻞ ﻛﺒﻴﺮ ﻓﻲ ﺍﻻﺛﻨﻲ ﻋﺸﺮﻱ‬.D
‫ ﺇﻥ ﺍﻹﺻﺎﺑﺔ ﺑﺎﻟﻤﺘﺤﻮﻻﺕ ﺍﻟﺤﺎﻟﺔ ﻟﻠﻨﺴﺞ ﺗﻘﺘﺼﺮ ﻋﻠﻰ‬.E
Les amibes parasitaires chez l'homme se trouvent dans la .‫ﺍﻟﺠﻬﺎﺯ ﺍﻟﻤﻌﻮﻱ‬ B
cavité buccale et le tractus intestinal. Laquelle des
affirmations suivantes décrit le mieux ces amibes
intestinales?
a. Ils sont généralement non pathogènes
b. Ils peuvent causer une péritonite et des abcès du foie
c. Ils sont généralement transmis sous forme de
trophozoïtes
d. Ils se produisent le plus abondamment dans le
duodénum
e. L’infection à Entamoeba histolytica se limite au tractus
intestinal
155. A 52-year-old woman has had diabetes mellitus since .‫ ﻋﺎﻣﺎ ﺗﻌﺎﻧﻲ ﻣﻦ ﺩﺍء ﺳﻜﺮﻱ ﻣﻨﺬ ﺍﻟﻄﻔﻮﻟﺔ‬52 ‫ﺍﻣﺮﺃﺓ ﻋﻤﺮﻫﺎ‬
childhood. She has controlled her glucose well and kept ‫ﻫﻲ ﺗﺴﻴﻄﺮ ﻋﻠﻰ ﻣﺴﺘﻮﻯ ﺍﻟﻐﻠﻮﻛﻮﺯ ﺑﺸﻜﻞ ﺟﻴﺪ ﻭﺗﺤﺎﻓﻆ ﻋﻠﻰ‬
her glycosylated hemoglobin (HbA1C) below 7% (normal, ‫ ﻷﻱ ﻣﻦ‬.(%6-2 =‫ )ﺍﻟﻄﺒﻴﻌﻲ‬%7 ‫ ﺗﺤﺖ‬HbA1C
2-6%). For which of the following complications is she still
‫ ﻋﻠﻰ ﺍﻟﺮﻏﻢ‬،‫ﺍﻟﻤﻀﺎﻋﻔﺎﺕ ﺍﻟﺘﺎﻟﻴﺔ ﻫﻲ ﻻ ﺗﺰﺍﻝ ﻣﻌﺮﺿﺔ ﻟﻠﺨﻄﺮ‬
at risk, despite excellent glucose control?
A. coronary heart disease
:‫ﻣﻦ ﺍﻟﺴﻴﻄﺮﺓ ﺍﻟﻤﻤﺘﺎﺯﺓ ﻋﻠﻰ ﺍﻟﻐﻠﻮﻛﻮﺯ؟‬
B. blindness .‫ ﻣﺮﺽ ﻗﻠﺒﻲ ﺗﺎﺟﻲ‬.A
C. peripheral neuropathy .‫ ﺍﻟﻌﻤﻰ‬.B
D. peripheral vascular disease .‫ ﺍﻋﺘﻼﻝ ﻋﺼﺒﻲ ﻣﺤﻴﻄﻲ‬.C
E. Amputation of lower limb .‫ ﻣﺮﺽ ﻭﻋﺎﺋﻲ ﻣﺤﻴﻄﻲ‬.D
.‫ ﺑﺘﺮ ﻟﻠﻄﺮﻑ ﺍﻟﺴﻔﻠﻲ‬.E A
Une femme de 52 ans est atteinte de diabète sucré depuis
son enfance. Elle a bien contrôlé sa glycémie est maintenu,
son hémoglobine glycosylé (HbA1C) est inférieure à 7%
(normale, 2 à 6%). Pour laquelle des complications
suivantes est-elle toujours à risque, malgré un excellent
contrôle de la glycémie?
A. maladie coronarienne
B. la cécité
C. neuropathie périphérique
D. maladie vasculaire périphérique
E. Amputation du membre inférieur

64 A
156. A 72-year-old man has the sudden onset of suprapubic ‫ ﻋﺎﻣﺎ ﻳﺸﻜﻮ ﻣﻦ ﺑﺪء ﻣﻔﺎﺟﺊ ﻷﻟﻢ ﻓﻮﻕ ﺍﻟﻌﺎﻧﺔ‬72 ‫ﺭﺟﻞ ﻋﻤﺮﻩ‬
pain and oliguria. His temperature is 38.0°C, pulse is ‫ﺩ ﺳﺮﻋﺔ ﺍﻟﺘﻨﻔﺲ‬/100 ‫ ﺍﻟﻨﺒﺾ‬،‫ﻡ‬°38 ‫ ﺣﺮﺍﺭﺗﻪ‬.‫ﻭﻗﻠﺔ ﺑﻮﻝ‬
100/min, respiration rate is 12/min, and BP is 110/72 ‫ ﻟﻮﺣﻆ ﻋﻨﺪ ﻓﺤﺺ‬.‫ ﻣﻠﻢ ﺯﺋﺒﻖ‬72/110 ‫ ﻭﺿﻐﻂ ﺍﻟﺪﻡ‬،‫ﺩ‬/12
mmHg. Abdominal examination is remarkable only for a
‫ ﺃﻱ ﻣﻦ‬.‫ ﻭﺃﻥ ﻫﻨﺎﻙ ﺍﻧﺘﻔﺎﺥ ﻓﻲ ﺍﻟﻤﺜﺎﻧﺔ‬،‫ﺍﻟﺒﻄﻦ ﻭﺟﻮﺩ ﺇﻳﻼﻡ ﻓﻘﻂ‬
tender, distended urinary bladder. Which of the following
is the most appropriate immediate management of this
:‫ﺍﻟﺘﺎﻟﻲ ﻫﻮ ﺍﻟﺘﺪﺑﻴﺮ ﺍﻟﻌﻼﺟﻲ ﺍﻟﻔﻮﺭﻱ ﻭﺍﻷﻓﻀﻞ ﻟﻬﺬﺍ ﺍﻟﻤﺮﻳﺾ؟‬
patient? .‫ ﺃﺷﻌﺔ ﺳﻴﻨﻴﺔ ﻟﻠﺒﻄﻦ‬.A
A. plain x-ray of the abdomen .‫ ﺗﺨﻄﻴﻂ ﺍﻟﺼﺪﻯ ﻟﻠﺒﻄﻦ‬.B
B. abdominal ultrasonography .‫ ﻗﺜﻄﺮﺓ ﺇﺣﻠﻴﻠﻴﺔ‬.C
C. urethral catheter .‫ ﻓﻴﺮﻭﺳﻴﻤﻴﺪ ﻭﺭﻳﺪﻱ‬.D
D. IV furosemide .‫ ﺗﺼﻮﻳﺮ ﺣﻮﻳﻀﺔ ﻭﺭﻳﺪﻱ‬.E
E. intravenous pyelogram (IVP)
C
Un homme de 72 ans souffre soudainement d'une douleur
suprapubienne et d'une oligurie. Sa température est de
38.0 ° C, le pouls est de 100 / min, la fréquence
respiratoire de 12 / min et la pression artérielle de 110/72
mmHg. Examen abdominal n’est remarquable que pour
une vessie douleureuse, distendue.
Lequel des suivants est la gestion immédiate la plus
appropriée de ce patient?
A. rayons X de l'abdomen
B. échographie abdominale
C. cathéter urétral
D. IV furosémide
E. pyelogram intraveineux (IVP)
157. The most common way in which tuberculosis is acquired :‫ﺍﻟﻄﺮﻳﻖ ﺍﻷﻛﺜﺮ ﺷﻴﻮﻋﺎ ﻻﻛﺘﺴﺎﺏ ﺩﺍء ﺍﻟﺴﻞ ﻫﻮ ﻋﺒﺮ‬
is via the .‫ ﺍﻟﺠﻠﺪ‬.A
a. Skin .‫ ﺍﻟﺠﻬﺎﺯ ﺍﻟﻬﻀﻤﻲ‬.B
b. Gastrointestinal tract
.‫ ﺍﻟﺠﻬﺎﺯ ﺍﻟﺘﻨﻔﺴﻲ‬.C
c. Respiratory tract
d. Genital tract
.‫ ﺍﻟﺠﻬﺎﺯ ﺍﻟﺘﻨﺎﺳﻠﻲ‬.D
e. Nasal tract .‫ ﺍﻟﺠﻬﺎﺯ ﺍﻷﻧﻔﻲ‬.E
C
Le point d’entrée le plus courant pour la tuberculose
consiste en :
a. Peau
b. Tube digestif
c. Voies respiratoires
d. Tractus génital
e. Voie nasale

65 A
158. A 33-year-old woman experiences visions of flashing ‫ ﻋﺎﻣﺎ ﺗﺸﻜﻮ ﻣﻦ ﺍﻻﻧﺰﻋﺎﺝ ﻣﻦ ﺭﺅﻳﺔ‬33 ‫ﺍﻣﺮﺃﺓ ﻋﻤﺮﻫﺎ‬
lights followed by throbbing left-sided temporal pain and ‫ﺍﻷﺿﻮﺍء ﺍﻟﺴﺎﻁﻌﺔ ﻭﺍﻟﺘﻲ ﻳﻠﻴﻬﺎ ﺃﻟﻢ ﻧﺎﺑﺾ ﻓﻲ ﺍﻟﺠﻬﺔ ﺍﻟﻴﺴﺮﻯ‬
nausea. Which of the following is the most likely ‫ ﺃﻱ ﻣﻦ ﺍﻟﺘﺎﻟﻲ ﻫﻮ ﺍﻟﺘﺸﺨﻴﺺ ﺍﻷﻛﺜﺮ‬.‫ﻟﻠﺼﺪﻍ ﻣﻊ ﻏﺜﻴﺎﻥ‬
diagnosis?
:‫ﺗﺮﺟﻴﺤﺎ؟‬
A. tension headache
B. transient ischemic attack (TIA)
.‫ ﺻﺪﺍﻉ ﻧﺎﺗﺞ ﻋﻦ ﺍﻟﺘﻮﺗﺮ‬.A
C. temporal arteritis .‫ ﻧﻮﺑﺔ ﺇﻗﻔﺎﺭﻳﻪ ﻋﺎﺑﺮﺓ‬.B
D. migraine headache .‫ ﺍﻟﺘﻬﺎﺏ ﺷﺮﺍﻳﻴﻦ ﺻﺪﻏﻴﺔ‬.C
E. Lupus .‫ ﺻﺪﺍﻉ ﻧﺼﻔﻲ‬.D
.‫ ﺫﺋﺒﺔ ﺣﻤﺎﻣﻴﺔ‬.E D
Une femme de 33 ans se plaint de vision de lumières
clignotantes suivies d'une céphalée et de nausées
palpitantes temporales du côté gauche. Lequel des
diagnostics suivants est le plus probable?
A. céphalée de tension
B. accident ischémique transitoire (AIT)
C. artérite temporale
D. migraine
E. Lupus
159. A 27-year-old female complains of dysuria and urinary ‫ ﻳﻈﻬﺮ‬.‫ ﻋﺎﻣﺎ ﺗﺸﻜﻮ ﻣﻦ ﻋﺴﺮ ﻭﺗﻜﺮﺍﺭ ﺗﺒﻮﻝ‬27 ‫ﺍﻣﺮﺃﺓ ﻋﻤﺮﻫﺎ‬
frequency. Urinalysis reveals 10-20 WBCs per high-power ‫ ﻭﺃﻥ ﻫﻨﺎﻙ‬20-10 ‫ﺗﺤﻠﻴﻞ ﺍﻟﺒﻮﻝ ﺃﻥ ﻛﺮﻳﺎﺕ ﺍﻟﺪﻡ ﺍﻟﺒﻴﻀﺎء‬
field and numerous gram-negative bacteria. She denies ،‫ ﻭﻫﻲ ﺗﻨﻔﻲ ﻭﺟﻮﺩ ﺣﺮﺍﺭﺓ‬.‫ﺍﻟﻌﺪﻳﺪ ﻣﻦ ﺍﻟﺠﺮﺍﺛﻴﻢ ﺳﻠﺒﻴﺔ ﺍﻟﻐﺮﺍﻡ‬
fevers, chills, and has no flank pain or tenderness. Which
‫ ﺃﻱ ﻣﻦ ﺍﻟﻌﺒﺎﺭﺍﺕ‬.‫ ﻭﻋﺪﻡ ﻭﺟﻮﺩ ﺃﻟﻢ ﻓﻲ ﺍﻟﺨﺎﺻﺮﺓ‬،‫ﻗﺸﻌﺮﻳﺮﺓ‬
of the following statements concerning urinary tract
infections is true?
:‫ﺍﻟﺘﺎﻟﻴﺔ ﻓﻴﻤﺎ ﻳﺘﻌﻠﻖ ﺑﺎﻟﺘﻬﺎﺏ ﺍﻟﻤﺴﺎﻟﻚ ﺍﻟﺒﻮﻟﻴﺔ ﺻﺤﻴﺤﺔ؟‬
A. A single dose of an antibiotic may be sufficient .‫ ﺟﺮﻋﺔ ﻭﺍﺣﺪﺓ ﻣﻦ ﻣﻀﺎﺩ ﺣﻴﻮﻱ ﺗﻌﺘﺒﺮ ﻛﺎﻓﻴﺔ‬.A
treatment. ‫ ﻻ ﻳﻨﺒﻐﻲ ﻣﻌﺎﻟﺠﺔ ﺍﻟﻨﺴﺎء ﺍﻟﺤﻮﺍﻣﻞ ﺍﻟﻤﺼﺎﺑﺔ ﺑﺠﺮﺍﺛﻴﻢ‬.B
B. Pregnant women with bacteriuria should not be treated .‫ﺑﻮﻟﻴﺔ ﻓﻲ ﺣﺎﻝ ﻛﺎﻧﺖ ﻻ ﻋﺮﺿﻴﺔ‬
if asymptomatic. ‫ ﻳﻨﺒﻐﻲ ﻋﻠﻰ ﺍﻟﻤﺮﺿﻰ ﺍﻟﻤﺼﺎﺑﻴﻦ ﺑﺄﻟﻢ ﻓﻲ ﺍﻟﺨﺎﺻﺮﺓ‬.C
C. Patients with flank pain or fever should be .‫ﻣﺘﺮﺍﻓﻖ ﻣﻊ ﺣﺮﺍﺭﺓ ﺃﻥ ﺗﺘﻢ ﻣﻌﺎﻟﺠﺘﻬﻢ ﻓﻲ ﺍﻟﻤﺴﺘﺸﻔﻰ‬
hospitalized. .‫ ﺗﺸﻴﺮ ﺍﻟﺒﻴﻠﺔ ﺍﻟﺪﻣﻮﻳﺔ ﺇﻟﻰ ﻣﺸﺎﻛﻞ ﻛﻠﻮﻳﺔ‬.D
D. Hematuria indicates renal involvement. ‫ ﻳﻨﺒﻐﻲ ﺇﺟﺮﺍء ﺍﻟﺘﺤﺮﻱ ﺍﻟﺒﻮﻟﻲ ﺑﻌﺪ ﺍﻻﻧﺘﻬﺎء ﻣﻦ ﺩﻭﺭﺓ‬.E
E. Urologic investigation is indicated after the treatment
.‫ﺍﻟﻌﻼﺝ‬
course is completed.
A
Une femme de 27 ans se plaint de dysurie et de
pollakiurie. L'analyse d'urine révèle 10 à 20 GB par champ
et de nombreuses bactéries à Gram négatif. Elle n’ a pas
de fièvre, frissons et n'a aucune douleur au flanc ni
aucune sensibilité. Laquelle des affirmations suivantes
concernant les infections des voies urinaires est vraie?
A. Une seule dose d'antibiotique peut constituer un
traitement suffisant.
B. Les femmes enceintes atteintes de bactériurie ne
doivent pas être traitées si elles sont asymptomatiques.
C. Les patients présentant une douleur au flanc ou une
fièvre doivent être hospitalisés.
D. L'hématurie indique une atteinte rénale.
E. Un bilan urologique est indiquée une fois le traitement
terminé.

66 A
160. Which of the following is the predominant flora of the ‫ﺃﻱ ﻣﻦ ﺍﻟﺘﺎﻟﻲ ﻫﻲ ﺍﻟﺰﻣﺮﺓ ﺍﻟﺠﺮﺛﻮﻣﻴﺔ ﺍﻟﻄﺒﻴﻌﻴﺔ ﺍﻟﺴﺎﺋﺪﺓ ﻓﻲ ﺍﻟﻔﻢ‬
mouth that is :‫ﻭﺍﻟﺘﻲ ﺗﺸﻜﻞ ﺍﻟﺴﺒﺐ ﺍﻟﺮﺋﻴﺴﻲ ﻟﺘﺴﻮﺱ ﺍﻷﺳﻨﺎﻥ؟‬
the major cause of dental caries? .α-‫ ﻣﻜﻮﺭﺍﺕ ﻋﻘﺪﻳﺔ ﺣﺎﻟﺔ ﻟﻠﺪﻡ‬.A
a. α-hemolytic streptococci
.‫ ﺍﻟﻌﺼﻴﺎﺕ ﺍﻟﻠﺒﻨﻴﺔ‬.B
b. Lactobacillus
c. S. epidermidis
.‫ ﺍﻟﻤﻜﻮﺭﺍﺕ ﺍﻟﻌﻨﻘﻮﺩﻳﺔ ﺍﻟﺒﺸﺮﻭﻳﺔ‬.C
d. E. coli .‫ ﺍﺷﺮﻳﻜﻴﺎﺕ ﻗﻮﻟﻮﻧﻴﺔ‬.D
e. B. fragilis .‫ ﺍﻟﻌﺼﻴﺎﺕ ﺍﻟﻬﺸﺔ‬.E
A
Laquelle des suivantes est la flore prédominante de la
bouche qui est la principale cause de carie dentaire?
a. streptocoques α-hémolytiques
b. Lactobacille
c. S. epidermidis
d. E. coli
e. B. fragilis
161. The bowel contains many microorganisms but the most ‫ﺗﺤﺘﻮﻱ ﺍﻷﻣﻌﺎء ﻋﻠﻰ ﺍﻟﻌﺪﻳﺪ ﻣﻦ ﺍﻟﻌﻀﻮﻳﺎﺕ ﺍﻟﺪﻗﻴﻘﺔ ﺃﻣﺎ‬
prevalent :‫ﺍﻟﺠﺮﺛﻮﻣﺔ ﺍﻷﻛﺜﺮ ﺍﻧﺘﺸﺎﺭﺍ ﻓﻬﻲ‬
bacterium is .α-‫ ﻣﻜﻮﺭﺍﺕ ﻋﻘﺪﻳﺔ ﺣﺎﻟﺔ ﻟﻠﺪﻡ‬.A
a. α-hemolytic streptococci
.‫ ﺍﻟﻌﺼﻴﺎﺕ ﺍﻟﻠﺒﻨﻴﺔ‬.B
b. Lactobacillus
c. S. epidermidis
.‫ ﺍﻟﻤﻜﻮﺭﺍﺕ ﺍﻟﻌﻨﻘﻮﺩﻳﺔ ﺍﻟﺒﺸﺮﻭﻳﺔ‬.C
d. E. coli .‫ ﺍﺷﺮﻳﻜﻴﺎﺕ ﻗﻮﻟﻮﻧﻴﺔ‬.D
e. B. fragilis .‫ ﺍﻟﻌﺼﻴﺎﺕ ﺍﻟﻬﺸﺔ‬.E
D
L'intestin contient de nombreux microorganismes mais la
bactérie la plus frequente est
a. streptocoques α-hémolytiques
b. Lactobacille
c. S. epidermidis
d. E. coli
e. B. fragilis
162. A 28-year-old man has the acute onset of colic pain in the ‫ ﻋﺎﻣﺎ ﻳﻌﺎﻧﻲ ﻣﻦ ﺑﺪء ﺣﺎﺩ ﻷﻟﻢ ﻣﻐﺼﻲ‬28 ‫ﺭﺟﻞ ﻳﺒﻠﻎ ﻣﻦ ﺍﻟﻌﻤﺮ‬
left costovertebral angle radiating into the groin, as well ‫ ﻓﻀﻼ‬،‫ﻓﻲ ﺍﻟﺰﺍﻭﻳﺔ ﺍﻟﻀﻠﻌﻴﺔ ﺍﻟﻔﻘﺮﻳﺔ ﺍﻟﻴﺴﺮﻯ ﻳﻠﺘﻒ ﻧﺤﻮ ﺍﻟﻔﺨﺬ‬
as gross hematuria. Abdominal x-ray discloses a stone in ‫ ﺗﻜﺸﻒ ﺃﺷﻌﺔ ﺍﻟﺒﻄﻦ ﺍﻟﺴﻴﻨﻴﺔ ﻋﻦ ﺣﺼﻴﺔ‬.‫ﻋﻦ ﺑﻴﻠﺔ ﺩﻣﻮﻳﺔ ﻛﺒﻴﺮﺓ‬
the left ureter. Which of the following is true concerning
‫ ﺃﻱ ﻣﻦ ﺍﻟﺘﺎﻟﻲ ﺻﺤﻴﺢ ﺑﺎﻟﻨﺴﺒﺔ ﻟﻬﺬﺍ‬.‫ﻓﻲ ﺍﻟﺤﺎﻟﺐ ﺍﻷﻳﺴﺮ‬
this disease?
A. The majority of renal stones are radiolucent.
:‫ﺍﻟﻤﺮﺽ؟‬
B. Radiolucent stones are usually composed of uric acid. .‫ ﻣﻌﻈﻢ ﺍﻟﺤﺼﻴﺎﺕ ﺍﻟﻜﻠﻮﻳﺔ ﺷﻔﻴﻔﺔ ﻟﻸﺷﻌﺔ‬.A
C. Coraliform calculi are associated with acid urine. ‫ ﺍﻟﺤﺼﻴﺎﺕ ﺍﻟﺸﻔﻴﻔﺔ ﻟﻸﺷﻌﺔ ﻫﻲ ﻋﺎﺩﺓ ﻣﺎ ﺗﺘﺄﻟﻒ ﻣﻦ‬.B
D. Radiopaque stones usually contain cystine. .‫ﺣﻤﺾ ﺍﻟﺒﻮﻝ‬
E. Urate stones are associated with alkaline urine. .‫ ﺗﺘﺮﺍﻓﻖ ﺍﻟﺤﺼﻴﺎﺕ ﻣﺮﺟﺎﻧﻴﺔ ﺍﻟﺸﻜﻞ ﻣﻊ ﺑﻮﻝ ﺣﻤﻀﻲ‬.C
‫ ﺍﻟﺤﺼﻴﺎﺕ ﺍﻟﻈﻠﻴﻠﺔ ﻟﻸﺷﻌﺔ ﺗﺤﺘﻮﻱ ﻋﺎﺩﺓ ﻋﻠﻰ‬.D
Un homme de 28 ans souffre de coliques à l'angle costo- .‫ﺳﻴﺴﺘﻴﻦ‬ B
vertébral gauche, de debut aigu, irradiant dans l'aine, .‫ ﺗﺘﺮﺍﻓﻖ ﺣﺼﻴﺎﺕ ﺍﻟﺒﻮﻻﺕ ﻣﻊ ﺑﻮﻝ ﻗﻠﻮﻱ‬.E
ainsi que d'une hématurie macroscopique. La
radiographie abdominale révèle un calcul dans l'uretère
gauche. Lequel des énoncés suivants est vrai concernant
cette maladie?
A. La majorité des calculs rénaux sont radiotransparents.
B. Les calculs radiotransparents sont généralement
composés d'acide urique.
C.Les calculs coraliformes sont associés à une urine acide.
D. Les calculs radio-opaques contiennent généralement de
la cystine.
E. Les calculs d'urate sont associés à l'urine alcaline.

67 A
163. Cervical cancer can be screened in the population by: ‫ﻳﻤﻜﻦ ﻓﺤﺺ ﺳﺮﻁﺎﻥ ﻋﻨﻖ ﺍﻟﺮﺣﻢ ﻋﻨﺪ ﻣﺠﻤﻮﻋﺔ ﻣﻦ ﺍﻟﺴﻜﺎﻥ‬
a.Clinical history :‫ﺑﻮﺍﺳﻄﺔ‬
b. Clinical examination .‫ ﺍﻟﺴﻴﺮﺓ ﺍﻟﻄﺒﻴﺔ‬.A
c. Cervical Pap smear
.‫ ﺍﻟﻔﺤﺺ ﺍﻟﺴﺮﻳﺮﻱ‬.B
d. ultrasound
e. Pelvic abdominal scan
.‫ ﻟﻄﺎﺧﺔ ﺑﺎﺑﺎ ﻧﻴﻜﻮﻻﻭ ﻟﻌﻨﻖ ﺍﻟﺮﺣﻢ‬.C
.‫ ﺑﻤﻮﺟﺎﺕ ﻓﻮﻕ ﺍﻟﺼﻮﺕ‬.D
.‫ ﻣﺴﺢ ﺿﻮﺋﻲ ﻟﻠﺒﻄﻦ ﻭﺍﻟﺤﻮﺽ‬.E C
Le cancer du col utérin peut être dépisté dans la
population générale par:
a. Histoire clinique
b. Examen clinique
c. Frottis cervical
d. Echographie
e. Scanner abdominal pelvien
164. Which person of the following patients would not be -‫ﺃﻱ ﻣﻦ ﺍﻟﺘﺎﻟﻲ ﻟﻦ ﺗﻜﻮﻥ ﻣﺮﺷﺤﺔ ﻹﺟﺮﺍء ﻟﻄﺎﺧﺔ ﻋﻨﻖ ﺭﺣﻢ‬
candidate for a cervico-vaginal smear :‫ﻣﻬﺒﻠﻴﺔ؟‬
a. 28 years old, G2 P2, mother of 2 children .‫ ﺃﻡ ﻟﻮﻟﺪﻳﻦ‬،G2P2 ،‫ ﻋﺎﻣﺎ‬28 .A
b. 39 years old, single with sexual activity for 5 years
5 ‫ ﻋﺰﺑﺎء ﻣﻊ ﻣﻤﺎﺭﺳﺔ ﻧﺸﺎﻁ ﺟﻨﺴﻲ ﻣﻨﺬ‬،‫ ﻋﺎﻣﺎ‬39 .B
c. 17 years old, having a first intercourse 4 months ago
d. 48 years old, divorced, no sexual activity for 2 years
.‫ﺳﻨﻮﺍﺕ‬
e. 31 years old, one relationship with one partner for 2 .‫ ﺃﺷﻬﺮ‬4 ‫ ﻛﺎﻥ ﺍﻟﺠﻤﺎﻉ ﺍﻷﻭﻝ ﻣﻨﺬ‬،‫ ﻋﺎﻣﺎ‬17 .C
years ‫ ﻣﻄﻠﻘﺔ ﺑﺪﻭﻥ ﻣﻤﺎﺭﺳﺔ ﻧﺸﺎﻁ ﺟﻨﺴﻲ ﻣﻨﺬ‬،‫ ﻋﺎﻣﺎ‬48 .D
.‫ﺳﻨﺘﻴﻦ‬ C
Quelle personne parmi les patientes suivantes ne serait ‫ ﻋﻼﻗﺔ ﻭﺍﺣﺪﺓ ﻣﻊ ﺷﺮﻳﻚ ﻭﺍﺣﺪ ﻣﻨﺬ‬،‫ ﻋﺎﻣﺎ‬31 .E
pas candidate à un frottis cervico-vaginal? ..‫ﺳﻨﺘﻴﻦ‬
a. 28 ans, G2 P2, mère de 2 enfants
b. 39 ans, célibataire avec activité sexuelle depuis 5 ans
c. 17 ans, premier rapport sexuel il y a 4 mois
d. 48 ans, divorcée, pas d'activité sexuelle pendant 2 ans
e. 31 ans, une relation avec un partenaire pendant 2 ans
165. What is the next step in investigating a 26 year old male ‫ ﻋﺎﻣﺎ‬26 ‫ﻣﺎ ﻫﻲ ﺍﻟﺨﻄﻮﺓ ﺍﻟﺘﺎﻟﻴﺔ ﻓﻲ ﺍﻟﺘﺤﺮﻱ ﻋﻨﺪ ﺭﺟﻞ ﻋﻤﺮﻩ‬
with solitary thyroid nodule 2 cm in size? :‫ ﺳﻢ؟‬2 ‫ﻭﻳﻌﺎﻧﻲ ﻣﻦ ﻋﻘﻴﺪﺓ ﺩﺭﻗﻴﺔ ﻭﺣﻴﺪﺓ ﺑﺤﺠﻢ‬
a) Radio Isotope scan .‫ ﺗﺼﻮﻳﺮ ﺑﺎﻟﻨﻈﺎﺋﺮ ﺍﻟﻤﺸﻌﺔ‬.A
b) Thyroid functions (T3,T4, TSH)
.(TSH ،T4 ،T3) ‫ ﻭﻅﺎﺋﻒ ﺍﻟﺪﺭﻕ‬.B
c) USG guided FNAC
d)CT Scan
.USG ‫ ﺑﺎﺳﺘﺮﺷﺎﺩ‬FNAC .C
e) Follow without investigations .‫ ﺗﺼﻮﻳﺮ ﻁﺒﻘﻲ ﻣﺤﻮﺭﻱ‬.D
.‫ ﺍﻟﻤﺘﺎﺑﻌﺔ ﺑﺪﻭﻥ ﺗﺤﺮﻱ‬.E C
Quelle est la prochaine étape dans le bilan d’un homme
de 26 ans avec nodule thyroïdien unique de 2 cm?
a) Radio Isotope scan
b) les fonctions thyroïdiennes (T3, T4, TSH)
c) Aspiration à l’aiguille echoguidee
d) CT Scan
e) Suivi simple sans bilan

68 A
166. Regarding cervical cancer, which of the following points ‫ ﺃﻱ ﻣﻦ ﺍﻟﻨﻘﺎﻁ ﺍﻟﺘﺎﻟﻴﺔ ﺳﺘﻜﻮﻥ‬،‫ﻓﻴﻤﺎ ﻳﺘﻌﻠﻖ ﺑﺴﺮﻁﺎﻥ ﻋﻨﻖ ﺍﻟﺮﺣﻢ‬
would the least interesting in the interview of the patient: :‫ﺍﻷﻗﻞ ﺃﻫﻤﻴﺔ ﻋﻨﺪ ﺍﻟﻤﻘﺎﺑﻠﺔ ﻣﻊ ﺍﻟﻤﺮﻳﻀﺔ؟‬
a. If she had multiple partners .‫ ﺇﺫﺍ ﻛﺎﻥ ﻟﺪﻳﻬﺎ ﻋﺪﺓ ﺷﺮﻛﺎء‬.A
b. If her husband had multiple partners
.‫ ﺇﺫﺍ ﻛﺎﻥ ﻟﺪﻯ ﺯﻭﺟﻬﺎ ﻋﺪﺓ ﺷﺮﻛﺎء‬.B
c. If she smokes
d. The age of the beginning of menstruation
.‫ ﺇﺫﺍ ﻛﺎﻧﺖ ﺗﺪﺧﻦ‬.C
e. If she had similar results in the past .‫ ﺑﺪﺍﻳﺔ ﺳﻦ ﺍﻟﺒﻠﻮﻍ‬.D
.‫ ﺇﺫﺍ ﻛﺎﻥ ﻟﺪﻳﻬﺎ ﻧﺘﺎﺋﺞ ﻣﺸﺎﺑﻬﺔ ﺑﺎﻟﻤﺎﺿﻲ‬.E
D
En ce qui concerne le cancer du col de l'utérus, lequel des
points suivants serait le moins intéressant lors de
l’interrogatoire du patient:
a. Si elle avait plusieurs partenaires
b. Si son mari avait plusieurs partenaires
c. Si elle fume
d. L'âge du début des règles (ménarche)
e. Si elle a eu des résultats similaires dans le passé
167. Which of the following is the most prevalent ‫ﺃﻱ ﻣﻦ ﺍﻟﻌﻀﻮﻳﺎﺕ ﺍﻟﺪﻗﻴﻘﺔ ﺍﻟﺘﺎﻟﻴﺔ ﻫﻲ ﺍﻷﻛﺜﺮ ﺍﻧﺘﺸﺎﺭﺍ ﻓﻲ‬
microorganism in the vagina that may also be protective? ‫ﺍﻟﻤﻬﺒﻞ ﻭﺍﻟﺘﻲ ﻳﻤﻜﻦ ﺃﻥ ﺗﺸﻜﻞ ﺃﻳﻀﺎ ﺍﻟﺰﻣﺮﺓ ﺍﻟﺠﺮﺛﻮﻣﻴﺔ‬
a. α-hemolytic streptococci :‫ﺍﻟﻄﺒﻴﻌﻴﺔ ﻟﻬﺬﻩ ﺍﻟﻤﻨﻄﻘﺔ؟‬
b. Lactobacillus
.α-‫ ﻣﻜﻮﺭﺍﺕ ﻋﻘﺪﻳﺔ ﺣﺎﻟﺔ ﻟﻠﺪﻡ‬.A
c. S. epidermidis
d. E. coli
.‫ ﺍﻟﻌﺼﻴﺎﺕ ﺍﻟﻠﺒﻨﻴﺔ‬.B
e. B. fragilis .‫ ﺍﻟﻤﻜﻮﺭﺍﺕ ﺍﻟﻌﻨﻘﻮﺩﻳﺔ ﺍﻟﺒﺸﺮﻭﻳﺔ‬.C
.‫ ﺍﺷﺮﻳﻜﻴﺎﺕ ﻗﻮﻟﻮﻧﻴﺔ‬.D
.‫ ﺍﻟﻌﺼﻴﺎﺕ ﺍﻟﻬﺸﺔ‬.E B
Lequel des organismes suivants est le microorganisme le
plus répandu dans levagin et qui peut aussi être
protecteur?
a. streptocoques α-hémolytiques
b. Lactobacille
c. S. epidermidis
d. E. coli
e. B. fragilis
168. A person who contracts gonorrhea is most likely to have ‫ ﻳﻜﺘﺴﺒﻬﺎ ﻋﻠﻰ‬، ‫ﺍﻟﺸﺨﺺ ﺍﻟﺬﻱ ﻳﺼﺎﺏ ﺑﺪﺍء ﺍﻟﺴﻴﻼﻥ ﺍﻟﺒﻨﻲ‬
acquired it via the :‫ﺍﻷﺭﺟﺢ ﻋﺒﺮ‬
a. Skin .‫ ﺍﻟﺠﻠﺪ‬.A
b. Gastrointestinal tract
.‫ ﺍﻟﺠﻬﺎﺯ ﺍﻟﻬﻀﻤﻲ‬.B
c. Respiratory tract
d. Genital tract
.‫ ﺍﻟﺠﻬﺎﺯ ﺍﻟﺘﻨﻔﺴﻲ‬.C
e. Nasal tract .‫ ﺍﻟﺠﻬﺎﺯ ﺍﻟﺘﻨﺎﺳﻠﻲ‬.D
.‫ ﺍﻟﺠﻬﺎﺯ ﺍﻷﻧﻔﻲ‬.E D
Une personne qui contracte la gonorrhée a plus de
chances de l’avoir contractée par le biais du
a.Peau
b. Tube digestif
c. Voies respiratoires
d. Tractus génital
e. Voie nasale

69 A
169. A 12-year-old boy presents with eye itching and redness. ‫ ﺍﻟﺘﺼﺮﻳﻒ ﻣﻦ‬.‫ ﻋﺎﻣﺎ ﻳﻌﺎﻧﻲ ﻣﻦ ﺣﻜﺔ ﻭﺍﺣﻤﺮﺍﺭ ﻓﻲ ﺍﻟﻌﻴﻦ‬12 ‫ﻓﺘﻰ ﻋﻤﺮﻩ‬
He has clear drainage from his eyes but no crusting. ‫ ﺍﻟﻔﺤﺺ ﻁﺒﻴﻌﻲ ﺑﺎﺳﺘﺜﻨﺎء ﻣﻠﺘﺤﻤﺔ ﻣﺤﺘﻘﻨﺔ‬.‫ﺍﻟﻌﻴﻨﻴﻦ ﺻﺎﻓﻲ ﻭﻟﻴﺲ ﻫﻨﺎﻙ ﺗﻘﺸﺮ‬
Examination today is normal except for mildly injected :‫ ﺃﻱ ﻣﻦ ﺍﻟﺘﺎﻟﻲ ﻫﻲ ﺍﻟﻤﻌﺎﻟﺠﺔ ﺍﻷﻣﺜﻞ؟‬.‫ﺑﺸﻜﻞ ﻁﻔﻴﻒ ﺛﻨﺎﺋﻴﺔ ﺍﻟﺠﺎﻧﺐ‬
conjunctiva bilaterally. Which of the following is the most .‫ ﻗﻄﺮﺓ ﻣﻀﺎﺩ ﺣﻴﻮﻱ ﻋﻴﻨﻴﺔ‬.A
appropriate treatment ? .‫ ﺍﺳﺘﺸﺎﺭﺓ ﻁﺒﻴﺐ ﻋﻴﻮﻥ‬.B
a- Antibiotic eye drops
.‫ ﻗﻄﺮﺓ ﻋﻴﻨﻴﺔ ﻣﻀﺎﺩﺓ ﻟﻼﻟﺘﻬﺎﺏ‬.C
b- Ophthalmology consultation
.‫ ﻣﺜﺒﻂ ﻟﻴﻜﻮﺗﺮﻳﻴﻦ ﻓﻤﻮﻱ‬.D
c- Anti-inflammatory eye drops
d- Oral leukotriene inhibitor
C
Un garçon de 12 ans présente des démangeaisons et des
rougeurs oculaires. Il a un drainage clair de ses yeux mais
pas de croûte. L'examen d'aujourd'hui est normal, à
l'exception de conjonctive légèrement injectée
bilatéralement. Lequel des éléments suivants est le
traitement le plus approprié?
A- Gouttes d' antibiotiques
B- Consultation ophtalmologique
C- Gouttes anti-inflammatoires
D- Inhibiteur de leucotriène oral
170. A 48-year-old man with no significant medical history ‫ ﺗﺒﻴﻦ ﻣﻦ‬،‫ ﻋﺎﻣﺎ ﺩﻭﻥ ﺳﻴﺮﺓ ﻣﺮﺿﻴﺔ ﺑﺎﺭﺯﺓ ﻭﺩﻭﻥ ﺃﻋﺮﺍﺽ‬48 ‫ﺭﺟﻞ ﻋﻤﺮﻩ‬
and no symptoms is found to have elevated cholesterol at ‫ ﺃﻱ ﻣﻦ ﺍﻟﺘﺎﻟﻲ ﻫﻮ ﺟﺰء ﻣﻦ ﺍﻟﺘﻘﻴﻴﻢ‬.‫ﻓﺤﺼﻪ ﺃﻥ ﻟﺪﻳﻪ ﺍﺭﺗﻔﺎﻉ ﻓﻲ ﺍﻟﻜﻮﻟﺴﺘﺮﻭﻝ‬
a health screening. Which of the following tests is part of :‫ﺍﻟﺮﻭﺗﻴﻨﻲ ﻟﻬﺬﻩ ﺍﻟﻤﺸﻜﻠﺔ؟‬
the routine evaluation of this problem ? .ECG .A
a- ECG
.‫ ﺍﺧﺘﺒﺎﺭ ﺍﻟﺠﻬﺪ‬.B
b- Stress test
.‫ ﺗﻌﺪﺍﺩ ﺩﻡ ﺷﺎﻣﻞ‬.C
c- Complete blood count (CBC)
.‫ ﺍﻟﻬﺮﻣﻮﻥ ﺍﻟﻤﺤﺮﺽ ﻟﻠﺪﺭﻕ‬.D
d- Thyroid-stimulating hormone TSH
D
Un homme de 48 ans sans antécédents médicaux
significatifs et aucun symptôme présente un taux de
cholestérol élevé lors d'un dépistage de santé. Lequel des
tests suivants fait partie de l'évaluation de routine de ce
problème?
A- ECG
B- Test de stress
C- Comptage sanguin complet (CBC)
D- L'hormone stimulant la thyroïde TSH

70 A
171. A 55-year-old woman presents to your office for follow- ‫ ﻟﻘﺪ ﺧﺮﺟﺖ‬.‫ ﻋﺎﻣﺎ ﺣﻀﺮﺕ ﺇﻟﻰ ﺍﻟﻌﻴﺎﺩﺓ ﻣﻦ ﺃﺟﻞ ﺍﻟﻤﺘﺎﺑﻌﺔ‬55 ‫ﺍﻣﺮﺃﺓ ﻋﻤﺮﻫﺎ‬
up. She was discharged from the hospital 1 week ago ‫ ﻟﻘﺪ ﺃﻗﻠﻌﺖ ﻋﻦ‬.‫ﻣﻦ ﺍﻟﻤﺴﺘﺸﻔﻰ ﻣﻨﺬ ﺍﻷﺳﺒﻮﻉ ﺑﻌﺪ ﺗﻌﺮﺿﻬﺎ ﻟﻨﻮﺑﺔ ﻗﻠﺒﻴﺔ‬
following a heart attack. She has quit smoking since then .‫ﺍﻟﺘﺪﺧﻴﻦ ﻣﻨﺬ ﺫﻟﻚ ﺍﻟﺤﻴﻦ ﻭﺗﻌﺪ ﺑﺄﻥ ﺗﺒﻘﻰ ﺑﻌﻴﺪﺓ ﻋﻦ ﺍﻟﺘﺪﺧﻴﻦ ﻣﺪﻯ ﺍﻟﺤﻴﺎﺓ‬
and vows to stay off cigarettes forever. Her lipid levels are ‫ ﺗﺮﻱ‬،‫ ﺩﺳﻞ‬/ ‫ ﻣﻠﻎ‬150 HDL ،‫ ﺩﺳﻞ‬/ ‫ ﻣﻠﻎ‬240 ‫ﺍﻟﻜﻮﻟﺴﺘﺮﻭﻝ ﺍﻟﻜﻠﻲ‬
total cholesterol 240mg/dL, HDL 150mg/dL, and
‫ ﺃﻱ ﻣﻦ ﺍﻟﺘﺎﻟﻲ ﻫﻮ ﺍﻟﺘﺪﺑﻴﺮ ﺍﻷﻣﺜﻞ ﻓﻲ ﻫﺬﺍ‬.‫ ﺩﺳﻞ‬/ ‫ ﻣﻠﻎ‬150 ‫ﻏﻠﻴﺴﻴﺮﻳﺪ‬
triglycerides 150mg/dL. Which of the following is the
:‫ﺍﻟﻮﻗﺖ؟‬
most appropriate management at this time ?
a- Institute therapeutic lifestyle changes alone .‫ ﻣﻌﻬﺪ ﻋﻼﺟﻲ ﻟﺘﻐﻴﻴﺮ ﻧﻤﻂ ﺍﻟﺤﻴﺎﺓ‬.A
b- Institute therapeutic lifestyle changes and start on .‫ ﻣﻌﻬﺪ ﻋﻼﺟﻲ ﻟﺘﻐﻴﻴﺮ ﻧﻤﻂ ﺍﻟﺤﻴﺎﺓ ﻭﺍﻟﺒﺪء ﺑﺴﺘﺎﺗﻴﻦ‬.B
a statin .‫ ﺍﻟﺒﺪء ﺑﺴﺘﺎﺗﻴﻦ‬.C
c- Start on a statin ‫ ﻣﻌﻬﺪ ﻋﻼﺟﻲ ﻟﺘﻐﻴﻴﺮ ﻧﻤﻂ ﺍﻟﺤﻴﺎﺓ ﻭﺍﻟﺒﺪء ﺑﺴﺘﺎﺗﻴﻦ ﻭﺣﻤﺾ‬.D
d- Institute therapeutic lifestyle changes and start on .‫ﺍﻟﻨﻴﻜﻮﺗﻴﻨﻲ‬
statin and nicotinic acid

Une femme de 55 ans présente à votre bureau un suivi.


B
Elle a quitte l'hôpital il y a 1 semaine ou elle a ete admise
suite à une crise cardiaque. Elle a cessé de fumer depuis ce
moment et promet de rester à l'écart des cigarettes pour
toujours. Ses taux de lipides sont le cholestérol total
240mg / dL, HDL 150mg / dL et les triglycérides 150mg /
dL. Lequel des éléments suivants est la gestion la plus
appropriée en ce moment?
A- Instaurer un nouveau style de vie
thérapeutique
B-Instaurer des changements de style de vie
thérapeutique et commencer une statine
C- Commencez une statine
D- Instaurer des changements de style de vie
thérapeutique et commencer la statine et
l'acide nicotinique

71 A
172. A 50-year-old woman with diabetes was recuperating ‫ ﻋﺎﻣﺎ ﺗﻌﺎﻧﻲ ﻣﻦ ﺩﺍء ﺳﻜﺮﻱ ﻭﻛﺎﻧﺖ ﺗﺘﻌﺎﻓﻰ ﻣﻦ ﺗﺮﻣﻴﻢ‬50 ‫ﺍﻣﺮﺃﺓ ﻋﻤﺮﻫﺎ‬
from the left inguinal hernia repair. Her glycosylated ‫ ﺃﺛﻨﺎء ﺍﻟﻤﺘﺎﺑﻌﺔ ﺑﻌﺪ‬.%10 ‫ ﻗﺒﻞ ﺍﻟﻌﻤﻠﻴﺔ ﻛﺎﻥ‬HbA1C .‫ﻓﺘﻖ ﺇﺭﺑﻲ ﺃﻳﺴﺮ‬
hemoglobin (HbA1C) prior to surgery was 10%. During ‫ ﻭﻣﺆﻟﻢ ﻣﻊ ﻭﺟﻮﺩ‬،‫ ﺩﺍﻓﺊ‬،‫ ﻛﺎﻥ ﻣﻮﻗﻊ ﺍﻟﻌﻤﻠﻴﺔ ﺣﻤﺎﻣﻲ ﺑﺸﻜﻞ ﻣﻠﺤﻮﻅ‬،‫ﺍﻟﻌﻤﻠﻴﺔ‬
postoperative follow-up a week after surgery, the surgical :‫ ﺃﻱ ﻣﻦ ﺍﻟﺘﺎﻟﻲ ﻫﻲ ﺍﻟﺨﻄﻮﺓ ﺍﻷﺧﺮﻯ ﻓﻲ ﺍﻟﻌﻼﺝ؟‬.‫ﻗﻴﺢ‬
site was markedly erythematous, warm, and tender with
.‫ ﺗﻄﺒﻴﻖ ﻣﻀﺎﺩ ﺣﻴﻮﻱ ﻣﻮﺿﻌﻲ ﻋﻠﻰ ﻣﻮﻗﻊ ﺍﻟﺠﺮﺍﺣﺔ‬-A
pus. Which of the following is the next step in treatment ?
.‫ ﺿﻤﺎﺩﺍﺕ ﺩﺍﻓﺌﺔ ﻓﻘﻂ ﺳﻮﻑ ﺗﺨﻔﻒ ﻣﻦ ﺍﻻﻟﺘﻬﺎﺏ‬-B
a- Apply topical antibiotic to the surgical site
.‫ ﻓﺘﺢ ﻣﻜﺎﻥ ﺍﻟﺠﺮﺡ ﻭﺗﺼﺮﻳﻒ ﺍﻹﻧﺘﺎﻥ‬-C
b- Warm compresses alone will relieve the
inflammation ‫ ﺇﺭﺳﺎﻝ ﺍﻟﻤﺮﻳﻀﺔ ﺇﻟﻰ ﺍﻟﻤﻨﺰﻝ ﻣﻊ ﻭﺻﻔﺔ ﻟﻤﻀﺎﺩ ﺣﻴﻮﻱ ﻓﻤﻮﻱ‬-D
c- Open the surgical site and drain the infected .‫ ﺃﻳﺎﻡ‬7 ‫ﻟﻤﺪﺓ‬
material
d- Send the patient home with prescription for oral
antibiotics for 7 days

Une femme de 50 ans souffrant de diabète se rétablissait


suite à la réparation d'une hernie inguinale gauche. Son C
taux d'hémoglobine glycosylée (HbA1C) avant
l'intervention chirurgicale était de 10%. Au cours du suivi
postopératoire une semaine après la chirurgie, le site
chirurgical était nettement érythémateux, chaud et tendu
avec du pus. Lequel des éléments suivants est la
prochaine étape du traitement?
A- Appliquer un antibiotique topique sur le site
chirurgical
B- Les compresses chaudes seules vont soulager
l'inflammation
C- Ouvrir le site chirurgical et drainer le matériel
infecté
D- Envoyer la patiente à la maison avec
prescription pour antibiotiques par voie orale
pendant 7 jours
173. A 32 y.o. woman is brought to the ER having reportedly ‫ ﻋﺎﻣﺎ ﺃﺣﻀﻠﺮﺕ ﺇﻟﻰ ﻗﺴﻢ ﺍﻟﻄﻮﺍﺭﺉ ﺑﻌﺪﻣﺎ ﺃﻓﻴﺪ ﻋﻦ ﺗﻨﺎﻭﻟﻬﺎ‬32 ‫ﺍﻣﺮﺃﺓ ﻋﻤﺮﺍ‬
taken a large dose of heroin. She is unresponsive, has ‫ ﻟﺪﻳﻬﺎ ﺗﻀﻴﻖ ﺣﺪﻗﺔ ﻭﺳﺮﻋﺔ‬،‫ ﻫﻲ ﻻ ﺗﺴﺘﺠﻴﺐ‬.‫ﺟﺮﻋﺔ ﻛﺒﻴﺮﺓ ﻣﻦ ﺍﻟﻬﻴﺮﻭﺋﻴﻦ‬
small pupils and has a respiratory rate of 6 breaths/min. :‫ ﺃﻱ ﻣﻦ ﺍﻟﺘﺎﻟﻲ ﻳﺠﺐ ﺍﻋﻄﺎﺅﻩ؟‬.‫ ﺩﻗﻴﻘﺔ‬/ ‫ ﻧﻔﺲ‬6 ‫ﺍﻟﺘﻨﻔﺲ‬
Which of the following should be administered? .‫ ﺃﺗﺮﻭﺑﻴﻦ‬.A
A. Atropine .‫ ﺇﻳﺘﺎﻧﻮﻝ‬.B
B. Ethanol .‫ ﻓﻠﻮﻣﺎﺯﻳﻨﻴﻞ‬.C
C. Flumazenil .‫ﺃﺳﺘﻴﻞ ﺳﻴﺴﺘﺌﻴﻦ‬-‫ ﻥ‬.D
D. N-acetylcysteine .‫ ﻧﺎﻟﻮﻛﺴﻮﻥ‬.E
E. Naloxone
E
Une femme de 32 ans est amenée aux urgencies; la famille
suspecte qu’elle a pris une grande dose d'héroïne. Elle ne
réagit pas, a des pupilles serrées et a une fréquence
respiratoire de 6 respirations / min. Lequel des
traitements suivants doit être administré?
A. Atropine
B. Ethanol
C. Flumazénil
D. N-acétylcystéine
E. Naloxone

72 A
174. Which of the following does not cause T-wave inversion ‫ ﻋﻨﺪ ﺗﺨﻄﻴﻂ ﺍﻟﻘﻠﺐ‬T - ‫ﺃﻱ ﻣﻦ ﺍﻟﺘﺎﻟﻲ ﻻ ﻳﺴﺒﺐ ﺍﻧﻘﻼﺏ ﻓﻲ ﻣﻮﺟﺔ‬
on ECG? :‫ﺍﻟﻜﻬﺮﺑﺎﺋﻲ؟‬
A. Hyperkalemia .‫ ﻓﺮﻁ ﺑﻮﺗﺎﺳﻴﻮﻡ ﺍﻟﺪﻡ‬.A
B. Left bundle branch block .‫ ﺣﺠﺐ ﻓﺮﻉ ﺍﻟﺤﺰﻣﺔ ﺍﻟﻴﺴﺮﻯ‬.B
C. Left ventricular hypertrophy .‫ ﺗﻀﺨﻢ ﺍﻟﺒﻄﻴﻦ ﺍﻷﻳﺴﺮ‬.C
D. Myocardial infarction .‫ ﺍﺣﺘﺸﺎء ﻋﻀﻠﺔ ﻗﻠﺒﻴﺔ‬.D
E. Myocardial ischemia .‫ ﺍﻗﻔﺎﺭ ﻋﻀﻠﺔ ﺍﻟﻘﻠﺐ‬.E
A
Leque lparmi les suivants ne provoque pas l'inversion de
l'onde T surl'ECG?
A. Hyperkaliémie
B. Bloc de branche gauche
C. Hypertrophie ventriculaire gauche
D. Infarctus du myocarde
E. Ischémie myocardique
175. A 48 year old man presents with epigastric pain. It began ،‫ ﻟﻘﺪ ﺑﺪﺃ ﻓﺠﺄﺓ‬.‫ ﻋﺎﻣﺎ ﺣﻀﺮ ﻭﻫﻮ ﻳﺸﻜﻮ ﻣﻦ ﺃﻟﻢ ﺷﺮﺳﻮﻓﻲ‬48 ‫ﺭﺟﻞ ﻋﻤﺮﻩ‬
suddenly, and reached a constant ache which radiates ،‫ ﻟﻘﺪ ﺃﻓﺎﺩ ﻋﻦ ﺣﺪﻭﺙ ﻏﺜﻴﺎﻥ‬.‫ﺣﺘﻰ ﻭﺻﻞ ﺇﻟﻰ ﺃﻟﻢ ﺛﺎﺑﺖ ﻳﻠﺘﻒ ﻧﺤﻮ ﺍﻟﻈﻬﺮ‬
through to the back. He reports nausea, vomiting and has ‫ ﻭﻟﻜﻦ ﺗﺒﻴﻦ‬،‫ ﺃﻳﺎﻡ‬3 ‫ ﻟﻘﺪ ﺃﻓﺎﺩﺕ ﻋﺎﺋﻠﺘﻪ ﺃﻧﻪ ﻛﺎﻥ ﺑﺤﺎﻟﺔ ﻳﺮﻗﺎﻥ ﻣﻨﺬ‬.‫ﻗﻴﺊ ﻭﺇﺳﻬﺎﻝ‬
diarrhoea. His family noted he was jaundice 3 days ago, ،‫ ﻋﻨﺪ ﺍﻟﻔﺤﺺ ﻛﺎﻥ ﻟﺪﻳﻪ ﺍﺭﺗﻔﺎﻉ ﺑﺎﻟﺤﺮﺍﺭﺓ‬.‫ﺃﻥ ﻫﺬﻩ ﺍﻟﺤﺎﻟﺔ ﻗﺪ ﺍﻧﺘﻬﺖ‬
but this appears to have resolved. On examination he is ‫ ﺍﻟﺘﺸﺨﻴﺺ‬.‫ ﻟﺪﻳﻪ ﺳﻴﺮﺓ ﻣﺮﺿﻴﺔ ﻗﺪﻳﻤﺔ ﺑﺎﻟﺮﺑﻮ‬.‫ ﻭﺃﻟﻢ ﺷﺮﺳﻮﻓﻲ‬،‫ﺍﻧﻌﻜﺎﺳﺎﺕ‬
pyrexial, is guarding, and has epigastric tenderness. He : ‫ﺍﻷﺭﺟﺢ ﻫﻮ‬
has a past medical history of Asthma. What is the most .‫ ﺍﻟﺘﻬﺎﺏ ﺯﺍﺋﺪﺓ ﺩﻭﺩﻳﺔ ﺣﺎﺩ‬.A
probable diagnosis .‫ ﺍﻟﺘﻬﺎﺏ ﻣﺮﺍﺭﺓ ﺣﺎﺩ‬.B
.‫ ﺍﻟﺘﻬﺎﺏ ﺑﻨﻜﺮﻳﺎﺱ ﺣﺎﺩ‬.C
a- Acute appendicitis
.‫ ﺍﻟﺘﻬﺎﺏ ﺍﻟﺮﺗﺞ‬.D
b- Acute cholecystitis
.‫ ﺍﺣﺘﺸﺎء ﻋﻀﻠﺔ ﺍﻟﻘﻠﺐ‬.E
c- Acute pancreatitis
d- Diverticulitis
e- Myocardial infarction
C
Un homme de 48 ans se présente avec une douleur
épigastrique qui a commencé subitement, et a atteint un
niveau constant irradiant au dos. Il rapporte des nausées,
des vomissements et une diarrhée. Sa famille a noté qu'il
était ictérique pendant 3 jours, mais cela semble etre
résolu. À l'examen, il est fébrile, avec une defense , et une
sensibilité épigastrique. Il a des antécédents médicaux
d'asthme. Quel est le diagnostic le plus probable
a- appendicite aiguë
b- cholécystite aiguë
c- pancréatite aiguë
d- diverticulite
e-infarctus du myocarde

73 A
176. A 25-year-old male presents to the clinic for evaluation of ‫ ﻟﺪﻳﻪ ﺳﻴﺮﺓ ﺣﻴﺎﺓ‬.‫ ﻋﺎﻣﺎ ﺣﻀﺮ ﺇﻟﻰ ﺍﻟﻌﻴﺎﺩﺓ ﻟﺘﻘﻴﻴﻢ ﺍﻟﻌﻘﻢ‬25 ‫ﺭﺟﻞ ﻋﻤﺮﻩ‬
infertility. He has a life-long history of a productive cough ‫ ﻋﻨﺪ ﻣﺮﺍﺟﻌﺔ‬.‫ﻁﻮﻳﻠﺔ ﻣﻦ ﺍﻟﺴﻌﺎﻝ ﺍﻟﻤﻨﺘﺞ ﻭﺍﻹﻧﺘﺎﻧﺎﺕ ﺍﻟﺮﺋﻮﻳﺔ ﺍﻟﻤﺘﻜﺮﺭﺓ‬
and recurrent pulmonary infections. On his review of ‫ ﻭﺻﻌﻮﺑﺔ ﻓﻲ‬،‫ ﺇﺳﻬﺎﻝ‬،‫ﺃﻋﺮﺍﺿﻪ ﺃﺷﺎﺭ ﺇﻟﻰ ﻣﺸﺎﻛﻞ ﻣﺰﻣﻨﺔ ﻣﻊ ﺃﻟﻢ ﺑﻄﻨﻲ‬
symptoms he has indicated chronic problems with ‫ ﺻﻮﺭﺓ ﺻﺪﺭﻩ ﺍﻟﺸﻌﺎﻋﻴﺔ‬.‫ ﻫﻮ ﺃﻳﻀﺎ ﻳﻌﺎﻧﻲ ﻣﻦ ﺩﺍء ﺳﻜﺮﻱ‬.‫ﻛﺴﺐ ﺍﻟﻮﺯﻥ‬
abdominal pain, diarrhea, and difficulty gaining weight. :‫ ﻣﺎ ﻫﻮ ﺍﻟﺘﺸﺨﻴﺺ ﺍﻷﻛﺜﺮ ﺗﺮﺟﻴﺤﺎ؟‬.‫ﺗﻘﺘﺮﺡ ﺗﻮﺳﻊ ﻗﺼﺒﺎﺕ‬
He also has diabetes mellitus. His chest x-ray suggests .‫ ﺍﻧﺴﺪﺍﺩ ﺭﺋﻮﻱ ﻣﺰﻣﻦ‬.A
bronchiectasis. Which is the most likely diagnosis? .‫ ﺍﻧﺘﺎﻥ ﺟﻬﺎﺯ ﺗﻨﻔﺴﻲ ﻋﻠﻮﻱ‬.B
.‫ ﺗﻠﻴﻒ ﻛﻴﺴﻲ‬.C
a. COPD
.‫ ﻧﺰﻳﻒ ﺭﺋﻮﻱ ﺩﺍﺧﻠﻲ‬.D
b. Upper respiratory infections
.‫ ﺭﺑﻮ‬.E
c. Cystic fibrosis
d. Intrapulmonary hemorrhage
e. Asthma
C
un homme de 25 ans se présente à la clinique pour
évaluation de l'infertilité. Il a une histoire tout au long de
sa vie d'une toux productive et d’infections pulmonaires
récurrentes. A la revue de ces symptômes, il a des
problèmes chroniques de douleur abdominale, de
diarrhée, et une difficulté à prendre du poids. Il a aussi un
diabète sucré. Sa radiographie pulmonaire suggère une
bronchectasie. Quel est le diagnostic le plus probable?
a. BPCO
b. infections des voies respiratoires supérieures
c. Fibrose kystique (mucoviscidose)
d. hémorragie intrapulmonaire
e. Asthme
177. In a patient with thalassemia, peripheral blood film for ‫ ﺃﻅﻬﺮﺕ ﻟﻄﺎﺧﺔ ﺍﻟﺪﻡ ﺍﻟﻤﺤﻴﻄﻲ ﺷﻜﻞ ﺍﻟﻜﺮﻳﺎﺕ‬، ‫ﻋﻨﺪ ﻣﺮﻳﺾ ﺗﻼﺳﻴﻤﻴﺎ‬
red cell morphology shows: : ‫ﺍﻟﺤﻤﺮﺍء‬
a) Hypochromic microcytic cells. . ‫ ﺧﻼﻳﺎ ﻧﺎﻗﺼﺔ ﺍﻟﺼﺒﺎﻍ ﺻﻐﻴﺮﺓ ﺍﻟﻜﺮﻳﺎﺕ‬-A
b) Normochromic normocytic cells. . ‫ ﺧﻼﻳﺎ ﺳﻮﻳﺔ ﺍﻟﺼﺒﺎﻍ ﺳﻮﻳﺔ ﺍﻟﻜﺮﻳﺎﺕ‬-B
c) Macrocytosis. . ‫ ﻛﺒﺮ ﺍﻟﻜﺮﻳﺎﺕ‬-C
d) Sickle cells. . ‫ ﺧﻼﻳﺎ ﻣﻨﺠﻠﻴﺔ‬-D
e) Spherocytes. . ‫ ﻛﺮﻳﺎﺕ ﺣﻤﺮﺍء ﻛﺮﻭﻳﺔ‬-E
A
Chez un patient de thalassémie le frottis sanguin pour la
morphologie cellulaire des globules rouges montre:
a) des cellules hypochromes microcytaires.
b) des cellules normochromes normocytaires.
c) macrocytose.
d) des cellules falciformes.
e) sphérocytes.

74 A
178. A patient presents with history of intermittent fever, ‫ ﺃﻅﻬﺮ ﻓﺤﺺ ﺍﻟﺒﻄﻦ ﺇﻳﻼﻡ‬. ‫ ﺃﻟﻢ ﺑﻄﻨﻲ ﻭﺻﺪﺍﻉ‬، ‫ﻣﺮﻳﺾ ﻳﻌﺎﻧﻲ ﻣﻦ ﺣﺮﺍﺭﺓ‬
abdominal pain and headache. Abdominal examination ‫ ﺍﻟﺪﻭﺍء‬. ‫ ﺯﺭﻉ ﺍﻟﺪﻡ ﺇﻳﺠﺎﺑﻲ ﻟﻠﺴﻠﻤﻮﻧﻴﻠﻼﺍﻟﺘﻴﻔﻴﺔ‬. ‫ﻭﺿﺨﺎﻣﺔ ﻛﺒﺪ ﻭﻁﺤﺎﻝ‬
shows tenderness and hepatosplenomegaly. His blood :‫ﺍﻟﻤﺨﺘﺎﺭ ﻟﻬﺬﺍ ﺍﻟﻤﺮﻳﺾ ﻫﻮ‬
culture is positive for salmonella typhi. The antibiotic of . ‫ ﺳﻴﺒﺮﻭﻓﻠﻮﻛﺴﺎﺳﻴﻦ‬-A
choice for this patient is: . ‫ ﺟﻨﺘﺎﻣﻴﺴﻴﻦ‬-B
a) Ciprofloxacin. . ‫ ﻣﺘﺮﻭﻧﻴﺪﺍﺯﻭﻝ‬-C
b) Gentamycin. . ‫ ﺗﺘﺮﺍﺳﻴﻜﻠﻴﻦ‬-D
. ‫ ﻓﺎﻧﻜﻮﻣﺎﻳﺴﻴﻦ‬-E
c) Metronidazole.
d) Tetracycline.
e) Vancomycin.
A
Un patient se présente avecdes antécédents de fièvre
intermittente, des douleurs abdominales et des maux de
tête. L'examen abdominal montre une légère douleur et
une hépatosplénomégalie. Hémoculture est positive pour
Salmonella typhi. L'antibiotique de choix pour ce patient
est:
a) ciprofloxacine.
b) gentamycine.
c) métronidazole.
d) tétracycline.
e) vancomycine.
179. Ulcerative colitis is a systemic disease involving many ‫ﺍﻟﺘﻬﺎﺏ ﺍﻟﻘﻮﻟﻮﻥ ﺍﻟﺘﻘﺮﺣﻲ ﻫﻮ ﻣﺮﺽ ﺟﻬﺎﺯﻱ ﻳﺘﻀﻤﻦ ﺍﻟﻌﺪﻳﺪ ﻣﻦ ﺍﻟﻤﻈﺎﻫﺮ‬
extraintestinal processes. They include all of the following :‫ ﻫﻲ ﺗﺸﻤﻞ ﻛﻞ ﻣﺎ ﻳﻠﻲ ﻣﺎ ﻋﺪﺍ‬.‫ﺧﺎﺭﺝ ﺍﻹﻣﻌﺎء‬
except: .‫ ﺍﻟﺘﻬﺎﺏ ﺍﻟﻌﻨﺒﻴﺔ‬.A
A. Uveitis .‫ ﺍﻟﺘﻬﺎﺏ ﺍﻷﻗﻨﻴﺔ ﺍﻟﺼﻔﺮﺍﻭﻳﺔ ﺍﻟﻤﺼﻠﺐ‬.B
B. Sclerosing cholangitis .‫ ﺍﻋﺘﻼﻻﺕ ﻣﻔﺼﻠﻴﺔ‬.C
C. Arthropathies .‫ ﺗﻘﻴﺢ ﺍﻟﺠﻠﺪ ﺍﻟﻐﻨﻐﺮﻳﻨﻲ‬.D
D. Pyoderma gangrenosum .‫ ﺣﻤﺎﻣﺔ ﻋﻘﺪﺓ‬.E
E. Erythema nodosum
A
La colite ulcéreuse est une maladie systémique impliquant
de nombreux processus extra-intestinales. Ils
comprennent tous les éléments suivants, sauf:
A. Uvéite
B. La cholangite sclérosante
C. Arthropathies
D. Pyoderma gangrenosum
E. L'érythème noueux
180. In evaluating a lymphoma, which of the following ‫ ﺃﻱ ﻣﻦ ﺍﻟﻨﺘﺎﺋﺞ ﺍﻟﻤﺨﺒﺮﻳﺔ ﺍﻟﺘﺎﻟﻴﺔ ﻏﻴﺮ ﺍﻟﻄﺒﻴﻌﻴﺔ‬،‫ﻋﻨﺪ ﺗﻘﻴﻴﻢ ﺍﻟﻮﺭﻡ ﺍﻟﻠﻤﻔﺎﻭﻱ‬
abnormal laboratory finding indicates a poor prognosis? :‫ﺗﺸﻴﺮ ﺇﻟﻰ ﺳﻮء ﺍﻟﺘﺸﺨﻴﺺ؟‬
A. Elevated uric acid .‫ ﺍﺭﺗﻔﺎﻉ ﺣﻤﺾ ﺍﻟﺒﻮﻝ‬.A
B. Elevated erythrocyte sedimentation rate .‫ ﺍﺭﺗﻔﺎﻉ ﻓﻲ ﺳﺮﻋﺔ ﺗﺮﺳﺐ ﻛﺮﻳﺎﺕ ﺍﻟﺪﻡ ﺍﻟﺤﻤﺮﺍء‬.B
C. Elevated LDH .LDH ‫ ﺍﺭﺗﻔﺎﻉ‬.C
D. Elevated ALT (SGPT) .(SGPT) ALT‫ ﺍﺭﺗﻔﺎﻉ‬.D
E. Elevated glucose .‫ ﺍﺭﺗﻔﺎﻉ ﻏﻠﻮﻛﻮﺯ‬.E

C
Dans l'évaluation d'un lymphome, lesquel des anomalies
de résultat de laboratoire suivants indique un mauvais
pronostic?
A. Une concentration élevée d’ acide urique
B. Vitesse de sedimentation élevée
C. LDH élevé
D. ALAT (SGPT) élevé
E. Glycémie élevée

75 A

Vous aimerez peut-être aussi